LSAT 2016

PART II LSAT Practice Tests

LSAT Practice Test 1

You can also take this practice test on your tablet or smartphone as well as your laptop or home computer. See page 7A of the Welcome insert for more information.

Answer Sheet

Directions: Before beginning the test, photocopy this answer sheet or remove it from the book. Mark your answer to each question by filling in the corresponding answer oval in the columns below. If a section has fewer questions than answer spaces, leave the extra spaces blank.

Section I

1. Images

2. Images

3. Images

4. Images

5. Images

6. Images

7. Images

8. Images

9. Images

10. Images

11. Images

12. Images

13. Images

14. Images

15. Images

16. Images

17. Images

18. Images

19. Images

20. Images

21. Images

22. Images

23. Images

24. Images

25. Images

26. Images

27. Images

28. Images

29. Images

30. Images

Section II

1. Images

2. Images

3. Images

4. Images

5. Images

6. Images

7. Images

8. Images

9. Images

10. Images

11. Images

12. Images

13. Images

14. Images

15. Images

16. Images

17. Images

18. Images

19. Images

20. Images

21. Images

22. Images

23. Images

24. Images

25. Images

26. Images

27. Images

28. Images

29. Images

30. Images

Section III

1. Images

2. Images

3. Images

4. Images

5. Images

6. Images

7. Images

8. Images

9. Images

10. Images

11. Images

12. Images

13. Images

14. Images

15. Images

16. Images

17. Images

18. Images

19. Images

20. Images

21. Images

22. Images

23. Images

24. Images

25. Images

26. Images

27. Images

28. Images

29. Images

30. Images

Section IV

1. Images

2. Images

3. Images

4. Images

5. Images

6. Images

7. Images

8. Images

9. Images

10. Images

11. Images

12. Images

13. Images

14. Images

15. Images

16. Images

17. Images

18. Images

19. Images

20. Images

21. Images

22. Images

23. Images

24. Images

25. Images

26. Images

27. Images

28. Images

29. Images

30. Images

SECTION I

Time—35 minutes

25 Questions

Directions: The questions in this section are based on the reasoning contained in brief statements or passages. For some questions, more than one of the choices could conceivably answer the question. However, you are to choose the best answer; that is, the response that most accurately and completely answers the question. You should not make assumptions that are by commonsense standards implausible, superfluous, or incompatible with the passage. After you have chosen the best answer, blacken the corresponding space on your answer sheet.

Questions 1–2

Jamey:   Scientists recently trained a bacterium to survive and grow on a diet of arsenic rather than phosphorus, which is one of the six required elements to sustain life in our universe. Based on their results, there may be life in the universe that does not require the same elements that we’ve assumed were necessary for life. This proves that we need to expand our search parameters for life on other planets.

Lorne:   No, the bacterium survived on arsenic, but it grew much faster when fed phosphorus. Thus, your conclusion seems unwarranted and to invest significant effort in expanding our search parameters for alien life forms would be a waste of scientific resources.

1. Which one of the following, if true, provides the strongest defense of Jamey’s argument against Lorne’s criticism?

(A) The bacterium, while on a diet of arsenic, had a life span only half that of a bacterium on a diet of phosphorus.

(B) The bacterium, while on a diet of arsenic, showed signs that other elements necessary for life’s existence, like carbon, were toxic to it.

(C) Infrared satellite images of planets rich in phosphorus exhibit only four of the five other elements necessary to sustain life.

(D) Scientists have proven that due to exposure to arsenic over long periods of time other organisms can develop a tolerance to small amounts of the poison.

(E) Infrared satellite images of planets rich in arsenic and devoid of phosphorus have shown no signs of conditions that could foster life.

2. Which one of the following most accurately describes Lorne’s criticism of Jamey’s argument?

(A) It argues that while Jamey’s facts are correct her conclusion does not follow from those facts.

(B) It provides an alternative set of facts that, while it does not completely invalidate Jamey’s set of facts, may undermine her conclusion.

(C) It presents new information that disproves Jamey’s conclusion.

(D) It offers facts irrelevant to Jamey’s argument in an attempt to challenge her credibility.

(E) It points out Jamey’s conclusion is too far of a leap in logic to be supported by her facts.

3. Public official:   Hospital admissions for dog bites has doubled over a 15-year period. This increase has outstripped considerably the growth in population and the increase in pet ownership over the same period. In addition, the average cost of treatment for dog bites is more than $18,000. We’re at a loss to explain this trend but one thing is clear: the high cost of treating these bites means that dog owners should be held legally accountable for their pets’ attacks on others.

Which one of the following, if true, most strongly undermines the public official’s conclusion?

(A) The high cost of treating dog bites can be attributed mostly to infections contracted in the hospital.

(B) The lack of growth in pet ownership can be attributed to a significant decrease in cat ownership over the 15-year period.

(C) Most dog bite incidents occur between the pet and its owner.

(D) The court costs required to litigate pet ownership liability cases would far outstrip the health costs of dog bites.

(E) The statistics do not take into account the slight reduction in number of smaller clinics that used to handle most dog bites before the 15-year period.

4. The eyes of the female cichlid fish feature five photoreceptor cones, the most of any vertebrate and two more than the three that human eyes have. The dull-colored female cichlid fish uses its highly discriminating color vision to locate the more colorful male cichlids so that it may successfully choose a mate. If the color of the male’s scales is altered the female can unknowingly bypass the male altogether. Increased water turbidity due to deforestation and economic development has altered the visual environment of fish. Thus, it will be difficult for the female cichlids to find mates and the survival of the cichlid population will be seriously threatened.

Which one of the following, if true, LEAST strengthens the argument above?

(A) An experiment showed that the eyes in some varieties of cichlids adapted when the fish were switched between blue to green waters.

(B) The female cichlid is incapable of distinguishing certain shades of red in its mates when the light permeating the water diminishes in the evening.

(C) The tilapia, a type of cichlid, has shown a marked decrease in population in a lake near an industrial park near Chicago.

(D) Turbidity in the waters has caused various non-cichlid species to move to other bodies of water, leaving the cichlids a more open mating environment.

(E) The discriminating vision of the female cichlid has been found to make it more susceptible to eye-borne diseases that threaten the cichlid population.

5. Coffee houses in City A have experienced a decrease in coffee revenues due to a downturn in the economy of City A, so much so that some of the most popular coffee houses may have to close. At the same time, however, more people are visiting coffee houses in City A than ever before and the number of successful coffee houses in City A is greater than ever.

Which one of the following, if true, most helps to resolve the apparent discrepancy in the information above?

(A) More coffee houses opened in City A during the last year have survived than in any previous year.

(B) Because of the economic downturn, people of City A are brewing their own coffee at home rather than buying it at coffee houses.

(C) The patronage for a popular bakery far exceeds the patronage of even the most popular coffee house.

(D) Coffee is the main source of revenue only for the most popular coffee houses; other coffee houses rely on pastries and other food items for revenue.

(E) Most coffee houses lower their prices for coffee during an economic downturn.

Questions 6–7

Manager:   Last year the total number of dropped cell phone calls on our network of cell towers decreased, so our effort to increase cell tower effectiveness worked.

Technician:   If you look at the statistics for individual cell towers, however, you find that the number of dropped calls actually increased substantially for every tower that was in operation both last year and the year before. The number of dropped calls has clearly increased, given that the set of towers, the only ones for which we have dropped-call figures that allow a comparison between last year and the year before, demonstrate a trend toward more dropped calls.

6. If the dropped-call figures cited by both the manager and the technician are accurate, which one of the following must be true?

(A) The market share of the company’s cell phone service fell last year.

(B) The company erected at least one new cell tower in the last two years.

(C) The volume of the company’s cell phone calls has changed over the last two years.

(D) The quality of the company’s cell tower system has not improved over the last two years.

(E) The company’s cell phone calls are of lower quality than they once were.

7. Which one of the following, if true, most seriously calls into question the technician’s argument?

(A) Most of the company’s cell towers that were in operation throughout last year and the year before are located in areas where cell phone usage overall skyrocketed last year.

(B) Discounts to the heavier cell phone users did not increase cell phone usage.

(C) Prior to last year there was an overall downward trend in cell tower traffic.

(D) Those cell towers that experienced a decrease in dropped calls were not positioned in high-traffic areas.

(E) The company’s cell towers last year handled less traffic from the company’s other wireless services.

8. Professor:   To define what is considered literature is futile. Any definition of what constitutes literature would either exclude some works that are, in fact, literature or include others that might not be worthy of the designation. The notion of music and art are equally problematic. This is why the viability of a humanities department at our university should be seriously questioned.

The reasoning in the argument is questionable because the argument

(A) is advanced by someone who has a vested interest in eliminating a humanities department

(B) makes the unsupported claim that defining literature is futile

(C) fails to show any specific link between the indefinability of the arts such as literature and the maintenance of a humanities department

(D) ignores the fact that some people view literature as definable

(E) generalizes from an unrepresentative sample to all literature

9. New research studies showed that there is a higher rate of stroke in the southern states than the rest of the country. The studies also found that people in these states eat more fried fish and less non-fried fish than the rest of the country. We can conclude from this that eating fried fish increases one’s chances of suffering a stroke. Therefore, people who want to reduce their risk of suffering a stroke should not eat fried fish.

The flawed pattern of reasoning exhibited by which one of the following is most similar to that exhibited by the argument above?

(A) People who drink more than two ounces of alcohol a day tend to suffer more recurring illnesses. Since excessive alcohol drinking increases one’s chances of incurring liver disease, people who want to avoid liver disease should give up drinking alcohol.

(B) Studies show that people who eat a diet that includes foods with high levels of antioxidants show a lower incidence of cancer. This proves that antioxidants prevent the occurrence of cancer, so the government should require that food producers increase the antioxidant content of their foods.

(C) Studies have shown that people who live near towers that emit powerful electromagnetic fields experience higher incidents of cancer, so the government should prohibit home construction close to electromagnetic field–emitting towers.

(D) The majority of people who wear their seat belts when involved in an auto accident avoid serious injury. It has also been shown that drunk drivers who cause an auto accident are less likely to be as seriously injured as the other people involved in the accident, whether or not the other people wear seat belts. Therefore, to reduce health care costs due to accidents, we should hire more traffic police to enforce the seat belt laws.

(E) The majority of people who participate in vigorous exercise experience less occurrence of illnesses beyond the age of 50. This proves that vigorous exercise is important to good health as one grows older, so those who want to avoid illnesses should participate in vigorous exercise.

10. Mr. Thomas:   Ms. Garcia said that she gave up her sales responsibilities and was focusing only on her administrative duties. But I overheard a colleague of hers say that Ms. Garcia is working harder than ever before and will be gone for much of the next year visiting customers; that does not sound like she has given up her sales responsibilities to me. At least one of them is not telling the truth.

Mr. Thomas’s reasoning is flawed because it

(A) fails to consider that Ms. Garcia’s colleague may have been deceived by her

(B) draws a conclusion on equivocal language

(C) is based partly on hearsay

(D) fails to infer that Ms. Garcia may be a person of poor character, given her lack of ambition

(E) criticizes Ms. Garcia rather than the claims she made

11. David:   The quality of Company A’s management is declining in its ability to provide effective leadership to its employees and to generate high growth. There must be a return to the company’s traditional values of risk and innovation.

Talia:   There is no need to switch gears. Revenue growth might not be what it used to be, but most managers consider the current growth rate acceptable. Expectations for growth are unrealistic for this stage of the company’s history.

David and Talia disagree over whether corporate growth

(A) can be changed

(B) is decreasing or increasing

(C) is perceived by management the same as employees

(D) is adequate as it is

(E) is changing over time

12. In defending the new toy-manufacturing regulations aimed at protecting children from harmful defects in toys, the government regulator in charge of enforcing the rules pointed to recent success in catching defects: inspections discovered 60 percent more defects among toys produced under the new regulations than under the previous rules.

Which one of the following, if true, most seriously undermines the defense offered by the regulator?

(A) The defects were caught mostly in toys aimed at children aged one to four years.

(B) The rules cause manufacturing companies to incur additional costs that hurt profitability.

(C) The quality of toys manufactured domestically is higher than that of toys produced by foreign manufacturers.

(D) The increase in discovered defects did not decrease the number of incidents where children were hurt by defective toys.

(E) The regulations are more stringent than those enforced by similarly industrialized nations.

13. Economist:   The vast majority of nations that have a low corporate tax rate tend to run high budget deficits, but some nations with a high corporate tax rate also run a high budget deficit. What all nations that run high budget deficits have in common, however, is that they are saddled with large entitlement programs.

If all of the economist’s statements are true, which one of the following must also be true?

(A) Some nations with large entitlement programs do not have a high budget deficit.

(B) The majority of nations with large entitlement programs have a low corporate tax rate.

(C) Some nations with large entitlement programs have a high corporate tax rate.

(D) Every nation with large entitlement programs has a low corporate tax rate.

(E) Fewer high corporate tax rate nations than low corporate tax rate nations have large entitlement programs.

14. Studies indicate that the rate at which soil erosion is increasing is leveling off: the amount of soil lost to erosion this year is almost identical to the amount of soil lost to erosion last year. If this trend continues, soil erosion will no longer be getting more serious.

The reasoning is questionable because it ignores the possibility that

(A) the effects of soil erosion are cumulative

(B) crop-destroying insects and climate change are becoming more serious

(C) the leveling-off trend of soil erosion will not continue

(D) some soil erosion has no noticeable effect on farm productivity

(E) the areas of soil erosion this year were less critical than those that occurred last year

15. Larry:   The federal government audited our books and assessed our company with a significant fine for a minor tax law infraction. Other companies in our industry were not audited even though we know for certain that they have committed the same infraction. This is unfair treatment by the federal government.

Elizabeth:   You were not treated unfairly, since the federal government cannot afford to audit every company that it thinks is breaking the law. Each company in your industry that broke the law had an equal chance of being audited.

Which one of the following principles, if established, would most help to justify Elizabeth’s position?

(A) It is fairer not to enforce a tax law at all than to enforce it in some, but not all, of the cases to which it applies.

(B) Fairness in the application of a tax law is ensured not by all violators’ having an equal chance of being fined for their violation of the law, but rather by fining all known violators to the same extent.

(C) The fines attached to tax laws should be imposed on all people who violate those laws, and only those people.

(D) If all companies that broke a tax law in a given year are equally likely of being audited and fined for breaking it, then the law is fairly applied to whomever among them is then fined.

(E) The fines attached to breaking the tax code should be assessed not as punishments for breaking the law but rather as deterrents for unethical bookkeeping.

16. After purchasing a trailer for his truck from a car dealer in Abilene, Jason was informed by an Abilene city official that he would not be allowed to keep his trailer, since the city codes consider a trailer an industrial vehicle, and individuals cannot park industrial vehicles within the city of Abilene.

The city official’s argument depends on assuming which one of the following?

(A) Trailers are not classified as residential vehicles in Abilene.

(B) It is legal for car dealers to sell trailers in Abilene.

(C) Vans and flatbed trucks are not classified as industrial vehicles.

(D) Any vehicle not classified as an industrial vehicle may be parked in Abilene.

(E) Jason lives in Abilene.

17. Energy expert:   It is true that over the past 10 years there has been a fivefold increase in government funding for the development of alternative energy technologies, while the practical applications in use from this research have increased alternative energy usage by a meager 10 percent. Even when inflation is taken into account, the amount of funding now is at least two times what it was 10 years ago. Nevertheless, current government funding for developing alternative energy technologies is inadequate and should be increased.

Which one of the following, if true, most helps to reconcile the energy expert’s conclusion with the evidence cited above?

(A) More people today, engineers and non-engineers, are working to develop alternative energy technologies.

(B) The idea that the government should develop alternative energy technologies was more popular 20 years ago than it is today.

(C) Research over the past 10 years has enabled engineers today to identify new alternative energy technologies faster than was thought possible 10 years ago.

(D) Over the past 10 years, the salaries of engineers employed by the government to work on alternative energy technologies have increased at a rate higher than the inflation rate.

(E) The government agency responsible for administering alternative energy funds has been consistently mismanaged and run inefficiently over the past 10 years.

18. Historian:   Anyone who thinks that the extreme military tactics of the ancient empire of T were the product of a warmongering political class is overlooking the basic truth: the political class was made up primarily of intellectuals with a vision of a more civilized world. Empire T conquered and slaughtered many tribes in pursuit of its goals; but it later became clear that the new world, as the empire defined it, was unachievable. So at least some of the ordinary people of T were in fact murderers.

Which one of the following principles, if valid, provides the most support for the historian’s argument?

(A) Aggressiveness in pursuit of what is eventually found to be unachievable constitutes warmongering.

(B) The pursuit of a civilized world justifies warmongering.

(C) The pursuit of a civilized world does not justify warmongering.

(D) Warmongering in pursuit of a civilized world constitutes inhumanity.

(E) Conquest in pursuit of what is later found to be an unachievable vision constitutes murder.

Questions 19–20

The sudden boom in business startups in a certain region’s economy is said to be causing people not even employed by these businesses to spend more freely, as if they too were gaining economic power. Clearly, however, actual spending by such people is remaining unchanged, because there has been no unusual decrease in the amount of money held by those people in savings accounts.

19. The argument in the passage proceeds by doing which of the following?

(A) arguing that because two alternative developments exhaust all the plausible possibilities, one of those developments happened and the other did not

(B) arguing that because people’s economic behavior is guided by economic self-interest, only misinformation or error will cause people to engage in economic actions that harm them economically

(C) concluding that because only one of the two predictable consequences of a certain sort of behavior is observed to happen, this observed phenomenon cannot, in the current situation, be a consequence of such behavior

(D) concluding that because a probable consequence of a supposed development failed to occur, that development itself did not occur

(E) concluding that because the evidence concerning a supposed change is ambiguous, it is most likely that no change is actually occurring

20. Which one of the following is an assumption on which the argument relies?

(A) There exist no statistics for sales of goods in the region.

(B) People in the region who are not employed by business startups and who have relatives employed by startups commonly borrow money from those relatives to make large purchases.

(C) If people in the region who are not employed by business startups are making large purchases, they are not paying for them by taking on debt.

(D) People in the region not employed by startups are optimistic about their prospects for increasing their incomes.

(E) If people in the region who are employed by startups change jobs, the new jobs generally pay more than the ones they lost.

21. Criminal actions, like all actions, are ultimately products of a natural predisposition. It is not the criminals or their environment but the inherent flaws in human nature that create and maintain the criminal element in society—it is simply human nature. Therefore, the prison system should be abolished immediately.

What is the assumption upon which the argument depends?

(A) It is the nature of humanity and nothing else that should be held responsible for crime.

(B) There is an ambiguity inherent in the term nature.

(C) The statistical evidence is drawn from only a small segment of the population.

(D) Someone becomes a criminal solely by virtue of having committed a crime.

(E) Some actions are socially acceptable and some are not.

22. Each December 31 in Country B, a tally is made of the country’s total available natural gas inventory—that is, the total amount of natural gas that has been produced throughout the country but not consumed. In 2010 that amount was considerably lower than it had been in 2009. Furthermore, Country B has not imported or exported natural gas since 1990.

If the statements above are true, which one of the following must also be true on the basis of them?

(A) In Country B, the amount of natural gas consumed in 2009 was greater than the amount of natural gas consumed in 2010.

(B) In Country B, more natural gas was mined in 2009 than was mined in 2010.

(C) In Country B, more natural gas was consumed during the first half of 2010 than was consumed during the first half of 2009.

(D) In Country B, the amount of natural gas consumed in 2010 was greater than the amount of natural gas mined in 2010.

(E) In Country B, the amount of natural gas consumed in 2010 was greater than the amount of natural gas consumed in 2009.

23. Richard:   Literary tools such as “prefaces” and “prologues,” unlike the novel itself, serve no purpose.

Jeri:   I agree; and since such literary tools are meaningless they should be eliminated from literature.

Jeri’s remarks indicated that she interpreted Richard’s statement to imply that

(A) literary tools that are not useful are meaningless

(B) only literary tools that serve a purpose have meaning

(C) literary tools that serve a purpose are meaningful

(D) all literary tools that serve a purpose are useful

(E) if a literary tool does not serve a purpose, it should be eliminated from literature

24. Joanie:   Property losses due to natural disasters have increased significantly. Also, fatalities due to natural disasters over the last two years have skyrocketed. It is obvious that the rate of natural disasters is on the increase.

Lonnie:   I agree with your facts but not with your inference. Increased development has given natural disasters more to destroy. Also, over the last two years, the world has experienced some unusually large-scale natural disasters.

Lonnie criticizes Joanie’s argument by pointing out

(A) that the rate of natural disasters is a difficult statistic to quantify

(B) that statistics, when it comes to natural disasters, are notoriously untrustworthy

(C) that the trends portrayed by the statistics on natural disasters do not represent the underlying causes that lead to her conclusion

(D) her lack of qualifications to make such a judgment regarding the rate of natural disasters

(E) other statistics that support the opposite conclusion

25. A political leader is considered great by assessing the decisions and actions that the political leader performed while in office. An inventory of the leaders’ decisions and actions is the only indicator of greatness. Therefore, to say that a political leader is great is just to summarize the quality of his or her known decisions and actions, and the leader’s greatness can provide no basis for predicting the quality of the leader’s future decisions or actions.

Which one of the following arguments contains reasoning that is most similar to that used in the argument above?

(A) The bacteria that cause ear infections are not all the same, and they differ in their effects. Therefore, although it may be certain that a child has an ear infection, it is impossible to predict how the infection will progress.

(B) Although ear infections are very common among children, there are some children who never or only very rarely develop an ear infection. Clearly these children must be in some way physiologically different from other children who develop ear infections frequently.

(C) The only way of knowing whether a child has an ear infection is to observe symptoms. Thus, when a child is said to have an ear infection, this means only that the child has displayed the symptoms of having an ear infection, and no prediction about the patient’s future symptoms is warranted.

(D) Unless a child displays ear infection symptoms, it cannot properly be said that the child has an ear infection. But each of the symptoms of an ear infection is also the symptom of another disease. Therefore, one can never be certain that a child has an ear infection.

(E) A child’s ear infection is caused by bacteria trapped by a blocked eustachian tube in the ear. Each type of bacteria is different and the aperture of a child’s eustachian tube changes constantly depending on the weather. Therefore, it is not possible to predict from a child’s history of infection how susceptible he or she will be in the future.

STOP

IF YOU FINISH BEFORE TIME RUNS OUT, CHECK YOUR WORK ON THIS SECTION ONLY. DO NOT GO ON TO ANY OTHER TEST SECTION.

SECTION II

Time—35 minutes

25 Questions

Directions: The questions in this section are based on the reasoning contained in brief statements or passages. For some questions, more than one of the choices could conceivably answer the question. However, you are to choose the best answer; that is, the response that most accurately and completely answers the question. You should not make assumptions that are by commonsense standards implausible, superfluous, or incompatible with the passage. After you have chosen the best answer, blacken the corresponding space on your answer sheet.

1. Reynolds Dairy is attempting to dominate the organic yogurt market by promoting its nonfat “Slim-gurt,” its most popular line of yogurt, with a costly new sweepstakes giveaway. But market research shows that, in the opinion of 76 percent of all consumers, “Slim-gurt” already dominates the market. Since any product with more than 55 percent of the sales in a market is, by definition, dominant in that market, Reynolds Dairy dominates the market now and need only maintain its current market share in order to continue to do so.

Which one of the following is an assumption on which the argument above depends?

(A) Market research studies describing the current situation are able to predict the future as well.

(B) What the consumer believes to be true about a product is in fact the true conditions in the marketplace.

(C) Consumers’ belief in market dominance is required for a company’s product to actually achieve market dominance.

(D) Market research studies are thorough and accurate and it is foolish for a company to ignore the results of a study that counters its strategy in the marketplace.

(E) Consumers’ perception of market dominance actually results in popular demand for a product.

2. Every school of psychology has a theory as to how we can achieve happiness. However, most people would judge someone who follows every prescription for happiness given by one of these theories to not be truly happy—not the life they would want for themselves or their children.

The statements above, if true, most strongly support which one of the following?

(A) It is impossible to develop a psychological theory that accurately describes happiness.

(B) A person who achieves what is necessary under one theory of happiness does not necessarily achieve happiness under another theory.

(C) Happiness as defined by psychology is unachievable in practice.

(D) Most people do not know what is truly necessary to achieve happiness.

(E) Most people do not have a conception of happiness that matches that defined by any school of psychology.

Questions 3–4

Aligorian Wildlife Defense League:   Uncontrolled logging in industrialized nations has endangered many species. To protect wildlife we must regulate the logging industry and its activities in Aligorian; future forest development must be offset by the development of replacement forest habitats. Thus, development would cause no net reduction of forests and pose no threat to the species that inhabit them.

Aligorian Logging Commission:   Other nations have flagrantly developed forest areas at the expense of wildlife. We have conserved. Since Aligorian logging might not affect wildlife and is necessary for economic growth, we should allow logging to proceed unfettered. We have as much right to govern our own resources as countries that have already put their natural resources to commercial use.

3. Which one of the following is an assumption on which the argument advanced by the Aligorian Logging Commission depends?

(A) The species that inhabit Aligorian forests are among the most severely threatened of the designated endangered species.

(B) Aligorian regulation of logging has in the past protected and preserved wildlife.

(C) In nations that are primarily agricultural, logging does not need to be regulated.

(D) The species indigenous to natural forests will survive in specially constructed replacement forest habitats.

(E) More species have been endangered by the uncontrolled logging of forests than have been endangered by any other type of development.

4. Which one of the following principles, if accepted, would most strongly support the Aligorian Logging Commission’s position against the Aligorian Wildlife Defense League’s position?

(A) Regulation should be implemented to prevent further damage only when a reduction of populations of endangered species by logging has been found.

(B) The economic needs of individual nations are not as important as the right of future generations to have wildlife preserved.

(C) It is careless to allow further depletion of natural resources.

(D) Environmental regulation must aim at preventing further environmental damage and cannot consider the different degrees to which different nations have already harmed the environment.

(E) When wildlife is endangered, national resources should be regulated by international agreement.

5. Goni:   You cannot blame me for the tax penalty we incurred. Even though the penalty was assessed due to my error, you know full well that the error was due to my poor handwriting, and I certainly cannot be held responsible for the fact that my handwriting has deteriorated with age.

Avi:   But I can hold you responsible for your poor bookkeeping, because you know how poor your handwriting is. People are responsible for the consequences of actions that they voluntarily undertake, if they know that those actions risk such consequences.

The principle that Avi invokes, if established, would justify which of the following judgments?

(A) Jack was not responsible for losing his board position, because, knowing that his position was vulnerable in the election, he did everything possible to preserve it.

(B) Jack was responsible for missing his daughter’s dance recital, because he decided to hold a meeting with his staff late in the workday even though he knew the meeting might go long and he might miss the recital.

(C) Jack was responsible for his dog being frightened, because, even though it was his brother who set off the firecracker, he knew that dogs are often frightened by loud noises.

(D) Jack was responsible for having offended his friend when he made an offensive comment about her colleague, although he did not know his friend would mistakenly understand the comment to be about herself.

(E) Jack was responsible for his bicycle being stolen last month, because he did not take any of the precautions that the city police recommended in the antitheft pamphlet they published this week.

6. Editorial:   The governor’s chief prosecutor assures the governor that with a thorough investigation and ample time to assemble the evidence he can root out political corruption at all levels of the state government. But the governor should not listen to the prosecutor, who in his youth was convicted of fraud. Surely his legal advice is as untrustworthy as he is himself, and so the governor should discard any hope of rooting out corruption and simply do the best he can to execute the state’s business despite such hindrances.

Which one of the following is a questionable argumentative strategy employed in the editorial?

(A) trying to win support for a plan by playing on people’s fears of what could happen if not implemented

(B) rejecting a plan on the grounds that a particular aspect of the plan is likely to fail

(C) presupposing what the plan sets out to prove

(D) criticizing the source of a plan rather than examining the plan itself

(E) taking lack of evidence for that a plan has worked in the past as proof that the plan will fail

7. Lawyer:   Consider any situation where a moral injustice is committed. There are always situations in which it is right to protest the nature of that injustice. So, there are always situations in which it is right to protest taxation without representation.

The conclusion of the argument follows logically if which one of the following is assumed?

(A) Taxation without representation can always be protested.

(B) Some kinds of injustices are dangerous to protest.

(C) An injustice is of a moral nature only if it can be protested.

(D) All injustices that can be protested are of a moral nature.

(E) Taxation without representation is a moral injustice.

8. Arisha:   Our city’s campaign to persuade homeowners to reduce their garbage and increase recycling should be implemented in your city. In the 18 months since the enactment of the legislation authorizing the campaign, the volume of garbage dumped at our landfill has decreased by 30 percent and our recycling centers have increased capacity.

Brandon:   But the major shift to recycling occurred during the first six months after that legislation was enacted, right after your city’s new recycling centers opened, and before the recycling campaign even got started.

In responding to Arisha, Brandon does which one of the following?

(A) He presents evidence to suggest that recycling campaigns are usually ineffective.

(B) He advances the claim that a campaign such as Arisha recommends is not necessary because most homeowners recycle.

(C) He introduces evidence to show that the campaign Arisha advocates is only effective for a short period of time.

(D) He cites facts that tend to weaken the force of the evidence with which Arisha supports her recommendation.

(E) He denies Arisha’s assumption that Brandon’s city wants to reduce garbage and increase recycling.

9. Most people who use their cell phone more than 10 times a day do not have a landline phone installed in their home. In Ronde County most people used their cell phone more than 10 times a day. Therefore, in Ronde County most people do not have a landline phone installed in their home.

Which one of the following arguments has a flawed pattern of reasoning most like the flawed reasoning in the argument above?

(A) It is clear that most citizens in Eastland are law abiding since there is a very low crime rate in Eastland and most crimes are not very serious.

(B) It is clear that Halley cannot cook, since she does not own a pot or pan and no one in her family who does not own a pot or pan can cook.

(C) It is clear that Alvarez’s friends usually carpool to work, since all of his friends can drive and all of his friends go to work.

(D) It is clear that most people in Daytown vote in early voting, since most people who live in retirement communities vote in early voting and most people in Daytown live in retirement communities.

(E) It is clear that most of Richard’s friends love jazz, since he only goes to concerts with people who love jazz and he goes to concerts with most of his friends.

10. Current sales maps showing the areas of the United States where our products are most likely to secure loyal customers are based on sales data gathered 10 years ago from a small number of regional salespeople. New maps are being compiled using sales data from hundreds of retail centers and input from a network of loyal customers. These sales maps will be much more useful.

Each of the following, if true, helps to support the claim that the new maps will be more useful EXCEPT:

(A) Market information has changed in the past 10 years.

(B) Sales information is the most important factor in determining whether customers will become loyal.

(C) Some of the retail centers have been open more than 10 years.

(D) Customers can provide information on the market that retail centers cannot.

(E) Sales information can be described more accurately when more information is available.

11. Editorialist:   TV dramas are contributing to an increase of violence in our society. By constantly being shown violent crimes such as murder and assault, viewers begin to think such actions are the norm and that there is something wrong with acting in a more civil manner.

TV drama producer:   Well, if there is such an increase, it’s not because of TV dramas: we simply give people what they want to see. What can be wrong with letting the viewers decide? Furthermore, if restrictions were put on my show, that would amount to censorship, which is wrong.

The editorialist and the TV drama producer’s statements provide the most support for holding that they disagree about whether

(A) TV dramas influence people’s conception of what is the norm

(B) it is wrong not to let the viewers decide what they want to see on TV

(C) TV dramas, by depicting violent crimes, are causing an increase of violence in society

(D) TV dramas should be censored

(E) the level of violence in society has changed

12. A billboard company executive recently announced an increase in its billboard rental rates by 5 to 10 percent next year over rates this year and expects other billboard companies to follow suit. The executive argued that despite this increase, advertisers will continue to profit from billboard advertising, and so billboard rentals will be no harder to secure than this year.

Which one of the following, if true, would most support the billboard company executive’s argument?

(A) The number of billboards leased by providers of services is increasing, while the number of billboards leased by products is decreasing.

(B) Most costs of production and distribution of services typically advertised on billboards are expected to rise 2 to 5 percent in the next year.

(C) Next year billboard leases will no longer be available for periods shorter than nine months.

(D) A recent survey has shown that the average number of people commuting in their own cars along routes with billboards is increasing at the rate of 2 percent every three months.

(E) The method of estimating the traffic passing a particular billboard will change next year.

13. Announcement for a television program:   Are female politicians more effective than male politicians at applying policies that help the homeless and indigent citizens among their constituency? To get the answer we’ll ask politicians of both sexes this question. Tune in tomorrow.

Which one of the following, if true, identifies a flaw in the plan for the program?

(A) Those who are best able to provide answers to the question are the homeless and indigent constituents of the respective politicians.

(B) The homeless and indigent need more advocacy groups arguing for their rights.

(C) Politicians are in general unwilling to disparage other politicians with whom they have to develop policy with.

(D) Homeless and indigent citizens among their constituents generally do not vote.

(E) Female politicians make up only 15 percent of the US Congress, and this number is even smaller in state legislatures.

14. Business consultant:   The practice of streamlining this business process to reduce costs cannot be adequately supported by the claim that any simplification of a business process is more effective than doing nothing at all to reduce costs. What must also be taken into account is that such streamlining may hurt quality of service and reduce customer satisfaction.

Which one of the following most accurately expresses the main point of the business consultant’s argument?

(A) Streamlining a business process is more effective than other forms of solutions for reducing costs.

(B) Streamlining a business process’s effectiveness at reducing costs is not sufficient justification for using it.

(C) Streamlining a business process is more effective than not streamlining a business process at all for reducing costs.

(D) Streamlining a business process hurts quality of service and reduces customer satisfaction more than other solutions for reducing costs.

(E) Streamlining a business process should not be used to reduce costs unless it improves quality of service and increases customer satisfaction.

15. Shipments are recorded in inventory on the date it arrives only if they are received before 3 p.m. Valerie knows that a shipment was received before 3 p.m. So, Valerie knows that the shipment was recorded on the date it arrived.

Which one of the following exhibits both of the logical flaws exhibited by the argument above?

(A) It is obvious to Jamey that she will receive a raise at the end of this financial quarter, because Jamey will receive a raise only if everyone in her department receives a raise, and Jamey knows everyone in her department will receive a raise.

(B) We know that Sally plays tennis every Tuesday evening. Today is Tuesday, so it follows that Sally is playing tennis this evening.

(C) Monte aspires to become a psychiatrist. It is well known that psychiatrists earn a great deal of money. Monte evidently wants to make a great deal of money.

(D) Phil believes that a square is a triangle and that a square has three sides. Therefore, Phil believes that there is a square that has three sides.

(E) Employees are the only ones who are allowed to eat their lunch in the company break room. Since Jessica is an employee, she will be allowed to eat lunch in the company break room.

Questions 16–17

Park ranger:   The only reason for us to restrict climbing in the park is to protect those inexperienced climbers from hurting themselves. Climbing on Mount Brunda is undoubtedly dangerous, but we should not restrict climbing. If inexperienced climbers cannot climb in our park they will most certainly travel to the next county over to climb in their parks, which have much more dangerous terrain than ours.

16. The pattern of reasoning in which one of the following is most similar to that in the park ranger’s argument?

(A) The reason for the ordinance restricting loud music in residential areas after 10 p.m. was to protect those who were being disturbed by parties in their neighborhood as well as to maintain property values. Loud music had become a serious problem, but it has now been eliminated from our neighborhoods. So, we can probably remove the restriction now.

(B) The reason for requiring financial audits is to ensure that our business plans are operating within the budget constraints set by our accountants. Currently, financial concerns are less compelling than product development concerns, but in the long run, our financial viability must be ensured. Therefore, the requirement of financial audits should not be discontinued.

(C) Detention is used to separate a poorly behaving child from the well-behaving children. A poorly behaving child hinders the education of the other children, but detention also stigmatizes that child. Since the disruption to the other students is less a problem than the stigmatization of a poorly behaving child the practice of detention should be discontinued.

(D) Speed bumps on streets are meant to slow down traffic and reduce the danger to pedestrians. However, it would be detrimental to the Riverside Street area to have traffic move slower. So, speed bumps should not be implemented there.

(E) The function of an antivirus software is to prevent harmful programs from infecting a computer. However, Bulwark computers are built to be resistant to harmful programs and thus do not need an antivirus program. Antivirus programs, therefore, should not be installed on Bulwark computers.

17. Which one of the following principles, if established, would provide the strongest support for the park ranger’s argument?

(A) Since safety in a recreational activity depends on the level of skill of the participant in that activity, the climbing restrictions should be left to the discretion of the professional climbing instructors operating in the area.

(B) If recreational activities constitute a danger to the participants in those activities, then the park administrators should enact restrictions prohibiting those activities.

(C) Since the park could be held legally liable for accidents that occur within the park’s borders, restrictions apply to any unnecessarily dangerous activities within the park.

(D) Restrictions that seek to eliminate dangers should not be enacted if their enactment would lead to dangers that are greater than those they seek to eliminate.

(E) Restrictions of recreational activities of the park visitors should not be put into effect unless those activities pose a danger to participants.

18. During an eight-month period, the total output of offshore oil rigs within the nation of Salveria remained constant. During this period monthly output of oil rigs owned by Cenpan Oil Company doubled, and its share of the oil market within Salveria increased correspondingly. At the end of this period, new safety standards were imposed on oil rigs owned by Salveria companies. During the four months following this imposition, Cenpan Oil Company’s share of the Salveria market declined substantially even though its monthly output within Salveria remained constant at the level reached in the eighth month of the eight-month period.

If the statements above are true, which one of the following CANNOT be true?

(A) A decrease in the total monthly output of oil within Salveria will occur if the safety standards remain in effect.

(B) Over the four months before the imposition of the safety standards, the combined market share of companies other than Cenpan Oil Company with offshore oil rigs in Salveria decreased.

(C) Since the imposition of the safety standards, Cenpan Oil Company’s average profit on each barrel of oil sold within Salveria has increased.

(D) If the safety standards had not been imposed, Cenpan Oil Company’s share of the Salveria oil market would have decreased even more than, in fact, it did.

(E) The total monthly output within Salveria of oil from oil rigs owned by companies other than Cenpan Oil Company decreased over the four months following the imposition of safety standards.

Questions 19–20

Because excessive income taxes can make business activity difficult if not impossible, their use by governments is never justified. Purists, however, claim that all business taxes, such as sales tax, transportation fees, and export tariffs, should be prohibited because government should never be a burden to business activity. This is ridiculous; almost every interaction between government and business is some sort of burden, from work safety to food safety rules to labor regulations to payroll requirements. Yet, none of these is prohibited on the basis of being a burden. Furthermore, we should be attending to far more serious infrastructure problems that cause unnecessary restrictions to growth and business failures. Therefore, the use of some taxes, fees, and tariffs is acceptable and should not be prohibited.

19. Which of the following statements, if true, would be the strongest challenge to the conclusion?

(A) Unnecessary restrictions to growth and business failures happen to nonprofits and individual citizens as well.

(B) There would be more unnecessary restrictions to growth and business failures if it were not for sales tax, transportation fees, and export tariffs.

(C) Income taxes are just as burdensome to businesses as other taxes such as sales taxes and transportation fees.

(D) Sales tax, transportations fees, and export tariffs together can make business activity difficult if not impossible.

(E) Some taxes, fees, and tariffs help businesses by making them avoid socially unproductive activities.

20. Which one of the following can be inferred from the passage above?

(A) Some of the unnecessary restrictions to growth and business failures are caused by excessive income taxes.

(B) The fact that something is a burden to business is not sufficient reason for prohibiting it.

(C) The use of excessive income taxes by governments should be prohibited because excessive taxes are burdensome to business.

(D) Excessive income taxes by governments on business are a less serious problem than are unnecessary business failures.

(E) There is nothing burdensome about the use of nonexcessive taxes by governments.

21. Fred achieved the highest revenues in the fourth-quarter sales challenge by beating Gigi, the winner of each of the three previous quarters. We can conclude from this that Fred took sales training during the last year.

The conclusion follows logically if which one of the following is assumed?

(A) If Gigi took sales training, she would win the sales challenge.

(B) Gigi did not take as much sales training as Fred did.

(C) Gigi is usually a better salesperson than Fred.

(D) If Fred took sales training, he would win the sales challenge.

(E) Fred could beat a three-time winner only if he took sales training.

22. Most people make major purchases such as cars and appliances without doing any research of their own. Some of these people rely solely on advice from friends or a salesperson, whereas some others make decisions merely on an emotional response. Only a few always do their own research before making such a purchase. Nonetheless, a majority of buyers say that they are happy with their purchase.

If the statements in the passages are true, which one of the following must also be true?

(A) Some people who research large purchases on their own, while just as often relying on advice from their friends and a salesperson or on emotions, are happy with their purchases.

(B) Most people who make large purchases either rely solely on advice from their friends or a salesperson or make decisions based merely on emotion.

(C) Some people are happy with their purchase even though they do not do any research on their own.

(D) Most people who rely solely on advice from their friends or a salesperson rather than emotions are happy with their purchases.

(E) Most people who make large purchases without doing any research on their own are happy with their purchases.

23. Many people believe that the remedy for our education problems is to reduce the size of the classes, but recent statistics from New York City show that while class sizes in the city varied widely, city-wide standardized test scores were similar.

The statistics cited function in the argument to

(A) illustrate the need for reducing class size in schools across the nation

(B) demonstrate that there is no relation between class size and the quality of education

(C) suggest that the size of classes is not the only influence on the quality of education

(D) establish that the size of classes does not need to be reduced

(E) prove that standardized testing is a poor way to gauge the success of an educational system

24. While most people normally believe that well-watered plants grow best on a bright sunny day, studies show that, in fact, the reverse is true. Plants actually benefit from the haze caused by carbon dioxide in the atmosphere, enabling them to absorb 25 percent more carbon than normally possible. The burning of fossil fuels and other human industrial activities are the main cause of increased carbon dioxide. Therefore, such industrial activities are obviously beneficial to agriculture and those who depend on it.

The flawed reasoning in the argument above is most similar to that in which one of the following?

(A) Vigorous exercise has been proven to help prevent a number of diseases, ailments, and injuries, so obviously no harm, and a great deal of good, can come from vigorous exercise.

(B) Excessive use of certain antibiotics increases one’s susceptibility to certain antibiotic-resistant infections. Therefore, the best policy is to avoid using such antibiotics, thereby strengthening the body’s ability to resist disease.

(C) Because a low-carbohydrate diet has been shown to be more healthful than a diet high in carbohydrates, a diet in which foods with low carbohydrate content have been entirely replaced by foods with high amounts of carbohydrates is bound to be even more healthful.

(D) Fresh fruits and vegetables contain more vitamins and minerals than processed fruits and vegetables. Therefore, one ought to completely abandon processed for fresh fruits and vegetables.

(E) Excessive stress can lead to sleep disorders and physical and mental illnesses, so if one wishes to avoid health problems associated with stress, one ought to take a day off from work every week.

25. Columnist:   A democratic society cannot function unless strong bonds of mutual trust have formed among its citizens. Only by participation in civic organizations, political parties, and other groups outside the family can such bonds be formed and strengthened. Thus, it is obvious that the widespread reliance on e-mail and Internet social networking for interaction has an inherently corrosive effect on democracy.

Which one of the following is an assumption on which the columnist’s argument depends?

(A) Civic organizations cannot usefully advance their goals by using social networking.

(B) Anyone who relies on e-mail and social networking for interaction is unable to form a strong bond of mutual trust with another citizen.

(C) Relying on e-mail and social networking for interaction generally makes people less likely to participate in groups outside their families.

(D) People who rely on e-mail and social networking for interaction are generally closer to their families than are those who do not.

(E) Meetings and other forms of personal interaction strengthen, rather than weaken, democratic institutions.

STOP

IF YOU FINISH BEFORE TIME RUNS OUT, CHECK YOUR WORK ON THIS SECTION ONLY. DO NOT GO ON TO ANY OTHER TEST SECTION.

SECTION III

Time—35 minutes

25 Questions

Directions: Each passage in this section is followed by a group of questions to be answered on the basis of what is stated or implied in the passage. For some of the questions, more than one of the choices could conceivably answer the question. However, you are to choose the best answer; that is, the response that most accurately and completely answers the question, and blacken the corresponding space on your answer sheet.

PASSAGE A

Images

Images

PASSAGE B

Images

Images

1. Which one of the following best expresses the main idea of passage B?

(A)  As globalization continues justices actively engaged in interpreting the Constitution make an effort to avoid the ever-increasing distraction caused by precedents set by the highly active foreign courts.

(B)  The opponents of comparativism have given into isolationist concerns and their ill-conceived attacks on comparativist justices are hindering American progressivism.

(C)  The United States is a unique and exceptional nation and due to the wisdom shown by its forefathers in designing the Constitution justices do not need to refer to precedents beyond its borders.

(D)  As a relatively new method of interpreting the Constitution comparativism has not established a long enough history to prove whether it is a passing fad among legal scholars or a practice that is here to stay.

(E)  Some legal scholars and jurists have offered legitimate criticisms of comparativism due to the inapplicability of foreign precedent to US law as established by the Constitution.

2. It can be inferred that the author of passage A would most likely hold which one of the following attitudes towards the opponents of comparativism discussed in passage B?

(A)  Derisive because the basis of their arguments is close-minded and rooted in bigotry.

(B)  Appreciative of the role they are taking in the public discourse while ultimately feeling their arguments are wrongheaded.

(C)  Critical because their arguments ignore the reality of the current legal and political situation.

(D)  Scornful because their arguments are irrelevant to the current scholarly discourse.

(E)  Dismissive of their political response but respectful of the basis for their opposition.

3. Which one of the following statements about those who oppose comparativism can be inferred from passage B?

(A)  They support the execution of minors and the mentally handicapped, support anti-sodomy laws, and oppose affirmative action.

(B)  They consider the fact that the United States is unique in the world in some of its legal practices as a relevant consideration when interpreting the Constitution.

(C)  They are not as well versed in foreign court systems as those who support comparativism.

(D)  They believe that globalization is a short-term economic trend and that legal scholars should not allow constitutional interpretation to be influenced by such fleeting shifts in the economy.

(E)  They believe that the trust that Americans have given their justices is misplaced and should be reinstituted in a smaller body of justices that will not be so easily swayed by foreign adjudications.

4. According to information contained in passages A and B, Supreme Court Justices Kennedy and Breyer would be most likely to agree about which of the following?

(A)  English law is more applicable to US law than that of other nations.

(B)  Foreign law can validly be used as precedent in US courts.

(C)  Foreign law should be the central consideration with regard to constitutional issues.

(D)  Comparativism has had more influence in other nations than in the United States.

(E)  The use of comparativism is only applicable in the Supreme Court.

5. The opponents of comparativism, as discussed in passage B, would most likely analogize the use of comparativism in domestic cases as

(A)  a doctor who is unsure what the correct medicine is for a treatment and consults with other doctors for their opinion before making a recommendation

(B)  a teacher who is unsure what method to use for teaching a difficult subject in grammar, so she consults several teaching guides, some of which are for foreign students learning English

(C)  a board member of a long-standing organization who must determine a new policy and uses the policies of another, younger but similar organization to determine the language of that policy

(D)  a candidate for public office who argues during a debate that a national policy currently in effect is wrongheaded because France and Germany do not have such a policy in effect

(E)  an international corporation that recalls a product in Germany because it fails to meet German health standards and then recalls it in France and England as well even though the product meets their health standards

6. It can be inferred that the opposition in the case in which Justice Kennedy used comparativism

(A)  held that Justice Kennedy made the right decision but that using foreign law as a justification was wrong

(B)  thought that Justice Kennedy’s decision would have been correct if he had based it on English common law instead of current foreign precedents

(C)  saw comparativism as unacceptable in cases involving laws governing moral issues but acceptable in all others

(D)  used comparativism as a cover for a deeper disagreement to Kennedy’s decision

(E)  alleged that comparativism was equivalent to committing treason

7. The issues that are central to both passages A and B can best be described as

(A)  offering an a priori justification for a new approach to interpreting the Constitution

(B)  criticizing a trend in constitutional interpretation by Supreme Court justices

(C)  proposing an alternative method of interpreting the Constitution that will resolve high profile cases being argued before the Supreme Court

(D)  explaining the origins of some new techniques of constitutional interpretation being used by current Supreme Court justices

(E)  describing the nature of a legal controversy and the polarization of those on both sides of the argument

Images

Images

Images

Images

8. Which one of the following best states the main idea of the passage?

(A)  Biophysical economics has shown that it is prone to the same flaws as neoclassical economics, so it should be discarded for not contributing any worthwhile economic ideas.

(B)  Its focus on energy proves that biophysical economics is a limited and somewhat immature economic field of study with dubious influence on predictive models.

(C)  The failure of predictive models based on the prevailing economic ideas has allowed a new and innovative economic theory to come forward from unlikely sources, including non-economists.

(D)  Biophysical economics has supplanted neoclassical economics as the dominant field of study because it uses some of the same concepts as traditional economics to create its new predictive models.

(E)  Biophysical economics is an intriguing subset of environmental economics worth studying mainly for its suggestion to use the laws of thermodynamics in economic analysis.

9. It can be inferred that which one of the following is true of the theories put forward by the biophysics economists?

(A)  An economy can sustain positive growth as long as it has a stable or increasing EROI.

(B)  An economy can sustain positive growth as long as it has the supply of natural resources to support its energy needs.

(C)  An economy can grow despite any technological advances or physical changes when there are market forces that overcome such deficiencies.

(D)  Biophysical economists and ecological economists agree about the physical limitations to the world economy but disagree about the solutions.

(E)  Of the proponents of biophysical economics, the ecologists are the majority of those who believe that the collapse of the world economy is imminent.

10. According to the passage, which of the following, if true, would most directly affect the biophysical economic models?

(A)  Consumers switch their preference from gasoline-powered to electric-powered automobiles.

(B)  The government increases taxes on gasoline, coal, and natural gas.

(C)  In order to maintain current supply levels, the largest mining companies must use a more costly and dangerous method to source coal.

(D)  In order to maintain current supply levels, oil companies must take on more debt to explore new areas and drill new wells.

(E)  An energy company derives a new technology for drilling a new deep-sea oil well that avoids an increase in cost the company had expected.

11. According to the passage, which one of the following best describes the biophysical economist view of oil supplies for energy?

(A)  scarce supply and expensive to source

(B)  stable or slightly decreasing supply, yet increasingly cheaper to source

(C)  unknown supply, yet increasingly cheaper to source

(D)  decreasing supply, yet inexpensive to source

(E)  abundant or stable supply, yet increasingly expensive to source

12. Which one of the following best describes the function of the final paragraph?

(A)  gives supporting facts for why biophysical economics has better predictive models than neoclassical economics

(B)  shows that while biophysical economics may offer stronger theories, certain practicalities will hinder its progress in overturning the dominance of classical economics

(C)  exhibits the conflict between biophysical economists and ecological economists

(D)  explains that while biophysical economics is new it is not immune to the problems that have led to the distrust of neoclassical economics

(E)  demonstrates how a disagreement between the factions within the biophysical economics community will only serve to make their arguments against neoclassical economics stronger

13. The author’s primary purpose in the passage is to

(A)  outline the ideas and pitfalls of a new economic theory intended to solve the failings of an older, prevailing economic theory

(B)  debunk an alternative economic model to the classically accepted economic model

(C)  explain how a current economic crisis has resulted in some unlikely theories on the economy

(D)  prove that economists supporting new economic theories are prone to the same pitfalls as economists supporting older, more well established theories

(E)  describe an economic conflict between two rival theories as to why an economic crisis occurred

Images

Images

Images

Images

14. Which one of the following best states the main idea of the passage?

(A)  Recent theories and discoveries put forward by scholars have raised some doubt about the authenticity of Galileo Galilei’s work and argue that the conflict with the Catholic Church may have been unjust.

(B)  Recent theories and discoveries put forward by scholars have put doubt on the church’s motives for trying Galileo Galilei, and a conspiracy may surround his fall from grace. Nevertheless, the facts of his trial and house arrest are undeniable.

(C)  Scholars questioning the story of Galileo Galilei’s conflict with the church have been heavily influenced by current political events and should be discounted, especially since the facts of his trial and house arrest are indisputable.

(D)  The recent discovery of authentic and unambiguous documents have put the story of Galileo Galilei’s fall from grace into serious question, and scholars are currently pursuing explanations regarding the truth of his crime and his house arrest.

(E)  The two areas in question today with scholars are whether Galileo Galilei was ever put on trial by the Catholic Church and for what reason. His house arrest was never in doubt, but why he was not burned at the stake like his compatriots still confounds some scholars.

15. Which of the following political events of his time was most likely to have influenced Giorgio de Santillana while writing his book The Crime of Galileo?

(A)  the dropping of the atomic bomb on Japan

(B)  the end of Nazism and the rise of communism

(C)  the House Un-American Activities Committee hearings

(D)  the cold war and the resulting Korean War

(E)  the rise of television as a source of entertainment and news

16. The passages suggests that Pietro Redondi would most likely agree with which of the following statements about Galileo Galilei?

(A)  The forgery of the injunction issued for the arrest of Galileo Galilei was one more piece of evidence that there was a conspiracy within the Catholic Church to cover up a more serious crime committed by the scientist.

(B)  Galileo’s contemporaries were jealous of his achievements and betrayed him to the church, so the church invented the crime of Copernicanism to cover up its culpability in this betrayal.

(C)  The Medici family financed Galileo Galilei and urged him to challenge church doctrine, but they failed to protect him when he came to trial, which is why he was ultimately put under house arrest.

(D)  The theory that the earth and other planets revolved around the sun was less offensive to the Catholic Church than the natural philosophy theorizing that atoms were indestructible and immutable.

(E)  The Jesuit astronomer Orazio Grassi, S.J., disliked Galileo Galilei and his theories, regardless of their truth, and it was Grassi’s determination to destroy the scientist that resulted in his trial and house arrest.

17. The passage implies which of the following with regard to the relationship between Giordano Bruno and Galileo Galilei?

(A)  Both scientists supported Copernicanism but differed on the theory of atomism and this is why their punishments were different.

(B)  Both scientists supported atomism but differed on the theory of Copernicanism and this is why their punishments were different.

(C)  Both scientists had a relationship with the church that was poisoned by Orazio Grassi, but Bruno was burned at the stake because he had committed the worse of their crimes.

(D)  Both scientists had offended the church by continuing to teach their scientific beliefs despite the church’s objections, but one of them fared better because he enjoyed the protection of a wealthy benefactor.

(E)  Both scientists supported Copernicanism but differed on God’s relationship to the order of the universe.

18. The passage implies which of the following with regard to the church and publications in the time of Galileo Galilei?

(A)  Publications were registered with the church regardless of whether they offended church doctrine.

(B)  The church probably would not have put an author on trial for a theory published in a book that had not been put on the list of proscribed books.

(C)  The church put every author who had a book on the proscribed list on trial and all except Galileo Galilei were burned at the stake.

(D)  Books on the church’s proscribed list were not necessarily considered offensive to the church and not every book that was offensive to the church was on the proscribed list.

(E)  Even though the church considered Galileo Galilei’s The Assayer offensive, it did not add it to the list due to the patronage of the Medici family.

19. The author’s primary purpose in the passage is to

(A)  summarize recent discoveries that give further insight into the story of Galileo Galilei’s conflict with the Catholic Church

(B)  describe a dispute between two scholars who disagree regarding the basis for Galileo Galilei’s trial and punishment by the Catholic Church

(C)  present and assess two theories from scholars that have called into question aspects of the story of Galileo Galilei’s conflict with the Catholic Church

(D)  explain the complicated aspects of the Catholic Church’s considerations when it put Galileo Galilei on trial and decided his punishment

(E)  criticize the theories of two scholars that have questioned the validity of aspects of the story of Galileo Galilei’s conflict with the Catholic Church

Images

Images

Images

Images

20. The passage is primarily concerned with discussing which of the following?

(A)  two interpretations of roles played by gender in the classroom and how it effects the learning process

(B)  two unsubstantiated theories of an educational strategy, neither of which agrees with the other or has quelled recent criticism

(C)  two studies on single-sex education that are controversial and have been criticized by the academic community

(D)  two revolutionary discoveries of how children learn in the classroom that are just beginning to be incorporated into public schools

(E)  how a practice long successful in private and parochial schools is being applied to public education but not without controversy over application and efficacy

21. The reference to studies on single-sex education in lines 13–18 serves primarily to

(A)  give credibility to the arguments in favor of single-sex education

(B)  exhibit the academic environment and research that currently surrounds the development of contemporary single-sex education

(C)  offer a caveat that might give the reader pause when accepting the arguments for single-sex education

(D)  create doubt in readers’ minds so that they will favor Dr. Sax’s approach over the other approach

(E)  describe the historical background that led education to develop the modern concept of single-sex education

22. The passage suggests that Dr. Sax would probably agree with which one of the following statements about the children in the classroom?

(A)  Because girls are more susceptible to pressures to keep quiet in social situations, they should be separated from boys to allow them to express themselves more freely.

(B)  Because boys would rather look strong than smart in front of the opposite sex, they should be separated from them so that they can express themselves more honestly.

(C)  Because there is a stark difference between how boys and girls perform on standardized tests, different tests should be given to each based on their learning differences.

(D)  Because boys are inherently more adept with visual stimuli, boys should be separated from girls so that they may benefit from a more visual educational environment.

(E)  Because there is no difference between how boys and girls perform on manual dexterity tests, any nonacademic courses should be coed.

23. Which of the following statements best exemplifies the American Civil Liberties Union’s concern mentioned in lines 46–49?

(A)  Boys are separated from girls because they can handle a more disorderly environment than girls. Therefore, only boys are taught how to organize their belongings more effectively.

(B)  Girls are separated from boys because their listening is better. Therefore, girls never learn to listen to the opposite sex effectively.

(C)  Boys are separated from girls because they have better visual acuity. Therefore, boys take only jobs that involve their visual abilities.

(D)  Girls are separated from boys because boys pester them in class. Therefore, girls never learn to negotiate the cross-gender dynamic.

(E)  Boys are separated from girls because they can handle a more disorderly environment than girls. Therefore, the school reinforces the misconception that boys are better than girls for jobs with chaotic environments.

24. The passage suggests that critics of both the genetic and social approaches to single-sex education would agree with which of the following statements?

(A)  Single-sex education makes sense as long as each class teaches the fundamentals of American values.

(B)  Single-sex education is warranted only to protect children from instilling in them un-American values.

(C)  Single-sex education is an excellent method of ensuring that American values are taught in a way that is appropriate for the particular gender.

(D)  Single-sex education is an inherently flawed model of educating children because it authorizes a particularly un-American separation rather than integration of the sexes.

(E)  Teachers of single-sex education classes are unable to instill truly good values in their students since the opposite sex is not present to give their unique perspective.

25. Which of the following statements can be inferred about single-sex education based on the passage?

(A)  Until researchers are able to put forward convincing evidence, single-sex education will remain a minor force in American public education.

(B)  As long as single-sex education controverts American values of integration people will oppose its growth in the American public education system.

(C)  Regardless of the failure of research to prove the efficacy of single-sex education, advocates and parents will continue to demand single-sex classes for children.

(D)  Anecdotal evidence and the advocacy of people like Dr. Sax will propel single-sex education into the mainstream of American public education.

(E)  The social malaise of the American public school system will quell the genetic approach and advance the sociological approach into dominance.

STOP

IF YOU FINISH BEFORE TIME RUNS OUT, CHECK YOUR WORK ON THIS SECTION ONLY. DO NOT GO ON TO ANY OTHER TEST SECTION.

SECTION IV

Time—35 minutes

26 Questions

Directions: Each group of questions in this section is based on a set of conditions. In answering some of the questions, it may be useful to draw a rough diagram. Choose the response that most accurately and completely answers each question and blacken the corresponding space on your answer sheet.

Questions 1–5

A conference planner must schedule six keynote speakers—Chapman, Dabu, Forest, Gant, Hart, and Jonas—over six days of a conference—days 1 through 6. The speakers are assigned to the days, one speaker per day, according to the following conditions:

Chapman and Dabu must speak on days that are separated from each other by exactly one day.

Chapman and Gant cannot speak on consecutive days.

Gant must speak on a later day than Forest.

Jonas must speak on day 3.

1. Which one of the following lists an acceptable schedule of the speakers for days 1 through 6, respectively?

(A)  Hart, Forest, Jonas, Dabu, Gant, Chapman

(B)  Forest, Gant, Chapman, Jonas, Dabu, Hart

(C)  Gant, Hart, Jonas, Chapman, Forest, Dabu

(D)  Forest, Chapman, Jonas, Dabu, Gant, Hart

(E)  Chapman, Dabu, Jonas, Forest, Gant, Hart

2. Which one of the following is a complete and accurate list of the days to which Chapman could be assigned?

(A)  2, 4, 5

(B)  2, 4, 6

(C)  2, 3

(D)  2, 4

(E)  1, 2

3. Which one of the following CANNOT be true?

(A)  Hart is scheduled to speak on day 2.

(B)  Dabu is scheduled to speak on day 2.

(C)  Chapman is scheduled to speak on day 2.

(D)  Forest is scheduled to speak on day 1.

(E)  Forest is scheduled to speak on day 5.

4. Which one of the following must be true?

(A)  Either Forest or else Gant is scheduled to speak on day 2.

(B)  Either Forest or else Gant is scheduled to speak on day 5.

(C)  Either Chapman or else Dabu is scheduled to speak on day 2.

(D)  Either Chapman or else Dabu is scheduled to speak on day 4.

(E)  Either Forest or else Hart is scheduled to speak on day 6.

5. Which one of the following CANNOT be true?

(A)  Dabu and Gant are scheduled to speak on consecutive days.

(B)  Forest and Chapman are scheduled to speak on consecutive days.

(C)  Forest and Hart are scheduled to speak on consecutive days.

(D)  Dabu and Gant are scheduled to speak on days that are separated from each other by exactly one day.

(E)  Forest and Jonas are scheduled to speak on days that are separated from each other by exactly one day.

Questions 6–12

Exactly six students—Bjorn, Chaim, Dottie, Fran, Gertie, and Heste—need to be assigned to four tutors: Sosa, Upton, Willie, and Zane. Each student is assigned to exactly one tutor, with at least one student assigned to each tutor. Sosa tutors only in math. Upton tutors only in physics and chemistry. Willie tutors only in math and physics. Zane tutors only in chemistry and English. Each student needs tutoring in only one of the subjects offered by the tutors and needs tutoring in no other subject.

The following rules govern the assignment of the students to the tutors:

At least Bjorn and Chaim are assigned to Willie.

At least Fran is assigned to Zane.

If Dottie is assigned to Upton, then Gertie needs tutoring in math

6. Each of the following could be true of the assignment of students to tutors EXCEPT:

(A)  It assigns Gertie to Upton and Dottie to Zane.

(B)  It assigns Gertie to Sosa and Dottie to Zane.

(C)  It assigns Fran to Zane and Gertie to Zane.

(D)  It assigns Dottie to Willie and Heste to Zane.

(E)  It assigns Dottie to Sosa and Gertie to Upton.

7. Which one of the following must be true?

(A)  Sosa is assigned exactly one of the students.

(B)  Zane is assigned fewer than three of the students.

(C)  Upton is assigned fewer than two of the students.

(D)  Zane is assigned exactly one of the students.

(E)  Willie is assigned exactly two of the students.

8. Each of the following could be true EXCEPT:

(A)  Both Dottie and Gertie need tutoring in English.

(B)  Both Dottie and Heste need tutoring in chemistry.

(C)  Both Dottie and Heste need tutoring in math.

(D)  Both Dottie and Gertie need tutoring in math.

(E)  Both Dottie and Bjorn need tutoring in physics.

9. If Fran and Heste need tutoring in the same subject as each other, then the maximum number of students who need tutoring in physics is

(A)  two

(B)  three

(C)  four

(D)  five

(E)  six

10. If Dottie and Fran need tutoring in the same subject as each other, then which one of the following must be true?

(A)  Dottie and Fran need tutoring in English.

(B)  At least two students need tutoring in math.

(C)  At least two students need tutoring in English.

(D)  At least one of Gertie and Heste needs tutoring in math.

(E)  At least one of Gertie and Heste needs tutoring in chemistry.

11. If exactly two students are assigned to Upton, then which one of the following could be true?

(A)  Gertie needs tutoring in chemistry and Dottie needs tutoring in chemistry.

(B)  Gertie needs tutoring in math and Dottie needs tutoring in chemistry.

(C)  Heste needs tutoring in math and Dottie needs tutoring in physics.

(D)  Heste needs tutoring in English and Fran needs tutoring in English.

(E)  Heste needs tutoring in math and Gertie needs tutoring in math.

12. If Bjorn, Chaim, Gertie, and Heste all need tutoring in the same subject as each other, then which one of the following could be true?

(A)  Exactly two of the students need tutoring in physics.

(B)  Dottie needs tutoring in English.

(C)  Dottie needs tutoring in math.

(D)  Exactly two students need tutoring in English.

(E)  Exactly three of the students need tutoring in chemistry.

Questions 13–19

In a small town, there are six buildings that need to be certified by two inspectors as earthquake-safe, three government buildings—I, J, and K—and three commercial buildings—M, N, and O—each evaluated once by inspector Harris and once by inspector Limon, during six consecutive weeks—week 1 through week 6. Each inspector inspects one building per week. No building will be inspected by Harris and Limon during the same week. The following additional constraints apply:

Limon cannot inspect any government building until Harris has evaluated that building.

Harris cannot evaluate any commercial building until Limon has evaluated that building.

Limon cannot inspect any two government buildings consecutively.

Harris must inspect building M during week 4.

13. Which one of the following is an acceptable inspection schedule, with the buildings listed in order of inspection from week 1 through week 6?

(A)  Harris: I, N, J, M, K, O

Limon: N, I, M, J, O, K

(B)  Harris: K, N, I, M, J, O

Limon: M, K, O, I, N, J

(C)  Harris: I, J, M, O, K, N

Limon: M, I, O, J, N, K

(D)  Harris: J, O, I, M, K, N

Limon: O, I, M, J, N, K

(E) Harris: J, K, I, M, N, O

Limon: M, J, K, N, O, I

14. If Harris inspects building K during week 3 and Limon inspects building J during week 6, which one of the following must be true?

(A)  Limon inspects N during week 5.

(B)  Limon inspects M during week 1.

(C)  Harris inspects J during week 2.

(D)  Harris inspects I during week 1.

(E)  Harris inspects O during week 6.

15. If Harris inspects building O during week 2, then Limon must evaluate which one of the following buildings during week 5?

(A)  O

(B)  K

(C)  N

(D)  M

(E)  I

16. Which one of the following must be true?

(A)  Harris does not inspect any two government buildings consecutively.

(B)  Harris inspects N before inspecting J.

(C)  Harris inspects a commercial building during week 2.

(D)  Limon inspects a commercial building during week 3.

(E)  Limon inspects K during week 6.

17. If Limon inspects M during week 1 and I during week 2, which one of the following could be true?

(A)  Harris does not inspect any two government buildings in a row.

(B)  Harris inspects O during week 6.

(C)  Harris inspects J during week 5.

(D)  N is the first of the commercial buildings to be inspected by Harris.

(E)  M is the third of the commercial buildings to be inspected by Harris.

18. Which one of the following is a complete and accurate list of the weeks during which Harris must inspect a government building?

(A)  week 1, week 3, week 5

(B)  week 1, week 2, week 3

(C)  week 1, week 5

(D)  week 1

(E)  week 6

19. Which one of the following could be true?

(A)  Limon inspects K during week 2.

(B)  Limon inspects I during week 3.

(C)  Limon inspects M during week 5.

(D)  Harris inspects O during week 1.

(E)  Harris inspects I during week 6.

Questions 20–26

A lumberyard offers exactly 10 types of lumber—both domestic and exotic varieties of cherry, maple, oak, pine, and walnut. The lumberyard is having a sale on some of these types of lumber. The following conditions must apply:

Exotic oak is on sale; domestic maple is not.

If both types of oak are on sale, then all walnut is.

If both types of cherry are on sale, then no pine is.

If neither type of cherry is on sale, then domestic oak is.

If either type of pine is on sale, then no walnut is.

20. Which one of the following could be a complete and accurate list of the types of lumber that are on sale?

(A)  exotic maple, exotic oak, domestic pine, exotic pine

(B)  exotic cherry, exotic oak, domestic walnut, exotic walnut

(C)  exotic maple, domestic oak, exotic oak, domestic walnut

(D)  domestic cherry, exotic oak, exotic pine, domestic walnut

(E)  domestic cherry, exotic cherry, exotic maple, exotic oak, domestic pine

21. If domestic walnut is not on sale, then which one of the following must be true?

(A)  At least one type of oak is not on sale.

(B)  At least one type of cherry is not on sale.

(C)  Exotic maple is not on sale.

(D)  Domestic pine is not on sale.

(E)  Domestic pine is on sale.

22. If both types of cherry are on sale, then which one of the following is the minimum number of types of domestic lumber that could be included in the sale?

(A)  one

(B)  two

(C)  three

(D)  four

(E)  five

23. Which one of the following CANNOT be true?

(A)  Neither type of cherry and neither type of maple is on sale.

(B)  Neither type of maple and neither type of pine is on sale.

(C)  Neither type of cherry and neither type of pine is on sale.

(D)  Neither type of maple and neither type of walnut is on sale.

(E)  Neither type of cherry and neither type of walnut is on sale.

24. If neither type of cherry is on sale, then each of the following must be true EXCEPT:

(A)  Exotic pine is not on sale.

(B)  Domestic walnut is on sale.

(C)  Exotic walnut is on sale.

(D)  Domestic pine is not on sale.

(E)  Exotic maple is on sale.

25. If domestic walnut is the only type of domestic lumber on sale, then which one of the following CANNOT be true?

(A)  Exotic walnut is not on sale.

(B)  Exotic cherry is not on sale.

(C)  Exotic pine is not on sale.

(D)  Exotic maple is not on sale.

(E)  Exotic walnut is on sale.

26. If exactly four of the five types of exotic lumber are the only lumber on sale, then which one of the following could be true?

(A)  Neither type of cherry is on sale.

(B)  Neither type of pine and neither type of walnut is on sale.

(C)  Exotic cherry is not on sale.

(D)  Exotic pine is not on sale.

(E)  Exotic maple is not on sale.

STOP

IF YOU FINISH BEFORE TIME RUNS OUT, CHECK YOUR WORK ON THIS SECTION ONLY. DO NOT WORK ON ANY OTHER TEST SECTION.

LSAT Practice Test 1 Answer Key

Section I

1. C

2. B

3. A

4. A

5. D

6. B

7. A

8. C

9. A

10. B

11. D

12. D

13. C

14. A

15. D

16. E

17. B

18. E

19. D

20. C

21. A

22. D

23. B

24. C

25. C

Section II

1. B

2. E

3. D

4. A

5. B

6. D

7. E

8. D

9. D

10. C

11. C

12. D

13. A

14. B

15. A

16. C

17. D

18. E

19. D

20. B

21. E

22. C

23. C

24. A

25. C

Section III

1. E

2. C

3. B

4. B

5. C

6. D

7. E

8. C

9. A

10. C

11. E

12. D

13. A

14. B

15. C

16. D

17. D

18. B

19. C

20. B

21. C

22. D

23. E

24. D

25. C

Section IV

1. D

2. B

3. A

4. D

5. C

6. D

7. B

8. A

9. B

10. D

11. B

12. C

13. A

14. D

15. C

16. D

17. B

18. D

19. A

20. B

21. A

22. A

23. E

24. E

25. B

26. D

Calculate Your Score

Complete the following table.

Your Raw Score

Images

Your Approximate Scaled Score

It is impossible to say with complete precision what raw score will translate to what scaled score on future LSATs, but here is a rough estimation.

Images

LSAT Practice Test 1 Answers and Explanations

SECTION I

1. Answer: C

STEP 1:  Read the question and identify your task.

This is a Strengthen question. The question asks that you find among the answers the strongest defense against Lorne’s criticism of Jamey’s argument.

STEP 2:  Read the argument with your task in mind.

You read through Jamey’s and Lorne’s statements, paying particular attention to the basis for Lorne’s criticism. In this case, Lorne believes that the results of the test that bacteria can survive on arsenic is not enough support to justify further research into life on other planets.

STEP 3:  Know what you’re looking for.

You look through the answers to assess their potential to undermine this belief.

STEP 4:  Read every word of every answer choice.

Answer A actually supports Lorne’s criticism by saying that bacteria survive better on phosphorus. Answer B brings up the toxicity of carbon while the bacteria were surviving on arsenic, which might also be seen as support for Lorne’s position since it gives further evidence that life based on an arsenic diet is unsustainable. Answer C provides evidence in favor of Jamey’s claim. It also shows that there is less chance of finding life on phosphorus-rich planets than on arsenic-rich planets, which implies the denial of Lorne’s implicit claim, that because of the higher likelihood of finding life on phosphorus-rich planets we should search there for life. Answer D gives some additional support to Jamey’s argument that life can survive on arsenic, but it does not necessarily undermine Lorne’s criticism that life on phosphorus is more sustainable. Finally, answer E says that they have not found proof of life on planets rich in arsenic and devoid of phosphorus. Although this doesn’t mean they do not exist, it does not strengthen Jamey’s argument against Lorne. Therefore, the correct choice is answer C.

2. Answer: B

STEP 1:  Read the question and identify your task.

This is a Describe question. It asks that you describe Lorne’s approach to criticizing Jamey’s argument.

STEP 2:  Read the argument with your task in mind.

You reread Lorne’s argument and see that he brings forth a new fact, that the bacterium survived longer on phosphorus than on arsenic, which may undermine Jamey’s conclusion.

STEP 3:  Know what you’re looking for.

The correct answer will describe Lorne’s attempt to undermine Jamey’s conclusion by bringing up that new fact.

STEP 4:  Read every word of every answer choice.

Answer A is fairly accurate. Lorne does agree with Jamey’s facts and says they do not support the conclusion. This may be your answer, but you should evaluate the rest of the options to see whether there is a better one. Answer B says that Jamey presents new facts that undermine her conclusion, which matches your own evaluation. This is much better than answer A and is very likely your answer. You should continue to evaluate the rest of the options. Answer C cannot be correct because Lorne definitely does not support Jamey’s conclusion. Regarding answer D, Lorne’s facts are not irrelevant and he attacks Jamey’s conclusion, not her credibility. Answer E says that Lorne attacks Jamey’s logic, but Lorne’s criticism is centered on the results of the study. He makes no criticism of Jamey’s logic. Thus, you cannot choose answer E. Answer B is the correct choice.

3. Answer: A

STEP 1:  Read the question and identify your task.

This is a Weaken question. It asks you to identify among the answers the statement or fact that most undermines the official’s conclusion.

STEP 2:  Read the argument with your task in mind.

You read the argument with this question in mind, but you are mainly concerned with the conclusion. The public official concludes that because of the upward trend in dog bites and the costly nature of the attacks, dog owners should be held more accountable for their pets’ attacks.

STEP 3:  Know what you’re looking for.

You can configure your own answer and match it against the given options. In this case, the answer should undermine the conclusion that the dog owners should be held accountable for the costly attacks by providing alternative reasons for the costs.

STEP 4:  Read every word of every answer choice.

Answer A points out that the high cost of the attacks is the fault of the hospital. This might very well be your answer, but you should evaluate the rest of your options. Answer B tries to explain the slow growth in pet ownership, which is one of the variables that leads to the conclusion, but it explains rather than contradicts or undermines that variable and thus cannot be your answer. Answer C says that most dog bites are between the owner and his or her dog, which does not address the issue of how the alleged negligence of the owners results in costly dog bites. Answer D goes beyond the argument at hand and discusses the ramifications of making increased accountability a reality, but this does not undermine the conclusion that owners should be held more accountable. It only undermines another argument about how to make them more accountable. Finally, answer E does bring some doubt to the statistics used in the argument, but there is still a possibility that the numbers justify the conclusion. You do not know. In the end, the best choice is answer A.

4. Answer: A

STEP 1:  Read the question and identify your task.

This is a Weaken question. The question asks that you find among the options the statement that least strengthens the argument. This means that four out of the five statements will strengthen the argument while one will actually weaken the argument.

STEP 2:  Read the argument with your task in mind.

In essence, the argument states that environmental damage will hinder the survival of the cichlid fish population.

STEP 3:  Know what you’re looking for.

You will look for a statement that says this is not true, that in fact the cichlid fish will survive despite environmental damage to the waters in which they swim.

STEP 4:  Read every word of every answer choice.

Answer A is very promising. It says that the eyes of the cichlids can adapt to a change in water conditions, which would suggest that the cichlid population would manage to survive despite the environmental damage described in the argument. This may be your answer, but you should evaluate the remaining options to be sure. Answer B gives a fact that further supports the argument’s notion that the cichlid’s enhanced vision requires clearer water for the species to survive. Answer C suggests that a particular population of cichlids may have been adversely affected by its proximity to an industrial plant, supporting the idea that pollution affects their survival and supporting the argument. Answer D says cichlids, in order to survive, have fled to clearer waters, also supporting the argument. Finally, answer E offers another phenomenon that hindered the cichlid vision and adversely affected the survival of the cichlid population, again linking the success of vision to survival and supporting the argument. The correct choice is answer A.

5. Answer: D

STEP 1:  Read the question and identify your task.

This is a Paradox question. The question asks that you choose the answer that helps resolve a discrepancy, or in other words, a seeming contradiction in the statements.

STEP 2:  Read the argument with your task in mind.

The statements discuss two economic conditions. The first says that coffee revenues are down and popular coffee houses may have to close. The second says that more people are visiting coffee houses and the number of successful coffee houses is higher. The first focuses on coffee and popular coffee houses, the second on coffee houses in general.

STEP 3:  Know what you’re looking for.

The correct answer will find a way to resolve this difference between these groups.

STEP 4:  Read every word of every answer choice.

Answer A focuses on coffee houses but does nothing to address the fact that coffee sales are down; thus, this answer does not resolve the discrepancy. Answer B may explain the downturn in coffee sales, but it does not address the growth of coffee houses in the second economic scenario. Answer C enters into another area altogether, comparing a bakery to coffee houses, and this is far afield of what you are looking for. Answer D says that the most popular coffee houses rely on coffee while other coffee houses rely on food for revenue. This would seem to explain the discrepancy, for if coffee revenues are down this would only affect those coffee houses that rely on coffee for most of their revenues, while the other coffee houses might grow and grow in number because they do not rely on coffee for most of their revenues. This seems to be your most likely candidate. Finally, answer E focuses on coffee and its price but not on the growth in coffee houses, so it cannot be the answer. The correct choice is answer D.

6. Answer: B

STEP 1:  Read the question and identify your task.

This is a Deduction question. This question asks you to derive a conclusion from the seemingly contradictory statements made by the manager and the technician.

STEP 2:  Read the argument with your task in mind.

You analyze their statements and discover that the manager says that the total number of dropped cell phone calls on the network is down. The technician says that for the cell towers in operation last year and the year before the trend is toward a higher number of dropped cell phone calls.

STEP 3:  Know what you’re looking for.

You can assume the answer will have to do with the fact that the manager is looking at the total number of dropped calls and the technician is looking at a specific group of cell towers (those that were in operation last year and the year before).

STEP 4:  Read every word of every answer choice.

Answer A discusses a drop in market share for the cell phone service. While this might be the result of a large drop in cell phone calls, there is no definitive connection between the two, so this cannot be your selection. Answer B says that at least one new cell tower was erected in the last two years. This would mean that there is at least one tower included in the manager’s statistics that is excluded in the technician’s and may explain why the manager’s number shows an increase and the technician’s a decrease. This could very likely be your answer, but you must review the rest of your options to make sure. Answer C focuses on cell phone calls, but the manager and technician are discussing the number of dropped cell phone calls. Also, this answer does not say what kind of change, so it is too inexact to be your choice. Regarding answer D, a lack of quality improvement would seem to indicate that the number of dropped calls would remain basically the same, so it does not reconcile the difference between the manager’s and technician’s statements regarding an increase or decrease in dropped calls. Finally, answer E discusses the quality of the calls that are not dropped, which is another topic altogether. The correct choice is answer B.

7. Answer: A

STEP 1:  Read the question and identify your task.

This is a Weaken question. This question asks you to choose the answer that most undermines the technician’s argument against the manager.

STEP 2:  Read the argument with your task in mind.

You can reread the technician’s argument and see that there has not been an increase in efficiency based on dropped cell phone calls for cell towers that existed last year and the year before. Also, the technician says there is a trend toward more dropped calls, so the second year’s number must be higher than the number for the previous year.

STEP 3:  Know what you’re looking for.

The correct answer will probably undermine one aspect of these bases.

STEP 4:  Read every word of every answer choice.

Answer A says that the cell towers that existed last year and the year before experienced an abnormally high number of calls last year. This might explain why there was a trend toward more dropped calls for those towers and seems to undermine the technician’s claim that the company has not achieved better efficiency. This may be your answer, but you must review the remaining options to be sure. Answer B in essence says the opposite of answer A, indicating that cell phone usage has not increased, which might be seen as support for the technician’s argument. Answer C says before last year there was a downward trend in usage, but it says nothing about what happened last year. While it implies that the trend might have changed, you cannot be sure and, thus, you cannot know this answer’s implications for the technician’s argument. Answer D somewhat undermines the manager’s argument, but it only addresses the decrease in dropped calls, not the increase, and is not as strong as answer A. Answer E compares the company’s results with those of another company, which is irrelevant. You only care about the internal results at issue with the manager and technician. The correct choice is answer A.

8. Answer: C

STEP 1:  Read the question and identify your task.

This is a Flaw question. The question asks that you identify why the argument is questionable.

STEP 2:  Read the argument with your task in mind.

The professor is arguing that the need for a humanities department should be questioned because literature and other areas of the humanities cannot be clearly defined.

STEP 3:  Know what you’re looking for.

The correct answer will most likely point out the unjustified leap the professor makes from the difficulty of defining literature to questioning the need for a humanities department.

STEP 4:  Read every word of every answer choice.

Considering answer A, you do not know what kind of professor is making the statements, so you do not know whether he or she has a vested interest in eliminating a humanities department. Answer B cannot be correct because the professor does give some support for literature’s lack of definability. Answer C says that the professor has failed to link the indefinability to the maintenance of a humanities department, which is exactly what you formulated as the expected answer. This is most likely your answer, but you must review the remaining options to see if one of them is better. Answer D may be true, but this is outside the argument the professor is making and does not deal with the logic of the statements. Answer E makes an observation that is not in evidence. The professor does not mention any sample at all. His statements are entirely general, so this answer is off the mark. The correct choice is answer C.

9. Answer: A

STEP 1:  Read the question and identify your task.

This is a Parallel question. The question is asking you to find the set of statements that follow the same pattern of logic as the statements regarding Southern diet and the rate of strokes in that area.

STEP 2:  Read the argument with your task in mind.

You read through the statements and follow the logic. The statements in the answer may not be ordered in the same manner, but the argument will follow the same logic. Basically, the statements describe a health phenomenon related to a particular group, connect that phenomenon to another related phenomenon, and make an extreme recommendation based on that connection.

STEP 3:  Know what you’re looking for.

The correct answer will describe a phenomenon for a particular group, connect that phenomenon to another related phenomenon, and make an extreme recommendation based on that connection.

STEP 4:  Read every word of every answer choice.

Answer A describes how people who drink more than two ounces of alcohol a day tend to suffer more recurring illnesses (first phenomenon), then says that excessive alcohol consumption increases the chances of developing liver disease (second phenomenon), and finally recommends that people who want to avoid liver disease give up drinking alcohol (extreme recommendation). While the structure is somewhat different, it is essentially the same logic. This is most likely your answer, but you should evaluate the remaining options to make sure. Answer B follows an “if … then” pattern, and deals with only one phenomenon, that antioxidants help prevent cancer. The study proves this phenomenon, the phenomenon is clarified, and a rather reasonable recommendation is made. This cannot be your answer. Answer C is missing a second phenomenon or observation. It simply says that people who live near powerful electromagnetic fields experience more cancer and recommends that the government do something about it. Answer D includes a second phenomenon or observation, but then provides a conclusion that completely ignores it. Answer E is similar to answer B and follows a typical “if … then” pattern, and it also deals with only one phenomenon, that vigorous exercise leads to fewer illnesses after age 50. Thus, the correct choice is answer A.

10. Answer: B

STEP 1:  Read the question and identify your task.

This is a Flaw question. The question asks that you identify the flaw in Mr. Thomas’s logic.

STEP 2:  Read the argument with your task in mind.

You read the argument and see that Mr. Thomas believes that the fact that Ms. Garcia said that she was giving up her sales responsibilities is in contradiction to the pattern of visiting customers.

STEP 3:  Know what you’re looking for.

The correct answer will most likely highlight a misunderstanding of Ms. Garcia’s words versus her perceived actions.

STEP 4:  Read every word of every answer choice.

Answer A says that Mr. Thomas failed to consider that the colleague misled him. This may be true, but you are only concerned with the reasoning Mr. Thomas uses with the given facts, not whether the foundation of that reasoning is flawed. Answer B says the flaw is making a conclusion based on equivocal language. This is very possibly the flaw because just working harder and visiting customers do not mean that Ms. Garcia is necessarily continuing her sales responsibilities. Such activities are ambiguous and could be interpreted in many ways. This may be your answer, but you must review the remaining options to be sure. Similar to A, answer C may be true, but again you are concerned only with Mr. Thomas’s logic using the facts he has and not the origin of them. Answer D also does not discuss the flaw in logic, but rather says Mr. Thomas failed to make a judgment regarding Ms. Garcia’s lack of character. Regarding answer E, Mr. Thomas does not really criticize Ms. Garcia. Instead, he says that either she or the colleague is lying, leaving open the option that Ms. Garcia is telling the truth. Therefore, the correct choice is answer B.

11. Answer: D

STEP 1:  Read the question and identify your task.

This is a Deduction question. The question asks that you qualify Talia and David’s disagreement with regard to growth.

STEP 2:  Read the argument with your task in mind.

In this case, David believes the company needs to change in order to return to earlier levels of growth. Talia believes that the company has reached an acceptable level considering the age of the company.

STEP 3:  Know what you’re looking for.

The correct answer will qualify the disagreement with regard to current versus past growth.

STEP 4:  Read every word of every answer choice.

Answer A says the disagreement is over whether growth can be changed, but this cannot be correct. Talia never says that the company cannot change, only that change is unnecessary. Answer B cannot be correct because both Talia and David acknowledge that growth has slowed. Answer C adds an irrelevant element not even mentioned in the discussion. Neither Talia nor David discusses a difference in perception among managers and employees, so this answer cannot be correct. Answer D says the disagreement is over whether growth is currently adequate, and this does seem to accurately qualify the disagreement because David believes that current growth is inadequate and the company needs to do something about it. Talia believes current growth is adequate and that the company should stay the course. This would seem to be your answer, but you must review the last option to be sure. Answer E says the disagreement is over whether growth is changing over time, but both agree that growth has changed from the early days. It is only the current status of growth that is under discussion. Thus, answer D is the correct choice.

12. Answer: D

STEP 1:  Read the question and identify your task.

This is a Weaken question. It asks you to find among the answers the one that most undermines the defense given by the government regulator for new regulations.

STEP 2:  Read the argument with your task in mind.

The regulator argues that the new rules are more effective because they have enabled the discovery of 60 percent more defects in toys than before.

STEP 3:  Know what you’re looking for.

You can formulate your own answer, expecting it to be a statement that negates that the improved discovery of defects actually benefits children.

STEP 4:  Read every word of every answer choice.

Answer A says the defects were found mostly in toys for a particular age group. This does not discredit the regulator’s argument. It is too narrow in scope. Answer B may be true but is irrelevant since it discusses another topic altogether, the effect the regulations might have on profitability. This has nothing to do with the regulator’s defense, which uses the improvement in discovery as justification. Answer C also brings up an irrelevant fact, the difference between domestic- and foreign-made toys. The regulator does not distinguish between domestic and foreign toys, so this is not pertinent to the question. Answer D brings up a fact that is very pertinent, for if the number of incidents of harm done by toys does not go down despite increased discovery of defects, then the new regulations are still inadequate and the regulator’s defense is seriously undermined. This is very likely your answer, but you must review the last option to be sure. Answer E is another statement that may be true, but the regulator’s defense has nothing to do with comparing his regulatory system with those of other industrialized nations. This is an irrelevant fact and you must conclude that the correct choice is answer D.

13. Answer: C

STEP 1:  Read the question and identify your task.

This is a Deduction question. The question asks that you take all the statements into account and determine which among the answers must also be true.

STEP 2:  Read the argument with your task in mind.

You must pay close attention to the logical terms—“vast majority,” “some,” and “all”—found in the statements.

STEP 3:  Know what you’re looking for.

The correct answer will be based on these qualifications for the particular groups: low corporate tax rate, high budget deficit, and large entitlement programs.

STEP 4:  Read every word of every answer choice.

Answer A cannot be the correct choice because the last statement says that all countries with a high budget deficit have large entitlement programs and the equation works in reverse as well. Answer B tries to draw a conclusion based on the last two statements, but those statements say that only some of the countries with a low corporate tax rate have a high budget deficit. The term some does not necessarily mean a majority. It could mean as few as one or two countries. Answer C tries also to draw a conclusion from the last two statements, and it succeeds where answer B failed. You know that all countries with high budget deficits have large entitlement programs. You also know that some of the countries that have high budget deficits have a high corporate tax rate. Therefore, it must be true that some of the countries with large entitlement programs also have a high corporate tax rate. This is most likely your answer. You can be sure that Answer D is not true. You know that all countries with a high budget deficit have large entitlement programs. The second statement says that some of the countries with a high budget deficit have a high corporate tax rate. Therefore, it cannot be the case that all countries with large entitlement programs have a low corporate tax rate. Finally, answer E may be true, but for the purposes of this exercise it leaves out an important link between the corporate tax rate and entitlement programs, that the countries have a high budget deficit. This answer wants to make a statement regarding all nations that have high and low corporate tax rates whether they have a high budget deficit or not, and that is not relevant to your discussion. Therefore, the correct choice is answer C.

14. Answer: A

STEP 1:  Read the question and identify your task.

This is a Flaw question. The question asks that you find among the possible answers a statement that identifies the failure in the argument. In this case, the question asks that you finish the sentence with the element that is ignored by the argument.

STEP 2:  Read the argument with your task in mind.

The argument makes a judgment that soil erosion will no longer be getting more serious based on two years of data, during which the amount of soil lost remained the same.

STEP 3:  Know what you’re looking for.

You expect the correct answer to point out the fact that the data only cover two years or that the level of erosion, regardless of the growth factor, is still a threat.

STEP 4:  Read every word of every answer choice.

Answer A seems a likely candidate since it says that the effects of soil erosion are cumulative, meaning that damage done previous to these two years as well as the loss over the two years has mounted and there needs to be significant movement in reverse to indicate that the problem is abating. Answer B may be true but these other problems (insects, climate change) are not relevant to the reasoning at hand. Regarding answer C, the statements do take into account the future and explicitly say “if this trend continues”; the argument does not ignore this possibility. Answer D links erosion to farm productivity, another factor that is irrelevant to the reasoning in the statements. The only reasoning you are concerned with is that which leads to the conclusion that soil erosion will no longer become more serious. The effects of soil erosion are not at issue. Finally, answer E also adds an irrelevant factor, geographic location of the erosion, in an attempt to confuse you. You are concerned with overall erosion only, not how it is distributed among areas. Therefore, the correct choice is answer A.

15. Answer: D

STEP 1:  Read the question and identify your task.

This is a Strengthen question. The question asks you to selected the axiom among the options that most justifies Elizabeth’s argument.

STEP 2:  Read the argument with your task in mind.

Elizabeth believes Larry’s complaint of unfairness is wrong because as long as each company in Larry’s industry has an equal chance of being audited by the government, then his company was treated fairly.

STEP 3:  Know what you’re looking for.

The correct answer will equate the equal probability of being audited to fairness.

STEP 4:  Read every word of every answer choice.

Answer A says it would be fairer not to enforce the rule at all than enforce it sometimes. Elizabeth might agree with this principle, but it is not supporting what she says regarding the fairness with which Larry’s company has been treated. Answer B discusses the fairness of fines, or punishment, and even though Larry mentions that his company was fined as a result of the audit, Elizabeth addresses only the fairness of audits. She makes no statement regarding the fairness of the fine, so this principle is beyond the scope of Elizabeth’s argument. Answer C is also about the fairness of fines and for the same reason as answer B is disqualified. Answer D expresses almost exactly what you formulated before reviewing the options. It equates equal likeness of being audited with fairness, regardless of who is eventually fined. This is very likely your answer, but you must review the last option to make sure. Answer E discusses the aspects of assessing fines and does not relate to the nature of auditing. Thus, the best choice is answer D.

16. Answer: E

STEP 1:  Read the question and identify your task.

This is a Strengthen question. The question asks that you choose which among the possible answers supports the argument made by the city official that Jason would not be able to keep his trailer due to an Abilene city ordinance denying parking rights for industrial vehicles.

STEP 2:  Read the argument with your task in mind.

Jason bought his trailer in Abilene, only to be told that he cannot keep it because trailers and other industrial vehicles cannot be parked within the city

STEP 3:  Know what you’re looking for.

The correct answer will match Jason’s situation with the stipulations of the city ordinance.

STEP 4:  Read every word of every answer choice.

Answer A says that trailers are not classified as residential vehicles, but this does not mean that they are necessarily classified as industrial vehicles either, which is the stipulation under the code. Answer B is about whether car dealers can sell trailers in Abilene, which is obviously true since Jason bought his trailer in that city. The code stipulates where people can park a trailer, not where they can sell it. Answer C says that vans and flatbed trucks are not classified as industrial vehicles, but the argument is concerning Jason’s trailer, not his van or flatbed truck, so this cannot be the right choice. Answer D is in essence a restatement of the code as it relates to nonindustrial vehicles, but Jason’s trailer is considered an industrial vehicle, so this cannot be what the official’s argument depends on. Finally, answer E says Jason lives in Abilene. By process of elimination this is your answer, but it also makes sense. For the official to argue that the code applies to Jason and his trailer, Jason must live in Abilene. Thus, the correct choice is answer E.

17. Answer: B

STEP 1:  Read the question and identify your task.

This is a Paradox question. It wants you to reconcile the energy expert’s conclusion with the contradictory evidence the expert presents.

STEP 2:  Read the argument with your task in mind.

Even though funding for alternative energy technologies has increased significantly, there has been little impact on the use of these technologies. Yet the expert concludes that the government should increase funding for alternative technologies.

STEP 3:  Know what you’re looking for.

The correct answer will give a rationale for why an increase in funding is justified today in spite of the slow adoption rate of alternative energy.

STEP 4:  Read every word of every answer choice.

Answer A discusses the number of professionals involved in developing alternative energy technologies, but it mentions nothing about the funding, which is the central focus of the energy expert’s conclusion, so this cannot be your answer. Answer B, regarding the popularity of the idea that government should fund the development of alternative energy technologies, doesn’t explain why an increase in funding is justified today. Answer C suggests technological change has made it possible for funding to be converted into success at a better rate. This would be a clear justification for increasing funding, despite the slow adoption rate. Answer D gives some explanation as to why the government may have to increase funding, mainly to fund higher salaries, but it does not reconcile why that increase should happen considering the meager adoption of practical applications of the new technologies. Answer E does not reconcile the expert’s conclusion; rather, it undermines it by saying that the money that has been spent so far has been wasted. Thus, the correct choice is answer B.

18. Answer: E

STEP 1:  Read the question and identify your task.

This is a Strengthen question. The question asks that you identify the axiom or principle that gives support for the historian’s judgment of the ordinary people of empire T.

STEP 2:  Read the argument with your task in mind.

The historian tells of an empire that conquered other nations in the name of its vision of a more civilized world. In the process, the people of the empire slaughtered people, and the historian says that because their vision was unachievable, the people of empire T were murderers.

STEP 3:  Know what you’re looking for.

The correct answer will explain how conquering other nations can be equated to murder.

STEP 4:  Read every word of every answer choice.

Answer A actually contradicts the historian’s statements by saying that empire T’s actions were warmongering; at the very beginning the historian argues that the leaders of the empire were not warmongering and were pursuing a vision of a more civilized world. Regarding answers B and C, the historian speaks of warmongering as the alternative view to their pursuit of a civilized world. One does not justify the other or vice versa. Also, the historian says they are not warmongering and both these principles state that they are warmongering. Answer D uses a term not even mentioned in the argument—the historian never discusses inhumanity—and it says empire T was warmongering, something the historian is actually arguing against. Finally, answer E is the only answer that actually discusses the historian’s accusation of murder. It also says that conquest in pursuit of what is later found to be an unachievable vision is murder, and this supports the historian’s accusation against the people of empire T perfectly (to a “T”). The correct choice is answer E.

19. Answer: D

STEP 1:  Read the question and identify your task.

This is a Describe question. The question asks you to identify the most accurate description of how the argument proceeds.

STEP 2:  Read the argument with your task in mind.

The argument states that a phenomenon is expected to occur, namely that people not employed by the startups will be encouraged by the boom to spend more money, and then argues that because people’s savings accounts are not decreasing, in actuality this cannot be the case.

STEP 3:  Know what you’re looking for.

The correct answer will most likely describe the argument as assuming that because one phenomenon is not occurring (savings not decreasing), the cause is not occurring (people not spending more).

STEP 4:  Read every word of every answer choice.

Answer A cannot be your answer because the argument does not give two alternative developments. One development, the boom, causes the other, the spending, but they are not alternatives to each other. Answer B goes off on a tangent. The argument says nothing about economic self-interest, misinformation or error, or economic harm. Answer C cannot be correct because the argument discusses only one consequence of their behavior, a decrease in savings, not two, and there is no indication that the consequence is predictable. Answer D is right on the mark. It says that the argument concludes that the development (increased spending) did not occur because a supposed consequence (lower savings) did not occur. This would seem to be your answer, but you should evaluate your last option. Answer E indicates the evidence of the change is ambiguous; however, the argument clearly states savings accounts have not decreased, so there is no ambiguity and this cannot be your answer. The correct choice is answer D.

20. Answer: C

STEP 1:  Read the question and identify your task.

This is an Assumption question. The question asks you to find the assumption upon which the argument relies.

STEP 2:  Read the argument with your task in mind.

The argument assumes that increased spending by those not employed by the new businesses would result in an unusual decrease in their savings.

STEP 3:  Know what you’re looking for.

You review your options with that understanding in mind.

STEP 4:  Read every word of every answer choice.

Whether answer A is true or not is irrelevant to the argument since it does not focus on sales. Rather, it focuses on spending by a certain subgroup of the population. Answer B actually contradicts the argument by saying that the people not employed by startups are in fact spending more by borrowing money from relatives. Answer C states the opposite from answer B, saying that people not employed by the business startups are not borrowing to make large purchases, which would support the notion that if they were making large purchases their savings would decrease. This is very likely your answer, but you must review the remaining options to be certain. Answer D also contradicts the argument by merely restating in other words the reason that people are expected to spend more. The argument says this is not the case because savings have not decreased. Finally, answer E discusses people who are employed by startups, but the argument is only concerned with people not employed by the startups and their spending habits. This cannot be your answer. The correct choice is answer C.

21. Answer: A

STEP 1:  Read the question and identify your task.

This is an Assumption question. It is asking you to find among the options the one answer that identifies the assumption that makes the argument work.

STEP 2:  Read the argument with your task in mind.

The argument says that criminal actions are simply human nature and concludes that the prison system should be abolished immediately. Therefore, the assumption is that criminals should not be punished for their crimes because they are naturally predisposed to commit those crimes.

STEP 3:  Know what you’re looking for.

The correct answer will state that criminals cannot be held accountable for the crimes they commit.

STEP 4:  Read every word of every answer choice.

Answer A fits your requirement perfectly because it states that only human nature can be held responsible for crime. This may very well be your answer, but you must review the remainder of the options to be certain. Answer B is not only inaccurate—there is no ambiguity inherent in the term nature in this argument—but it also does not address the issue of accountability, and therefore is not your answer. Answer C seems way off the mark because the argument is not based on any statistical evidence, and it is not trying to extract observations of criminals to make a generalization on the entire population. And again, it does not address the issue of responsibility, so this cannot be correct. Answer D cannot be the correct choice because the argument does not distinguish criminals from crimes. It directly links them in the first sentence. Answer E seems to be discussing another argument altogether, since the argument you are concerned with does not discuss socially acceptable or socially unacceptable actions. These are vague terms when your discussion deals with the more specific actions of crime. Thus, the correct choice is answer A.

22. Answer: D

STEP 1:  Read the question and identify your task.

This is a Deduction question. The question asks that you select from among the possible answers the one statement that must be true after taking into account the given statements regarding a country’s natural gas inventory.

STEP 2:  Read the argument with your task in mind.

The discussion says that the country’s supply is down from the year before and that the country has not imported or exported natural gas in 10 years.

STEP 3:  Know what you’re looking for.

The correct answer will have to be consistent with a scenario in which what is used decreases the inventory and what is not used remains in inventory. There is no external use or source of inventory.

STEP 4:  Read every word of every answer choice.

Answer A is impossible to know because you do not know anything about the inventory that carried over from 2008. The inventory was higher in 2009, but the carryover from 2008 could have been very little or none, so it is possible that consumption was much lower than 2010. There is simply no way to know for sure. Without the 2008 data or consumption numbers, answer B is difficult to conclude. You cannot know which year had more mining. Answer C muddles things by breaking the years into halves, but the statements only discuss full years and there is nothing in the statements that enables you to conclude anything with regard to parts of the year. This cannot be your answer. Answer D says more natural gas was consumed than mined in 2010. You know that some inventory carried over from 2009, so for the inventory in 2010 to be lower than that in 2009, the country must have consumed more natural gas in 2010 than was produced. This must be your answer. Answer E requires that you know the inventory that carried over from 2008. Otherwise, you cannot know how consumption in 2010 compared to 2009. The correct choice is answer D.

23. Answer: B

STEP 1:  Read the question and identify your task.

This is a Deduction question. The question asks that you assess Jeri’s interpretation of Richard’s statement.

STEP 2:  Read the argument with your task in mind.

Jeri says that all prefaces and prologues are meaningless and recommends that all prefaces and prologues be eliminated from literature. This recommendation is based on Richard’s assessment that they are extraneous and serve no purpose.

STEP 3:  Know what you’re looking for.

The correct answer will likely have something do with Jeri’s equating “extraneous and serve no purpose” with being “meaningless,” since this is the justification for her recommendation that prefaces and prologues be eliminated from literature. It will behoove you to pay close attention to the language of each answer.

STEP 4:  Read every word of every answer choice.

Answer A uses the word useful, which is not a term used by either Richard or Jeri. While “not useful” can be seen as an equivalent of “extraneous and serve no purpose,” you must be careful not to make such a leap, even if it seems warranted. Answer B is a viable option because this is how Jeri perceives Richard’s meaning, but in the positive. Jeri does think that Richard is saying that only literary tools that serve a purpose (Richard’s exact words) have meaning, which could be flipped to say all literary tools that serve no purpose are meaningless. This is most likely your answer, but you must review the remaining options to make sure there is not a better one. Answer C is very similar to answer B, but in the interest of choosing the best answer you might consider answer C a lesser choice. It changes the word meaning to meaningful, which has a different connotation and it lacks the absolute meaning that the word only offers in answer B. Answer B is still your best choice so far. Answer D does not link Jeri’s remarks to Richard’s. It also uses the word useful, which is not the same terminology as that used by Richard. Answer E takes the logical leap that Jeri makes in her statement, but it does not explain her interpretation and how it might lead to the conclusion to eliminate prefaces and prologues from literature. The correct choice is answer B.

24. Answer: C

STEP 1:  Read the question and identify your task.

This is a Describe question. The question asks that you qualify what exactly Lonnie is pointing out in his criticism of Joanie’s statement.

STEP 2:  Read the argument with your task in mind.

Joanie observes an increase in property losses and fatalities due to natural disasters and concludes that there has been an increase in natural disasters. Lonnie agrees about the increased property losses and fatalities but says there are other reasons for the increase, including increased development and more “large-scale” disasters, and these other reasons undermine Joanie’s conclusion.

STEP 3:  Know what you’re looking for.

The correct answer will most likely say that Lonnie agrees with the result but not necessarily with the underlying causes.

STEP 4:  Read every word of every answer choice.

For answers A and B, Lonnie never questions the statistics and actually uses them in his criticism, so you can eliminate both these options. Answer C is fairly close to what you want. Lonnie accepts the statistics but states that they do not represent the underlying causes that lead to her conclusion. This is most likely your answer, but you must review the remaining options. Answer D cannot be your choice because Lonnie never questions Joanie’s qualifications. Lastly, regarding answer E, Lonnie never brings up other statistics. He accepts the same statistics that Joanie uses, but he interprets them differently. Thus, the correct choice is answer C.

25. Answer: C

STEP 1:  Read the question and identify your task.

This is a Parallel question. The question asks you to read the argument and find another argument that uses a similar form of reasoning, questionable though it may be.

STEP 2:  Read the argument with your task in mind.

In this case, the argument states that a political leader is considered great based on past decisions or actions and only on this basis, and that being considered great now is no basis for predicting future performance by the leader. In essence, this is saying that an assessment can be made based on past conditions, but given that assessment you cannot predict whether that assessment will remain valid in the future.

STEP 3:  Know what you’re looking for.

Scanning through the answers, you notice they all are oriented around the same subject, ear infections in children. The correct answer will also make a statement about past or present situations not allowing you to predict the future of that condition.

STEP 4:  Read every word of every answer choice.

Answer A gives you a fact about the unpredictability of the various bacteria that cause ear infections and says that if the child has an ear infection, it is impossible to predict how it will progress. This answer switches from the unpredictability of bacteria to the unpredictability of the infection, but your sample remains focused on one subject, a leader, so this answer does not follow the same pattern. Answer B discusses children who do not contract ear infections, but it never discusses whether their special nature allows you to predict or not predict whether they will contract one in the future, so this does not follow the pattern. Answer C essentially argues that children are only considered to have an ear infection if they show the symptoms, but having such symptoms does not allow you to predict whether such symptoms will continue in the future. This would seem to approximate the logic of the argument and is likely your answer, but you must review the remaining options to be sure. Answer D starts out well, saying that only with the proper symptoms can a child be considered to have an ear infection, but then it says that the child may not have an ear infection because the symptoms are shared by other diseases. Nothing about predicting future ear infections is mentioned. Answer E never says that a child has an ear infection. Instead it speaks generally about the disease itself. Although the last sentence approximates the conclusion of your statements, this cannot be your answer. The correct choice is answer C.

SECTION II

1. Answer: B

STEP 1:  Read the question and identify your task.

This is an Assumption question. It is asking you to identify among the answers the assumption upon which the argument bases its statement that “Slim-gurt” already dominates the market.

STEP 2:  Read the argument with your task in mind.

The argument bases its claim on an opinion poll in which 76 percent of the respondents consider “Slim-gurt” the dominant yogurt on the market. Then, you learn that a company that enjoys more than 55 percent of sales in the marketplace is considered dominant.

STEP 3:  Know what you’re looking for.

The correct answer will justify the leap from the poll to the dominance qualification in the marketplace.

STEP 4:  Read every word of every answer choice.

The argument does not make any claim about future dominance. It is only concerned about the current situation, so answer A cannot be correct. Answer B states that what the consumer believes is true is in fact true in the marketplace. This would justify the argument’s link between the consumer opinion poll and the conclusion that the company is in fact dominant in the marketplace. This is most likely your answer, but you should review the remaining options to make sure. Answer C says that the consumer belief in dominance is a requirement or precondition for dominance, but there is nothing in the statements that implies such a causal relationship. Most of the argument revolves around the current situation, not a causal one. Answer D focuses on the market research but says nothing about market dominance, which is the main idea of the argument. Also, nothing in the argument implies that the company is ignoring the market research. Answer E may be true, but it is speculative about a future dominance while the argument is concerned with current dominance. The correct choice is answer B.

2. Answer: E

STEP 1:  Read the question and identify your task.

This is a Strengthen question. The question asks that you identify among the answers the one that is justified by the statements in the argument.

STEP 2:  Read the argument with your task in mind.

The argument says that there are many schools of thought as to how to achieve happiness, but most people would not consider someone who follows every tenet of one of these schools to be happy. This seems to imply that either happiness is unachievable or that there is some other solution to achieving happiness.

STEP 3:  Know what you’re looking for.

The correct answer will support the notion that people have their own definition of happiness that may not be addressed by the schools of thought.

STEP 4:  Read every word of every answer choice.

Answer A cannot necessarily be concluded from the argument because the argument does not say that psychology has failed to accurately describe happiness. It says only that abiding by every prescription of the theories may not lead to happiness as people define it. Answer B discusses the difference in results between following competing theories. Nothing in the argument suggests that some psychological theories regarding happiness are mutually exclusive, so this cannot be the correct choice. Regarding answer C, the argument does not say that happiness as defined by the theories is not achievable in practice. Rather it says that achieving it might not be perceived as happiness by most people. Answer D may be true, but the argument is only concerned with their definition of happiness with regard to those who abide by psychological theories. Answer E says that most people’s conception of happiness does not match that defined by psychology and this is definitely supported by the argument, because in essence it is saying that most people who look at those who abide by the psychological theories do not see their definition of happiness. The correct choice is answer E.

3. Answer: D

STEP 1:  Read the question and identify your task.

This is an Assumption question. It asks that you identify an assumption upon which the Commission bases its argument.

STEP 2:  Read the argument with your task in mind.

Reading through the League’s and the Commission’s arguments you learn that the Commission believes that due to conservation efforts, unfettered logging will have no effect on the country’s wildlife.

STEP 3:  Know what you’re looking for.

The correct answer will support the notion that the nation’s conservation efforts are adequate or that unfettered logging will not cause harm to those efforts.

STEP 4:  Read every word of every answer choice.

Answer A says that the Aligorian wildlife includes some of the most threatened species, but this would seem to be the basis of the League’s argument, not the Commission’s. The Commission’s argument is that its conservation efforts are adequate regardless of what wildlife lives in the forest. Answer B also seems to be supporting the League’s argument and not the Commission’s, since the League is arguing that there should be a continuance or increase of such regulation, while the Commission is arguing against further regulation. Answer C says that agricultural nations need not have logging regulated, but neither the League nor the Commission tells you that Aligoria is a primarily agricultural nation, so this cannot be the correct choice. Answer D says that the indigenous species will survive in specially constructed habitats. The assumption is that such conservation efforts are adequate to protect the nation’s threatened species. Even though it seems rather general, this would fit your expectations for a valid assumption upon which the Commission bases its argument. Though not perfect, it might be your best option. Answer E is again an assumption supporting the League’s argument and not the Commission’s, since the Commission believes that uncontrolled logging will not affect its wildlife. The correct choice is answer D.

4. Answer: A

STEP 1:  Read the question and identify your task.

This is a Strengthen question. It asks that you identify a principle among the possible answers that supports the Commission’s position.

STEP 2:  Read the argument with your task in mind.

Since the Commission believes that the nation’s conservation efforts have done well enough to allow unfettered logging, you would expect the principle to justify allowing an unregulated logging industry to thrive.

STEP 3:  Know what you’re looking for.

The correct answer will most likely support the notion that regulations should be slowed or not instituted at all.

STEP 4:  Read every word of every answer choice.

Answer A says that regulations should not be implemented until logging has actually caused a reduction in population of an endangered species. This is a fairly supportive statement for the Commission’s argument, since it supports holding off on any regulations until something bad actually happens. This is a good contender, but you must review the remainder of the answers to see whether a better one can replace it. Answer B puts wildlife preservation for future generations above the current economic needs of the nation, which does not support the Commission’s argument. Answers C and D are actually supported by the League’s statement, which argues for more control over logging and thus against further depletion of natural resources. The League’s argument also supports the notion that Aligoria must handle its own environmental problems without regard to other nations’ experiences. Thus, these answers are unsupportive of the Commission’s argument, which is what your question is asking for. Finally, answer E is supported by neither argument since both argue under the assumption that it is Aligoria’s responsibility to deal with the issue, not an international governing body. Thus, the correct choice is answer A.

5. Answer: B

STEP 1:  Read the question and identify your task.

This is a Principle question. It is asking that you apply Avi’s reasoning to a judgment in a different situation.

STEP 2:  Read the argument with your task in mind.

Goni argues that the tax penalty is not her fault because her advanced age has resulted in her having poor handwriting, which led to the bookkeeping error. Avi argues that regardless of such a physical limitation, Goni was aware of the problem and its effect on her bookkeeping and thus was taking a risk in her job, and thus she can be held responsible for her mistake.

STEP 3:  Know what you’re looking for.

The correct answer will be a situation where a person knowingly takes a risky action and must be held accountable for the resulting negative effect.

STEP 4:  Read every word of every answer choice.

Answer A says that Jack was not responsible for losing his board position. You know immediately this cannot be right because your answer must hold the person responsible. Answer B says that Jack was responsible for missing the recital because he held a meeting that he knew would go on for too long. Here, a person did something knowing it was risky and he must be held responsible for the consequences. This would seem to be your answer, but you must review the remaining options to be sure. Answer C says that Jack is responsible for something his brother did and had no control over just because he knew his dog would be scared by that action. This does not follow your pattern since Jack was not taking the risky action himself. Answer D says Jack is responsible for offending a friend by telling her something that he had no idea she would take personally. This does not follow your pattern because he did not know that his action would result in such an offense, so he did not willingly take such a risk. Finally, answer E says that Jack is responsible for something that happened to him because he did not take recommended precautions. The key with this one is that the consequence happened to him rather than being something that he in fact caused. The correct choice is answer B.

6. Answer: D

STEP 1:  Read the question and identify your task.

This is a Flaw question. The question asks that you identify among the answers the faulty strategy used by the editorial.

STEP 2:  Read the argument with your task in mind.

The editorial argues that the governor should ignore the statements made by the chief prosecutor due to the prosecutor’s own criminal conviction in the past.

STEP 3:  Know what you’re looking for.

The correct answer will most likely focus on the editorial’s strategy of using the prosecutor’s past behavior to justify its recommendation.

STEP 4:  Read every word of every answer choice.

Answer A cannot be the correct choice because the editorial does not support the plan recommended by the prosecutor. It actually argues against implementing the plan. Answer B is out of scope. The editorial speculates as to the success or failure of the prosecutor’s plan based on the prosecutor’s character, not a particular aspect of the plan. Answer C is a bit vague, but while the editorial does presuppose that the prosecutor is untrustworthy, this is not what the editorial is setting out to prove. The editorial is not really trying to prove anything. It is merely making a recommendation. Answer D says the editorial is criticizing the prosecutor rather than addressing the prosecutor’s plan itself, and this is exactly what the editorial is doing. This is most likely your answer, but you have one more option to consider. Answer E states that without proof that the prosecutor’s plan has worked before, then it is destined to fail. Again, the editorial focus is on the character of the prosecutor, not the viability of the plan. The correct choice is answer D.

7. Answer: E

STEP 1:  Read the question and identify your task.

This is an Assumption question. The question asks you to determine which answer is an assumption upon which the lawyer’s argument is based.

STEP 2:  Read the argument with your task in mind.

The lawyer argues that it is always justified to protest a moral injustice and then argues that it is always justified to protest taxation without representation. You see quickly that the argument equates taxation without representation with moral injustice.

STEP 3:  Know what you’re looking for.

The correct answer will most likely state something similar.

STEP 4:  Read every word of every answer choice.

Answer A is just a restatement of the last sentence in the argument and is not really an assumption upon which the argument is based. Answer B speaks of danger, but the argument says nothing about the danger of protesting. It does not even imply such danger. Answer C offers a qualification for an injustice to be considered moral, but the argument assumes the injustice is already considered moral regardless of whether it is protested or not, so this qualification does not function in this case. Answer D tries to reverse an “if … then” statement given by the lawyer. The lawyer says if it is a moral injustice, then it should be protested. This answer is saying that if an injustice can be protested, it must be of a moral nature. You cannot reverse an “if … then” statement and expect it to be true, and the lawyer’s statements do not support such a reversal. Answer E says that taxation without representation is a moral injustice. This fits your expected answer and is, in fact, the correct answer since it connects logically the lawyer’s second statement concerning injustice and the statement regarding taxation without representation. The correct choice is answer E.

8. Answer: D

STEP 1:  Read the question and identify your task.

This is a Weaken question. It is asking you to find among the possible answers the one that describes Brandon’s counter to Arisha’s statements.

STEP 2:  Read the argument with your task in mind.

Arisha argues that the city’s campaign to reduce waste has been successful based on certain facts and statistics. Brandon argues that the timing and certain events prove that progress was made despite the city’s campaign.

STEP 3:  Know what you’re looking for.

The correct answer will point out how Brandon uses his facts to call into question Arisha’s conclusion with regard to the campaign.

STEP 4:  Read every word of every answer choice.

Answer A cannot be your choice because Brandon never states that the campaign was not effective. He only points out that other circumstances may have been responsible for the positive results, so this cannot be the correct choice. Also, Brandon never makes any claims that the campaign was unnecessary, nor does he point out the commonality of recycling, so answer B cannot be correct either. Answer C says Brandon takes issue with the length of the campaign, but his argument has more to do with the timing and circumstances under which it ran than the length of the campaign. Answer D says Brandon presents facts that weaken the force of Arisha’s evidence, and this is exactly what Brandon’s argument is doing. This is most likely your answer, but you must review the last option to be sure you have the right one. Answer E says that Brandon doubts the city really wants to reduce waste, but this is clearly not so. He only doubts the facts upon which Arisha bases her conclusion. Therefore, the correct choice is answer D.

9. Answer: D

STEP 1:  Read the question and identify your task.

This is a Parallel question. The question asks that you find among the possible answers the one that has reasoning that matches the reasoning in the argument.

STEP 2:  Read the argument with your task in mind.

In essence, the argument says that most people who depend on their cell phone have no landline. In Ronde County, most people depend on their cell phone. Therefore, in Ronde County most people don’t have a landline. The logic goes from general to the specific without taking into consideration that the specific group, in this case Ronde County, might have its own distinguishing characteristics.

STEP 3:  Know what you’re looking for.

You look at the answers for a similarly flawed logic.

STEP 4:  Read every word of every answer choice.

Answer A bases a conclusion about the citizens of Eastland on crime facts about Eastland. There is no generalization applied to a specific group, so this does not follow your expected pattern. Answer B is problematic from the start because its conclusion is regarding one person, not a group. Answer C has the same problem as answer A; it judges a group based on facts about that group. There is no generalization being applied to them. Answer D says that most people who live in retirement communities take part in early voting, and since most people in Daytown live in retirement communities, then most of them must take part in early voting. This is exactly the reasoning used in your argument. A generalization regarding retirement communities and early voting is applied to the people of Daytown who mostly live in retirement communities. This is most likely your answer, but you must review the last option to make sure you have found your correct answer. Answer E has the same problem as answers A and C in that it judges a group by facts about that group. No generalization is applied to them. The correct choice is answer D.

10. Answer: C

STEP 1:  Read the question and identify your task.

This is a Weaken question. The question asks that you identify the one statement among the answers that fails to support the argument that the new sales maps will be more useful.

STEP 2:  Read the argument with your task in mind.

The argument rests on the idea that the maps are based on newer data sourced from more locations around the country.

STEP 3:  Know what you’re looking for

The correct answer will most likely fail to support that expectation or will simply have nothing to do with it.

STEP 4:  Read every word of every answer choice.

Answer A supports the claim because obviously if the data have changed over the last 10 years, newer data will be more useful. Answer B supports the claim because it justifies the need for the data in the first place: the data provide information about the company’s customers. Answer C says retail centers have been open more than 10 years, but it is unclear how this supports the claim that the new report will be useful. This seems only to indicate that the older report excluded these centers for some reason, but you do not know why. This may be your answer, but you must review the remaining options to be sure. Answer D supports the claim because it justifies the inclusion of input from the network of loyal customers. Lastly, answer E supports the claim because it justifies the use of more data in the new report. You are left with answer C, which does not seem to support any particular claim at all. The correct choice is answer C.

11. Answer: C

STEP 1:  Read the question and identify your task.

This is a Deduction question. The question asks that you choose from among the answers the statement that describes what exactly the editorialist and the producer disagree on.

STEP 2:  Read the argument with your task in mind.

The editorialist believes that TV dramas contain enough violence to actually increase violence in society. The producer believes TV dramas just reflect what people want to see and that limiting content would be censorship.

STEP 3:  Know what you’re looking for.

You would expect the correct answer to center on the role TV dramas play in fostering violence in society.

STEP 4:  Read every word of every answer choice.

Answer A is promising because the editorialist does believe dramas influence people’s conception of the norm while the producer believes dramas are just answering people’s demand, but it is also true that the producer does not directly address the influence issue, which makes this answer somewhat inadequate. You must review the remainder of the answers to see if this is the best answer. Regarding answer B, the editorialist does not argue that viewers should not be able to decide what they want to see on TV. The editorialist only describes the influence of dramas, so this cannot be your answer. Answer C says they disagree whether TV dramas are causing violence in society by depicting violent crimes, and this is exactly what they disagree on. The editorialist surely believes this is true, and the TV drama producer says in the first statement that such an increase is not because of TV shows. This is a better answer than A and is probably your best choice. Answer D would be a good choice if the editorialist actually argued for censorship, but he or she does not. Only the producer brings up censorship. Answer E cannot be your choice because the editorialist and the producer both accept that violence has increased in society. The correct choice is answer C.

12. Answer: D

STEP 1:  Read the question and identify your task.

This is a Strengthen question. The question asks that you identify the statement among the possible answers that most supports the billboard company executive’s argument.

STEP 2:  Read the argument with your task in mind.

You read the executive’s statements and learn that the executive defends an increase in rates by arguing that there will be no ill consequences as a result, namely that advertisers will still profit from this form of advertising and they will still be able to find billboards to rent.

STEP 3:  Know what you’re looking for.

You can assume the correct answer will support this logic by giving credence to the claim that billboard advertising will continue to be successful for advertisers.

STEP 4:  Read every word of every answer choice.

Answer A differentiates billboards leased by service providers from those rented by product makers. The executive makes no such distinction, so this cannot be your answer. Also, without knowing the amount of increase and decrease for these groups, you cannot know whether the net change supports or weakens the executive’s argument. Answer B discusses the cost of production or delivery of services, but the executive’s reference to profitability is intended to refer to the benefit derived from advertising expenditures, not the traditional accounting profit as this answer suggests. Answer C says short leases will no longer be available to advertisers, but the length of the lease is irrelevant to the executive’s argument, since it only concerns pricing, not length of leases. Answer D suggests that billboards will enjoy more exposure going forward. This does support the executive’s claim that despite the price increase, billboards will continue to be a good investment for advertisers. This could very well be your answer, but you must review the last option to be sure. Answer E says a measurement factor will change for billboards, but it does not say what effect that change will have on the metrics for billboard advertisers. This option is too tangential to be your answer. The correct choice is answer D.

13. Answer: A

STEP 1:  Read the question and identify your task.

This is a Flaw question. It asks you to identify among the possible answers the one that describes accurately a flaw in how the program will achieve its goal.

STEP 2:  Read the argument with your task in mind.

By reading the description of the program you learn that the moderators will attempt to find out which politicians are more effective, males or females, by asking the politicians themselves.

STEP 3:  Know what you’re looking for.

The correct answer will probably point out that the program is going to foolishly ask politicians to assess their own effectiveness, something even a nonpolitician would have difficulty doing well.

STEP 4:  Read every word of every answer choice.

Answer A says that the target of the politicians’ treatment would be a better source for evaluation, and this is another way of saying that the politicians are not the right people to be asking to evaluate their own effectiveness. This is probably your answer, but you should evaluate the remaining options to make sure you have selected the right one. Answer B may be true, but it does not seem to have anything to do with the television show’s topic, which focuses on the politicians, not advocacy programs that hold them accountable on this matter. Answer C might be a flaw if the politicians on the program work together on policy. There is nothing to suggest such a relationship. They could be politicians in completely different governing bodies, so this cannot be your answer. Answer D describes a situation beyond the scope of the show. It does not matter whether or not homeless and indigent citizens vote. Finally, answer E may be true, but the program is not focused on how many male or female politicians there are in the country. Thus, the correct choice is answer A.

14. Answer: B

STEP 1:  Read the question and identify your task.

This is a Conclusion question. The question asks that you identify the statement that gives the essence of the business consultant’s point.

STEP 2:  Read the argument with your task in mind.

The consultant is arguing that streamlining for the sake of doing anything to reduce costs is not worth it if it hurts the quality of service and reduces customer satisfaction.

STEP 3:  Know what you’re looking for.

The correct answer will simplify that very sentiment that streamlining for streamlining’s sake is insufficient reason to do it.

STEP 4:  Read every word of every answer choice.

Answer A cannot be your answer because the consultant does not express any preference for a particular solution and actually makes an argument against streamlining as a means to just reduce costs. Answer B says that the effectiveness of streamlining at reducing costs is not justification enough to do it, which is exactly what the consultant is saying and it matches what you formulated as your expected answer. This is very likely your answer, but you must review the remaining options. Answer C restates what the consultant is arguing against, that streamlining to reduce costs is better than not doing it, so this cannot be your answer. Answer D cannot be correct because the consultant says only that streamlining may hurt quality of service and reduce customer satisfaction, and the consultant does not make an evaluation that streamlining is more or less harmful in this way. Answer E cannot be your choice either because the consultant does not discuss the possibility that streamlining might improve these aspects of the business. The consultant only mentions the risk that the opposite will happen. The correct choice is answer B.

15. Answer: A

STEP 1:  Read the question and identify your task.

This is a Parallel question. The question asks that you identify which of the sets of statements relies on the same flawed logic used in the argument.

STEP 2:  Read the argument with your task in mind.

You read the argument, paying close attention to the logic exhibited. You are given an expected pattern, then you are given a fact that Valerie knows, and finally you are given a dubious assumption Valerie makes by fitting her fact to the expected pattern.

STEP 3:  Know what you’re looking for.

The correct answer will follow a similar pattern even if the statements are not in the same order.

STEP 4:  Read every word of every answer choice.

Answer A is tricky because the statements are reordered, but they follow your pattern. You get the concluding assumption first, that Jamey knows she will get a raise. Then you are given the pattern that Jamey will receive a raise if everyone in her department receives a raise. Finally, you are given the fact that feeds the assumption, that Jamey knows everyone in her department received a raise. The order is different, but the pattern is there and this is most likely your answer. However, you must review the remaining options in case there is a better one. Answer B is close, but the conclusion is an expected result in the future, not something that has already happened. Also, there is no person making the logical assumption. This cannot be your answer. Answer C is problematic because none of the statements involves someone making the assumption based on an acknowledged pattern and a fact known to that person. Answer D gives you Phil’s belief and the assumption he makes, but there is no pattern upon which that assumption is based. Answer E does not have a dubious assumption. It is a pattern that leads to a valid “if … then” statement. If she is an employee, then she will be allowed to eat in the company break room. Thus, the correct choice is answer A.

16. Answer: C

STEP 1:  Read the question and identify your task.

This is a Parallel question. The question asks that you find among the answers the one that uses the same logic as the park ranger’s argument.

STEP 2:  Read the argument with your task in mind.

You read the ranger’s argument, paying close attention to the ranger’s logic. The ranger argues that restrictions are to protect less-experienced climbers and enacting such restrictions would only force them to go to other, more dangerous parks to climb. The logic is that rules are meant to protect someone, but if by enacting those rules you are actually causing more harm to those people, you should not enact those rules.

STEP 3:  Know what you’re looking for.

The correct answer will follow a similar pattern.

STEP 4:  Read every word of every answer choice.

Answer A argues that the rule was to protect someone, but now that the problem is gone, the rule no longer should be enforced. This does not match your pattern. Answer B argues that a rule is in effect for a good reason, but priorities have changed, making the rule less important, but in the long run, problems may arise so the rule should remain in effect. This is obviously not your pattern either. Answer C says that a rule is used to protect someone but this rule actually causes harm to that person, and since that harm is greater than it would be without the rule, that rule should not be in effect. This is very close to your pattern and is most likely your answer. You must review the remaining options to be sure. Answer D says a rule will protect people, but that rule will have an unintended negative consequence to other people and thus the rule should not be instituted. This is close, but this answer has the rule having an ill effect on people other than those who must abide by the rule. In your pattern the ill effect is experienced by those directly affected by the rule. Answer E says that a program can protect something but that something is already protected against the danger, so the program should not be instituted. This is not your pattern because it is saying that no harm will come to the protected something because it already has enough defenses against the danger. Thus, the correct choice is answer C.

17. Answer: D

STEP 1:  Read the question and identify your task.

This is a Strengthen question. This question asks you to identify among the possible answers the axiom that gives support to the ranger’s argument.

STEP 2:  Read the argument with your task in mind.

You read the ranger’s argument, paying close attention to the justifications for his argument. The ranger argues that restrictions are to protect less-experienced climbers and enacting such restrictions would only force them to go to other, more dangerous parks to climb. The logic is that rules are meant to protect someone, but if by enacting those rules you are actually causing more harm to those people, you should not enact those rules.

STEP 3:  Know what you’re looking for.

The correct answer will say that rules should not be put into effect if they cause more harm than if they were not in effect.

STEP 4:  Read every word of every answer choice.

Answer A may or may not support the ranger’s argument depending on the instructors in the ranger’s area. The ranger does not give you this information and thus, this cannot be your answer. Answers B and C both are principles supporting the opposition to the ranger’s argument. They both support the idea that restrictions should be instituted, either to avoid harm or for legal reasons, so these cannot be correct. Answer D says the rules should not be instituted if those rules will cause more harm than the harm they are supposed to eliminate. This is very much in support of the ranger’s argument and is likely your answer. Answer E is a principle that supports the ranger’s opposition, although more weakly than B and C. The correct choice is answer D.

18. Answer: E

STEP 1:  Read the question and identify your task.

This is a Deduction question. The question asks you to identify the statement that cannot be true based on the statements in the argument. Four out of the five options are true.

STEP 2:  Read the argument with your task in mind.

The argument might seem complicated, but you can simplify it quickly. It says that for the first eight months, Cenpan increased its output and market share while the output of the entire industry (including Cenpan) remained constant. After the regulations, Cenpan’s share decreased but its output remained the same, while the output of the entire industry (including Cenpan) remained constant. You can deduce that in the first eight months the companies other than Cenpan lost market share and decreased output. Also, you can deduce that after the regulations, because Cenpan’s output did not change and its market share decreased, the total output of companies other than Cenpan must have increased. Most likely more companies came into the market.

STEP 3:  Know what you’re looking for.

You judge the answers based on these deductions. If an answer is possible, then it cannot be the correct answer.

STEP 4:  Read every word of every answer choice.

Answer A is possible. Even though nothing in the argument indicates that this will definitely happen, there is no reason to say it would not happen. Answer B is not only possible, it is certain, for if Cenpan gained market share, then the other companies as a whole must have lost market share. Answer C is possible. You know nothing about Cenpan’s costs or pricing, and it is hard to believe that the increased cost of abiding by the new safety standards did not adversely affect the company’s profit margin. Nevertheless, there is nothing in the argument to indicate that the company’s average profit did not improve after the imposition of the new safety standards. Answer D also is possible because nothing in the argument would cause us to deny that the company might have been worse off if the safety standards had not been imposed. Finally, answer E cannot be true, because you know that after the imposition of the safety standards, Cenpan’s market share decreased while its output remained the same. For that to happen, the other companies as a whole must have produced more. Thus, their output could not have decreased. The correct choice is answer E.

19. Answer: D

STEP 1:  Read the question and identify your task.

This is a Weaken question. The question asks you to identify the one statement among the possible answers that most challenges the argument.

STEP 2:  Read the argument with your task in mind.

You read the argument and discover that it claims, put simply, that some business taxes are acceptable even if burdensome because business is able to function with other burdens that nobody objects to.

STEP 3:  Know what you’re looking for.

You expect the correct answer to challenge this notion by saying that the taxes are in fact a worse burden than suggested in the argument and should be opposed.

STEP 4:  Read every word of every answer choice.

Answer A cannot be your choice because it is outside the scope of the argument. What happens to nonprofits and individual citizens has no bearing on the argument, which is only concerned with businesses. Also, the argument says nothing about failures. Answer B might be used to support the argument, saying that further restrictions would occur if it were not for the imposition of such taxes. Answer C is a restatement of the argument but using income taxes instead of the regulatory burdens as the comparison, and since this supports the argument, this cannot be your choice. Answer D says that taxes mentioned in the argument are in fact so burdensome that business activity is almost impossible. This would be a strong challenge to the notion that such taxes are no more burdensome than regulation and thus tolerable. This may be your answer, but you have one more option to consider. Answer E offers support to the argument by saying that such taxes are actually helpful to businesses. The correct choice is answer D.

20. Answer: B

STEP 1:  Read the question and identify your task.

This is a Deduction question. This question asks that you choose a statement among the possible answers that can be inferred from the argument.

STEP 2:  Read the argument with your task in mind.

The argument states that some business taxes are acceptable even if burdensome because business is able to function with other burdens that nobody objects to.

STEP 3:  Know what you’re looking for.

The correct answer will have something to do with the level of burden that taxes, regulations, and so on, have on businesses and whether businesses can viably function under that burden.

STEP 4:  Read every word of every answer choice.

Answer A contradicts the argument by saying the taxes are too burdensome, and thus, this statement cannot be inferred. Answer B says that being burdensome to business is not reason enough for a prohibition, and this is exactly what the argument is saying about the business taxes. This statement can be inferred from the argument and is most likely your choice, but you must review the remaining options to be sure. Regarding answer C, the argument says that excessive income taxes are not justified, but it never says that they should be prohibited. It is conceivable that the person making the argument might say this, but given the argument you cannot know this. Answer D says that business failures are a bigger problem than excessive taxes, but the argument says that business failures are just a symptom of what is really the more important issue, infrastructure problems. This inference is mixing up issues to confuse you and cannot be your answer. Answer E says that the nonexcessive taxes are not burdensome at all, but this is not what the argument implies. Actually, the argument says that they do create a burden but a burden no different from that created by regulations that are considered acceptable. Therefore, the correct choice is answer B.

21. Answer: E

STEP 1:  Read the question and identify your task.

This is a Strengthen question. The question asks that you identify the statement among the possible answers that helps the argument reach its conclusion.

STEP 2:  Read the argument with your task in mind.

After reading the argument you determine that the statement must link sales training to Fred’s success at winning the sales challenge over Gigi.

STEP 3:  Know what you’re looking for.

The correct answer will link sales training to winning the challenge.

STEP 4:  Read every word of every answer choice.

Answer A would be correct if the conclusion involved Gigi, but it doesn’t. The conclusion is regarding Fred, so this cannot be your answer. Answer B would be useful if the conclusion was a matter of who had more sales training than the other, but your conclusion is concerned only with Fred’s sales training. Answer C restates something already established or implied by the first statement in the argument. Gigi has obviously been a better salesperson in the past, but you are only concerned with supporting the conclusion, which explains why Fred beat Gigi in the challenge in the fourth quarter, not with what is usual or in the past. Answer D is a viable option for your choice. It says that if Fred took training, he would win the sales challenge. You might choose this as your answer if you stopped here, but you must review the final option. Answer E is your strongest contender. It says that the only way Fred could win over Gigi is if he took sales training. The certainty of its term only makes it a much stronger choice. The correct choice is answer E.

22. Answer: C

STEP 1:  Read the question and identify your task.

This is a Deduction question. The question asks that you identify among the answers the one statement that must also be true, when taking into consideration the statements in the argument.

STEP 2:  Read the argument with your task in mind.

The argument describes how people make decisions on major purchases. It speaks about them in terms of groups, using words like mostsome, and majority. These terms are important to help you identify your answer.

STEP 3:  Know what you’re looking for.

Since the “majority” is the most exact number and the largest, the correct answer will most likely be about that group.

STEP 4:  Read every word of every answer choice.

Answer A may be true but does not necessarily have to be so. Since so few do research on their own (according to the argument) and rely on other advice, then it is only slightly less likely that this group is in the minority that are not happy with their purchases than it is that they are in the majority that are happy with their purchases. Answer B is trying to trick you: it seems to be saying exactly what the argument is saying, but it is not. The argument says that most people make the purchases without doing any research, but only some of those are relying on advice from friends or a salesperson and only some make the decision based on emotions. You have no idea what share each of those groups composes within the larger group, so answer B may or may not be true and thus cannot be your answer. Answer C is your best option so far. You know that a majority of buyers are happy with their purchases, and you know that most buyers make these purchases without doing any research. Logically, majority and most are interchangeable, and there must be some overlap of these groups. Therefore, you know that at least some of the buyers who do not do any research must be happy with their purchases. This is most likely your answer, but you must review the remaining options. Answer D is impossible to know. You know only that some of the people who purchase without research use advice from their friends or salespeople, and this group could easily be in the minority that is unhappy with their purchases. Nothing gives you an indication that this group overlaps with the majority that is happy with their purchases. Answer E is possible, but there is no way to know. There is no indication as to the level of overlap with these groups. As indicated with answer C, you know that at least some of those who buy without research are happy with their purchases, but any more than that is impossible to know. The correct choice is answer C.

23. Answer: C

STEP 1:  Read the question and identify your task.

This is a Describe question. The question asks that you identify the reason that the argument cites the statistics.

STEP 2:  Read the argument with your task in mind.

The argument discusses a particular view, that smaller class size improves education. The statistics seem to contradict this notion, or at least they indicate that class size is an insufficient solution.

STEP 3:  Know what you’re looking for.

The correct answer will most likely describe how the statistics counter the view that class size is the answer to the problem.

STEP 4:  Read every word of every answer choice.

Answer A cannot be the correct choice because it indicates that the statistics support the idea of smaller class size. Answer B is a bit too extreme. The argument definitely uses the statistics to create doubt about reduced class size as a solution, but it does not use the statistics to eliminate it outright. Answer C is a viable option. The argument does indeed use the statistics toward establishing that the size of classes is not the only influence. For now, this is your best choice, but you need to review the remaining options in case there is a better one. Answer D, like answer B, is too extreme. The argument does not seek to completely eliminate the idea of reducing class size, especially since the statistical sample is described as a mix of class sizes. If the argument wanted to eliminate the notion, it would compare small classes with large classes. This cannot be your answer. And the viability of standardized testing as a measurement of an effective school system is outside of the scope of the argument, so answer E is incorrect. The correct choice is answer C.

24. Answer: A

STEP 1:  Read the question and identify your task.

This is a Parallel question. It asks that you identify the argument among the possible answers that uses the same reasoning as in the main argument.

STEP 2:  Read the argument with your task in mind.

You read the main argument paying close attention to how the logic functions. In essence, it tells you that an activity (industrial activity) has a positive effect (haze for plants) and that such activity is beneficial to everyone involved without reservation.

STEP 3:  Know what you’re looking for.

The correct answer will describe an activity that has a positive effect, and it will say that everyone who engages in that activity derives such benefit without reservation.

STEP 4:  Read every word of every answer choice.

Answer A says that vigorous exercise is good for people and that no harm can come from it. While this does not follow the same pattern as your argument, it is very similar. It says that an activity (vigorous exercise) has a positive effect (prevents ailments) and that activity is beneficial to everyone involved without reservation. This would seem to be your answer, but you have to review the remaining options to be sure. Answer B is easy to eliminate right off because it describes an action’s negative effect and simply makes a judgment that the effect justifies avoiding that action. This is not your pattern at all. Answer C is close to using the same reasoning, but it makes an extreme recommendation, beyond that suggested by the possible benefit, and the argument does not make a recommendation that people increase or transform all activity to industrial activity. Answer D does the same as answer C; it makes a drastic recommendation based on an observed benefit. The argument does not recommend that people increase their industrial activities based on the observed benefit. Answer E gives a negative effect and then recommends against the causal behavior. This is not your pattern. Also, the answer gets a bit too specific with its recommendation (take a day off). Your argument remains very general with its statements. The correct choice is answer A.

25. Answer: C

STEP 1:  Read the question and identify your task.

This is an Assumption question. The question asks that you identify among the possible answers the assumption upon which the columnist’s argument is based.

STEP 2:  Read the argument with your task in mind.

You read the argument seeking the basis for the columnist’s conclusion. The columnist argues that the reliance on e-mail and Internet social networking threatens the strong bonds of mutual trust necessary for a democratic society.

STEP 3:  Know what you’re looking for.

The correct answer will link e-mail and Internet social networking to this negative effect.

STEP 4:  Read every word of every answer choice.

Answer A cannot be your choice because the argument makes no claim with regard to effectiveness of social networking as a tool for civic organizations. This may be an implication of the argument but not an assumption upon which it depends. Regarding answer B, the argument may suggest that these people are not forming the necessary strong bonds, but it is not saying that they are incapable of it, so this cannot be the correct choice. Answer C is very promising. The argument that e-mail and social networking are corroding democracy is based on the judgment that these activities keep people from forming strong social bonds outside their families. This would seem to be your answer, but you should review the remaining options to be sure. Also, while the argument mentions family, it makes no judgment regarding the closeness of citizens to their families. It only judges their ability to form bonds outside the family. Thus, answer D cannot be correct. Finally, answer E is an implication or restatement of the second statement and is certainly a partial basis for the argument, but it is not comprehensive enough to consider as an assumption upon which the argument depends. The correct choice is answer C.

SECTION III

Passage 1

1. Answer: E

STEP 1:  Read the question and identify your task.

This is a Main Idea question. The question asks you to identify among the possible answers the one statement that represents the main idea of the passage.

STEP 2:  Go back to the passage to find the answer.

Refer to your passage summary.

STEP 3:  Read every word of every answer choice.

Answer A expresses the opposite of what is discussed in the passage, saying that justices are striving to ignore precedents set by foreign courts, but the passage discusses how more and more justices are paying close attention to precedents set by foreign courts. Answer B makes too pointed a statement against the critics of comparativism. The passage actually acknowledges the validity of these critics’ concerns. Answer C could be a statement made by the critics of comparativism, but it does not encompass the full scope of the passage. Answer D cannot be your choice because the passage says that comparativism has been around since the nineteenth century, so it is not relatively new and cannot be considered a passing fad. Finally, answer E seems on the mark. It correctly articulates the author’s point of view, who suggests in the second paragraph of passage B that some of the arguments made by the opponents of comparativism are legitimate. The correct choice is answer E.

2. Answer: C

STEP 1:  Read the question and identify your task.

This is an Inference question. It asks that you find among the possible answers the one that expresses the author of passage A’s attitude toward the opposition to comparativism.

STEP 2:  Go back to the passage to find the answer.

Scan for key words in the second paragraph that clue you into the author’s perspective. The author states that comparativism’s supporters “rightly” contend that the United States is taking on an increased role in world events.

STEP 3:  Read every word of every answer choice.

Answer A is too one-sided and we can quickly eliminate it as an option. Answer B has the basic disagreement between the two authors correct, but there is no evidence to support either the “appreciative” or “wrongheaded” claim. Answer C is most likely your answer. Since the author of passage A suggests that the increasingly role of the United States in world affairs is inevitable and that it is right to be involved in that legal framework, then the author must also believe that those who ignore this inevitability must be wrong and are ignoring the reality of the global legal and political situation. You must review the remaining options to see whether any one of them is better. Answer D is too strong a condemnation of the opponents to comparativism. The author is certainly not that antagonistic to their opposition. While the author of A is not “scornful,” neither is that author “dismissive” of the claims made by the opponents of comparativism, making answer E unlikely. Therefore, the correct choice is answer C.

3. Answer: B

STEP 1:  Read the question and identify your task.

This is an Inference question. The question asks that you identify among the possible answers a statement about the opposition to comparativism that can be inferred from the passage.

STEP 2:  Go back to the passage to find the answer.

Look for the correct answer in paragraph four, which is where the author explores opponents’ views.

STEP 3:  Read every word of every answer choice.

Answer A is too extreme. The anti-comparativists did not object to the Supreme Court decisions regarding executions, anti-sodomy laws, and affirmative actions. Rather, they objected to the court using comparativism to make those decisions. Answer B might very well be inferred from the passage. The opponents definitely believe that the United States should develop its own unique legal precedents based on its own legal system as set forward by the nation’s forefathers. This is one of the bases for opposing comparativism. This is most likely our answer but we must review the remaining options. Answer C cannot be inferred from the passage because the author never discuses the qualifications of those involved in the public discourse. As far as we know it is possible the opposition is more versed in foreign court systems. Regarding answer D, nothing in the passage indicates one way or the other the opposition’s views on globalization. Also, none of their objections are based on globalization. Finally, while the opposition took measures to restrain comparitivism in the courts they never suggested that the structure of the justice system be changed. The correct choice is answer B.

4. Answer: B

STEP 1:  Read the question and identify your task.

This is a Line ID question. Go back to the statements concerning Kennedy and Breyer made in passages A and B and find the point of overlap.

STEP 2:  Go back to the passage to find the answer.

Passages A and B only mention that both Kennedy and Breyer have used comparativism in making judicial decisions. Look for any answer that suggests this commonality in the use and acceptance of foreign law.

STEP 3:  Read every word of every answer choice.

Answer choice A discusses a point of view held by the opponents of comparativism, not Kennedy and Breyer. Answer choice B, however, tells you that foreign law can be used in foreign courts, something you know that the two justices believe. This must be your correct answer. Answer choice C is too extreme. While you know that Kennedy and Breyer have used foreign law in making their decisions, there is nothing to suggest that this is the central concern. Answer choice D compares the use of comparativism in the United States to other nations, something that is never discussed. And answer choice E is too extreme. While Breyer and Kennedy are both identified as Supreme Court justices, you don’t know that they believe comparative law to be only applicable in the Supreme Court. The correct choice is answer B.

5. Answer: C

STEP 1:  Read the question and identify your task.

This is an Information Retrieval question. The question asks that you find an analogy that the opponents of comparativism might use for those who use comparativism in domestic cases.

STEP 2:  Go back to the passage to find the answer.

You refer to passage B and look for comparisons.

STEP 3:  Read every word of every answer choice.

Answer A says doctors will consult other doctors, but the opposition would support judges consulting other judges to make a decision, so this does not fit. Answer B says teachers use guides, some of which are for teaching foreign students, but the opposition would not oppose judges referring to law books or even law books used by foreign judges to understand American law, so this analogy does not work. Answer C says a board member uses the policies of another, very different organization to write its own policies. The opposition would very well oppose judges using the law of another nation to write law in the United States. This analogy seems close enough to be your answer, but you should review the remaining options. Answer D says the candidate argues that a rule is bad because other countries do not have that same rule. The opposition would not object to a judge voicing his or her opinion regarding US law versus foreign law. This might lead to politicians changing the law, so this analogy is not right. Answer E is a company applying the rules in a stricter nation universally to its product. This analogy does not fit because the judge is not judging a law solely on the standards of a foreign country and then discarding it altogether. Therefore, the correct choice is answer C.

6. Answer: D

STEP 1:  Read the question and identify your task.

This is an Inference question. The question asks that you identify an inference that can be derived from the passage’s discussion of Justice Kennedy’s use of comparativism.

STEP 2:  Go back to the passage to find the answer.

You refer to the passage and see that opposition members suggested impeaching Kennedy, which is a harsh reaction. The correct answer will reflect this.

STEP 3:  Read every word of every answer choice.

Answer A says that the opposition supported the decision but not Kennedy’s use of comparativism. Again, the vehemence of their reaction to his decision does not support this as a possible correct answer. Answer B also suggests that the opposition supported the decision and does not seem to be implied by the passage. Answer C says the opposition was of two minds with regard to comparativism, but the passage does not indicate any such doubt. The passage implies the opposite, that opponents were fairly strict in wanting no foreign involvement at all in determining law in the United States. Answer D, that the opposition perhaps utilized the comparativism argument as a cover, could very well be inferred from the passage and is exactly what you expected to see as your correct answer. Answer E is too extreme. While opposition members wanted to impeach Kennedy, they did not suggest that he had committed treason, nor did they demand a trial to determine if he had done so. The correct choice is answer D.

7. Answer: E

STEP 1:  Read the question and identify your task.

This is a variation of a Main Idea question—a Primary Purpose question. The question asks you to identify among the answers the one that expresses the primary purpose of the passage.

STEP 2:  Go back to the passage to find the answer.

Refer to your passage summary.

STEP 3:  Read every word of every answer choice.

Answer A cannot be the correct choice because the passage is not justifying comparativism. Answer B attributes too strong a position to the author. The author is not so biased as to be considered a critic of either side of the comparativism argument. Regarding answer C, the author cannot be considered to be proposing the use of comparativism. It is already in use and the author does not seem to be promoting it. In fact, at the end of the passage the author indicates that adopting it is more a matter of giving in to certain globalization pressures than anything the author has to say about the practice. Answer D cannot be correct because it ignores the fact that the author uses much of the passage to discuss the opposition to comparativism. Finally, answer E must be your answer. The passage is intended to discuss the nature of a legal controversy, in this case comparativism, and it does describe the opposing sides of the issue. The correct choice is answer E.

Passage 2

8. Answer: C

STEP 1:  Read the question and identify your task.

This is a Main Idea question. The question asks that you identify the main idea of the passage from among the possible answers.

STEP 2:  Go back to the passage to find the answer.

Refer to your passage summary.

STEP 3:  Read every word of every answer choice.

Answer A takes the last paragraph and exaggerates the importance of its observation that biophysical economics is subject to the same flaws as other schools of economics. This answer also goes further than the passage in saying that biophysical economics should be discarded because of those flaws, a recommendation that the passage definitely does not make. For answer B, the passage does say that there is some difficulty applying the EROI to certain aspects of the economy, but it does not make such a harsh assessment of the energy focus, nor does the passage say that its influence on predictive models is dubious since the passage also discusses some predictive models based on the EROI. Answer C seems to fit the passage perfectly. The passage indicates that the failure of predictive models based on classical economics has opened the way for new economic schools of thought and that economists along with non-economists have formulated biophysical economics. This would seem to be your answer, but you should review the remaining options. Answer D is wrong in many ways. First, the passage makes no claim that biophysical economics has replaced neoclassical economics, only that it has come about because of certain failures by the older school of thought. Second, the passage makes clear that biophysical economics does not use the same concepts as traditional economics and in fact has developed its own concepts based on energy. Lastly, answer E cannot be your answer because the passage states that biophysical economics is a break from even environmental economics, so even though it is somewhat related by its concern for natural resources, it is not a subset of environmental economics. Thus, the correct choice is answer C.

9. Answer: A

STEP 1:  Read the question and identify your task.

This is an Inference question. The question asks that you determine which of the possible answers can be inferred from the theories put forward by biophysical economists.

STEP 2:  Go back to the passage to find the answer.

You refer to the passage.

STEP 3:  Read every word of every answer choice.

Answer A says that an economy can survive as long as it has a stable or increasing EROI. This is exactly how the passage describes the function of the EROI. It says that an economic system requires a positive EROI in order to survive. This is probably your answer, but you must review the remaining options to be certain. Answer B tries to trick you by linking natural resources to energy, but the focus of this statement is still natural resources, which is the central focus of ecological economics, not biophysical economics. Answer C is an argument that would be made by neoclassical economics, which believes that market forces can solve most major economic problems. Answer D seems viable as a choice, but the passage really does not indicate what solutions these two groups offer for solving the world’s economic problems, so you do not know whether they agree or not about solutions. You know only that these two groups have different ways of approaching and defining the problems. Answer E cannot be concluded from the information you have. You know only that the ecologists tend to focus less on energy than other proponents of biophysical economics. Thus, the correct choice is answer A.

10. Answer: C

STEP 1:  Read the question and identify your task.

This is an Information Retrieval question. The question asks that you choose which event among the possible answers would most directly affect a biophysical economic analysis.

STEP 2:  Go back to the passage to find the answer.

Since their approach is based on the EROI, you are looking for an event that directly affects the energy delivered by a process or used in that process.

STEP 3:  Read every word of every answer choice.

Answer A seems like a good potential answer. A consumer switch from gasoline-powered to electric-powered automobiles may indirectly affect energy consumption, but this is still a consumer preference and a market-based influence and it could be masking an underlying process that is using more energy (like the production of electric cars). Answer B involves government intervention in the economy, and taxes will only affect consumer behavior and not the underlying energy delivery or consumption. Answer C definitely affects the EROI economic model. If the cost to maintain current energy supply levels goes up significantly, it could affect the long-term viability of that energy source and thus the survival of the economic system. This is most likely your answer, but you must review the remaining options. Answer D involves debt in the equation and debt may or may not be related to increased costs. It could also be due to poor management. Because you do not know the reason for taking on more debt, you cannot know whether or not this affects the biophysical economic model. Answer E says the new technology helps the company avoid an increase, which means the cost remains the same and thus has no impact on the economic model at all. The correct choice is answer C.

11. Answer: E

STEP 1:  Read the question and identify your task.

This is an Information Retrieval question. The question asks you to identify among the possible answers the view that the biophysical economist has of oil supplies for energy.

STEP 2:  Go back to the passage to find the answer.

Look back at paragraph three and read the last sentence, which states that biophysical economists believe the EROI for oil has already begun its descent, which means that the cost of producing oil is becoming increasingly expensive while supplies have not risen equally.

STEP 3:  Read every word of every answer choice.

Answer A says oil is scarce and expensive, but the passage implies that oil has not reached its peak in supply, so the first part is wrong even if the second part is right. Answer B says oil is stable or slightly decreasing and cheaper to source. The first part might be right, but the second part is incorrect: the passage makes it clear that oil is becoming more expensive to source. Answer C says the supply is unknown and it is cheaper to source. Again, the second part cannot be right according to the passage. Answer D is completely wrong. Oil is not decreasing in supply and it is not inexpensive to source, according to the passage. Answer E is your last available option, and it is right on target. The economists view oil as abundant or stable in supply and it is increasingly expensive to source, which is why the EROI has already begun its descent. The correct choice is answer E.

12. Answer: D

STEP 1:  Read the question and identify your task.

This is a Main Idea question. The question asks that you identify the answer that best describes the function of the final paragraph within the passage.

STEP 2:  Go back to the passage to find the answer.

The final paragraph is essentially saying that biophysical economics is not immune to the problems that other economic schools experience, so the paragraph is attempting to equalize things a bit between the new type of economics and the old.

STEP 3:  Read every word of every answer choice.

Answer A is in many ways the antithesis of what the final paragraph is saying, establishing the superiority of biophysical economics over neoclassical. This does not match what you know and cannot be your answer. Answer B says that certain practicalities will hinder the ability of biophysical economics to surpass classical economics, but the last paragraph says only that biophysical economics experiences the same problems as classical economics, not that it will not overcome those problems and even surpass the other schools of thought. Answer C feeds off the second sentence, which indicates a small dissonance between the ecologists and the rest of biophysical economics supporters, but to say that the entire paragraph functions to show the “conflict” between them is extreme and inaccurate. Answer D matches your understanding of how the last paragraph functions. The last paragraph does indeed explain that the new economics is not immune to the problems that have plagued classical economics and made it problematic for noneconomists. This is probably your answer, but you must review your last option. Answer E says the last paragraph makes the disagreement within the biophysical economics community into an advantage over classical economics, but you know that the paragraph implies that the flaw makes the new economics more similar to classical economics, at least as an economic discipline if not in substance. Thus, the correct choice is answer D.

13. Answer: A

STEP 1:  Read the question and identify your task.

This is a variation of a Main Idea question—the Primary Purpose question.

STEP 2:  Go back to the passage to find the answer.

Refer to your passage summary.

STEP 3:  Read every word of every answer choice.

Answer A is pretty much on target. The passage does explore the substance and the problems surrounding a new economic theory that is supposed to supplant an earlier, more dominant economic theory. This option may very well be your answer, but you must explore the remaining options as well. Answer B cannot be correct because the passage does not favor the neoclassical economic model over the biophysical economic model. If anything, the passage gives the new model equal standing to the classical model. Answer C makes the economic crisis responsible for the development of the biophysical economic theories, but the passage says the crisis resulted in disenchantment with the old model. It does not make a causal connection between the crisis and the development of new models. Actually, the passage seems to make more of a connection between environmental concerns and the new models. This answer also seems too limited to the first paragraph, while the remaining sections of the passage do much more than what is described in answer C. Answer D focuses on the last paragraph and ignores all that came before. This is too limited a statement to be your primary purpose. Answer E cannot be your answer because the passage does not really give the theories of biophysical economics or neoclassical economics regarding the economic crisis. Therefore, the correct choice is answer A.

Passage 3

14. Answer: B

STEP 1:  Read the question and identify your task.

This is a Main Idea question.

STEP 2:  Go back to the passage to find the answer.

Refer to your passage summary.

STEP 3:  Read every word of every answer choice.

Answer A says that the passage puts doubt on Galileo’s work, but you know from the passage that the doubt is not with his work but with the story behind the church’s persecution of Galileo, so this answer cannot be correct. Answer B encapsulates the idea of the passage perfectly. The passage does in fact purport that there are new theories as to why Galileo was targeted by the church and it does say that his trial and house arrest are fact. This may be your answer, but you must review the remaining options. Answer C focuses on a part of the passage that mentions that Santillana was influenced by the political environment in the United States, but while this observation may show the author’s own doubts regarding his theories, the author does not suggest that Santillana’s theories be discounted on the basis of the factual elements in the case. Answer D is a promising candidate, but it falls just short of answer B by not mentioning the last paragraph and its factual claims. Also, it says that scholars are “currently pursuing explanations,” but the passage indicates that much of the pursuit has already occurred. Either way, answer B is still a better option. Answer E makes the error of saying that the existence of a trial is in question, when this is one of the elements that is considered fact by the passage. The correct choice is answer B.

15. Answer: C

STEP 1:  Read the question and identify your task.

This is an Inference question. The question asks you which historical political event influenced Santillana while writing his book.

STEP 2:  Go back to the passage to find the answer.

Reread the section about his book, The Crime of Galileo. Santillana wrote about how Galileo’s contemporaries turned on him and denounced him to the church to protect their own way of life. You look through the events and assess their similarity to Santillana’s description of Galileo’s situation.

STEP 3:  Read every word of every answer choice.

Answer A cannot be correct because Galileo’s contemporaries obviously did not bomb Galileo’s home. Answer B cannot be right because the fall of Galileo was not the fall of a homicidal dictator, nor was it the rise of a new type of government. Answer C is promising. The House Un-American Activities Committee demanded that people suspected of being un-American come before the committee and name their associates who were Communists or consorting with Communists. Scientists and artists denounced their friends in order to save their own careers. This is probably your answer due to its striking similarity to Galileo’s situation, but you should review the remaining options. Answer D cannot be correct because Galileo and his peers were not in a standoff that resulted in two sides building up armaments, which led to an indirect conflict. Answer E cannot be your choice because Galileo’s peers were not supporting an industry that would supplant Galileo’s outdated form of industry. Note: If you were unfamiliar with the House Un-American Committee hearings you could easily eliminate the incorrect answers, which would leave answer C as your only optionThe correct choice is answer C.

16. Answer: D

STEP 1:  Read the question and identify your task.

This is an Information Retrieval question. The question asks you to identify the statement that Pietro Redondi would agree with regarding Galileo Galilei.

STEP 2:  Go back to the passage to find the answer.

Refer to the part of the passage that explains Redondi’s views. Redondi believed that Galileo’s persecution by the church had nothing to do with his theory that the earth revolved around the sun (i.e., Copernicanism) and not the other way around. Redondi believed it had more to do with Galileo’s belief in atomism, which contradicted the church’s belief in transubstantiation.

STEP 3:  Read every word of every answer choice.

Answer A focuses on the forged injunction, but Redondi regards the Copernicanism argument irrelevant to his theory, and there is no evidence in the passage for his views on the injunction. Answer B says that Galileo’s contemporaries betrayed him out of jealousy, but the passage says that they did so to protect themselves. Also, Redondi believes the church invented the Copernicanism justification not to protect Galileo’s contemporaries but to abide by the wishes of wealthy benefactors who liked Galileo. Answer C cannot be your choice because nothing in the passage indicates that Galileo’s face-off with the church was at the urging of the Medici family. Also, the passage says Redondi believed his house arrest rather than the death penalty was probably the result of the intervention of the Medici family, not the result of their inaction. Answer D would probably be supported by Redondi. He believed that Galileo’s real crime of believing in atomism was covered up with the Copernicanism justification to gratify the Medici family. That indicates Copernicanism was less offensive than atomism, and this is probably your answer, but you have one last option. Answer E puts Orazio Grassi, S.J., at the center of the entire story, and while Redondi might entertain such a notion, it is doubtful he would agree with this since Pope Urban VIII must have been involved as the leader of the church. Therefore, the correct choice is answer D.

17. Answer: D

STEP 1:  Read the question and identify your task.

This is an Inference question. The question asks that you identify a relationship between Bruno and Galileo that can be inferred from the passage.

STEP 2:  Go back to the passage to find the answer.

Your only connection between the two in the passage is that one was executed (Bruno) while the other (Galileo) was spared. The passage suggests that Galileo was spared because he had the favor of the Medici family while Bruno did not have such a supporter. You can expect the answer will revolve around these connections in some way.

STEP 3:  Read every word of every answer choice.

Answer A says that Bruno and Galileo agreed on Copernicanism but differed on atomism, but the passage suggests no such thing. Actually, the passage gives no insight as to whether they agreed or disagreed on Copernicanism, and there is a slight suggestion that they agreed on atomism since the author uses their punishment differences as part of his argument. Answer B has the same problem as answer A. Answer C makes an unwarranted leap by linking them both to Orazio Grassi, who in the passage is linked only to the Galileo story. Also, there is no indication in the passage that Bruno committed a worse crime than Galileo. On the contrary, the passage suggests that they were equally guilty but received very different punishments. Answer D says that both scientists offended the church by teaching their beliefs, and one fared better due to the protection of a wealthy benefactor. This is exactly what you expected the answer to look like. The passage does imply that both offended the church by insisting on teaching their beliefs, but Galileo was saved from execution by his relationship with the Medici family. This is very likely your answer, but you should review your last option to be certain. Answer E mentions that they differed on God’s relationship to the order of the universe. This topic is not even discussed in the passage, and their agreement or disagreement on the matter cannot be ascertained. The correct choice is answer D.

18. Answer: B

STEP 1:  Read the question and identify your task.

This is an Inference question. The question asks that you choose a statement from among the possible answers that describes the relationship between the church and publications during the time of Galileo.

STEP 2:  Go back to the passage to find the answer.

This question deals with the last part of paragraph three, which discusses how the church’s list of proscribed publications figured into Redondi’s analysis.

STEP 3:  Read every word of every answer choice.

Answer A cannot be your choice because the passage questions why The Assayer was not added to the proscribed list if the ideas in it were offensive to the church, suggesting that offensive books were denied registration with the church. Answer B says that an author probably would not have been put on trial for a book that had not been proscribed, and this is exactly the question posed by the passage. The author questions why The Assayer was not on the proscribed list if, according to Redondi, Galileo was really persecuted for the ideas put forward in this book. This answer is very much implied by this section of the passage and is probably your answer, but you must review the remaining options. Answer C is a bit extreme. Nothing in the passage indicates that every author of a book that was proscribed was put on trial and (except for Galileo) burned at the stake. Answer D may be an answer to the question posed by the author of the passage, but it is not the implication of the question. Answer E is an interesting theory and might connect the dots for the author of the passage, much like answer D, but it is a bit too far a reach for an answer to this question. The correct choice is answer B.

19. Answer: C

STEP 1:  Read the question and identify your task.

This is a variation of a Main Idea question—the Primary Purpose question.

STEP 2:  Go back to the passage to find the answer.

Refer to your passage summary.

STEP 3:  Read every word of every answer choice.

Answer A is a viable option. The passage does discuss recent discoveries and give insight into the Galileo story, but the answer is a bit too general and words like summarize and discoveries seem rather inexact when applied to what is presented in the passage. The author is doing more than summarizing, and the passage presents scholarly theories in addition to discoveries. Therefore, this answer is somewhat unsatisfactory. Still, you should keep it in mind as you review the remaining options. Answer B cannot be the correct choice because the passage does not mention a conflict between two scholars. For all you know, Santillana and Redondi agree completely on each other’s theories. Answer C is a more specific and more accurate description of the passage’s purpose. The passage does present and assess Santillana’s and Redondi’s theories that call into question aspects of the Galileo story, so this is probably your answer, but you should review the remaining options to be certain. Answer D focuses on the church’s considerations when putting Galileo on trial, and this is certainly not the primary purpose of the passage since it ignores Santillana’s and Redondi’s theories altogether. Finally, answer E uses the word criticize to characterize the author’s purpose, and the passage does not exactly criticize Santillana’s and Redondi’s theories; rather, the author poses some questions that might cast some doubt on them. Still, the author seems to acknowledge that they do have some valid ideas. Therefore, the correct choice is answer C.

Passage 4

20. Answer: B

STEP 1:  Read the question and identify your task.

This is a Main Idea question.

STEP 2:  Go back to the passage to find the answer.

If needed, you can review the first sentence of each paragraph to remind yourself of the progression of thought through the passage.

STEP 3:  Read every word of every answer choice.

Answer A would be more likely a study performed by the psychologist Leonard Sax, but it does not encompass the full scope of the passage and cannot be the answer. Answer B seems like a good description of the passage. The passage does put forward two theories, both of which have questionable evidence to support them and have drawn criticism. This might be your answer, but you should review the remaining options to be certain. With answer C, language is important. The passage is not discussing two studies. Rather, it is discussing two approaches to single-sex education, each of which is under study. The difference is subtle but important and disqualifies this option. Answer D makes the mistake of saying that the practice is just beginning to be incorporated into public schools. The passage makes it clear that this is not a new practice. Finally, answer E is tempting because the passage does discuss single-sex education generally in this manner, but this answer ignores the two schools of thought with regard to single-sex education and thus cannot be your answer. The correct choice is answer B.

21. Answer: C

STEP 1:  Read the question and identify your task.

This is a Line ID question. The question asks you to find the answer that best describes the purpose of the specified lines.

STEP 2:  Go back to the passage to find the answer.

The lines in question discuss how the studies that have been done are not reliable as evidence to support single-sex education and that in some cases they actually undermine such support. The correct answer will be some variation on this summary.

STEP 3:  Read every word of every answer choice.

Answer A says the opposite of what the lines say. The content of the lines definitely does not favor single-sex education. Answer B is too innocuous. The lines do not simply exhibit or discuss the research. There is an obvious point being made by the lines discussing such uncertain studies. Answer C is an accurate description of the lines. The discussion of the questionable studies offers a caveat to any support offered by Sax or other supporters of single-sex education. This is probably your answer, but you must review the remaining options to be certain. Answer D says the lines give favor to Dr. Sax’s approach to single-sex education, but the lines speak of studies on single-sex education in general. They do not specify whether the studies diminish one approach in relation to another. Answer E cannot be your answer because these particular lines give no historical perspective. In fact, the lines that come before these are intended for that purpose. The correct choice is answer C.

22. Answer: D

STEP 1:  Read the question and identify your task.

This is an Inference question. The question asks that you identify the statement among the possible answers with which Dr. Sax would agree.

STEP 2:  Go back to the passage to find the answer.

You know from the passage that Dr. Sax believes that using genetic differences is how single-sex education should be formulated. So the answer will most likely be consistent with that perspective.

STEP 3:  Read every word of every answer choice.

Answer A says that social pressures should be involved in determining single-sex education, and this is consistent with the other school of thought, not Sax’s, so this cannot be your answer. Answer B says boys fear looking smart in front of girls, but this is not a genetic factor. Rather, it seems to be a social factor, something the opposing school of thought would use to justify single-sex education. Answer C says that because boys and girls test differently, they should be given different tests based on learning differences. This cannot be the correct choice because it is basing its conclusion on performance on academic tests rather than on genetic factors. Answer D says that because of an inherent difference between boys and girls, they should be separated into different environments that take this difference into account. This is much stronger than answer C, because the term inherent suggests that the difference is genetically determined, and using such a determinant in single-sex education is right in line with Sax’s views. This is most likely your answer, but you have one last option to review. Answer E argues for coed education based on testing, and Sax does not support coed education, so this cannot be your answer. The correct choice is answer D.

23. Answer: E

STEP 1:  Read the question and identify your task.

This is an Information Retrieval question. The question asks you to identify a statement that is consistent with the ACLU’s concerns discussed in the specified lines.

STEP 2:  Go back to the passage to find the answer.

The lines discuss the point that the ACLU is concerned that Sax’s approach will encourage dangerous stereotypes and ruin any gender progress that has been made in education. The correct answer will be consistent with this fear of stereotypes.

STEP 3:  Read every word of every answer choice.

Answer A is Sax’s approach in action, and while an imbalance is described, there is no indication that a dangerous stereotype is being encouraged. Answer B may be the negative effect of Sax’s theory in practice, but this is a result, not a stereotype being propagated. Answer C may be an effect of Sax’s approach, but this could be a positive result and the ACLU is not concerned that students might find jobs more appropriate to their genetic disposition. Answer D is another possible negative effect of Sax’s theory in practice, but this is not a stereotype being fostered. Finally, answer E says the perception that boys are better than girls for jobs with chaotic environments is a stereotype created or fostered by Sax’s ideas, and this is exactly what the ACLU is concerned about. The correct choice is answer E.

24. Answer: D

STEP 1:  Read the question and identify your task.

This is an Information Retrieval question. The question asks you to determine which of the statements is consistent with the views of single-sex education critics.

STEP 2:  Go back to the passage to find the answer.

Review the final paragraph, which deals mainly with the critics.

STEP 3:  Read every word of every answer choice.

Answer A says single-sex education is OK as long as the fundamentals of American education are taught, but this is not what the critics are saying. The passage says nothing about the critics taking issue with the content of the education. Rather, they object to the structure itself. Answer B has the same problem as answer A. It is not the content of education or some threat of learning un-American values that is the danger. Answer C is in support of single-sex education as a method of instilling American values, and this is definitely in conflict with your understanding of the critics’ point of view in the final paragraph. They believe quite the opposite. Answer D is very much in line with the critics’ point of view. They view the model of single-sex education as inherently flawed; the structure itself is an un-America idea of segregation. This is probably your answer, but you must review your last option to be sure. Answer E focuses on pedagogy, saying that single-sex education teachers are incapable of instilling good values because of the structure. This seems off the mark and is not as good an answer as D. The correct choice is answer D.

25. Answer: C

STEP 1:  Read the question and identify your task.

This is an Inference question. The question asks you to identify the statement that can be inferred regarding single-sex education after reading the passage.

STEP 2:  Go back to the passage to find the answer.

You refer to the passage.

STEP 3:  Read every word of every answer choice.

Answer A creates an “if … then” statement that is unsupported by the passage. It says that if convincing evidence is produced, then single-sex education will grow as a movement, but the passage says in the final sentence of the first paragraph that single-sex education will grow despite the evidence, so this cannot be your answer. Answer B attempts to apply the critics’ point of view as a general understanding about single-sex education, and this is not necessarily true. Answer C says that parents will continue to support single-sex education despite the lack of research supporting it, and this is implied in the last sentence of the first paragraph, which says the same thing in different words. This is probably your answer, but you should review the remaining options. Answer D is a bit too optimistic, and it is unsupported by the passage. The passage says that growth will continue, but it does not go so far as to suggest it will become mainstream. Answer E says that the sociological approach will win out over Dr. Sax’s approach, and this is definitely not suggested by the passage. The passage does not actually pick a winner or suggest that one will dominate the other. The correct choice is answer C.

SECTION IV

Questions 1–5

As with all logic games you follow the six-step process.

STEP 1:  Identify the game type.

You are asked to arrange the elements in sequential order, one speaker per day and according to certain conditions, so you recognize this as a one-tier ordering game.

STEP 2:  Begin your diagram.

Since there are six days in the conference, you can build a grid with six columns, each representing a day of the conference, and a placeholder for each. You shorten the names of the speakers to their initials (C, D, F, G, H, and J) and write them in the upper right corner above the grid.

Images

STEP 3:  Symbolize the clues.

In the game description, the first rule you notice is that there can be only one speaker per day and that each speaker cannot repeat. To symbolize your clues you go through them one by one.

Clue 1: Chapman and Dabu must speak on days that are separated from each other by exactly one day.

Using the box form for symbolization, you can symbolize this clue as follows:

Images

Since C and D could speak before or after each other, you alter it slightly as follows:

Images

Clue 2: Chapman and Gant cannot speak on consecutive days.

You can use the box form for symbolizing this clue as well, as follows:

Images

Notice that you use the same double arrow to indicate that C cannot be before G and G cannot be before C.

Clue 3: Gant must speak on a later day than Forest.

This can be symbolized simply by using a line to indicate the order in which they have to be placed:

F — G

Clue 4: Jonas must speak on day 3.

This requires no symbolization. You can insert J into your diagram where he belongs, on day 3.

STEP 4:  Double-check your symbolizations.

To double-check your symbolizations you translate your symbolized clues back into normal English and see whether they match the original language of each clue. Once you are certain that your symbolizations are correct, you can insert them beside your diagram for easy reference. Below is what your diagram should look like after you have created and confirmed the validity of all your symbolizations.

Images

STEP 5:  Make deductions.

1. Can’t-be-first-or-last deductions

From your third clue you know that F can never be last because G must come after F. You can put ~F above the sixth column on your diagram to represent that F can never be in that column.

2. Repeated-element deductions

You see that speaker C is repeated in the first two clues and G is repeated in the second and third clues. No hard and fast deductions can be derived from these repetitions. You can see that there might be situations where G is forced to be before or after D because C and D must have one day between them and G cannot occupy that gap, but depending on where C is located, this will not always be the case.

3. Down-to-two deductions

There are no down-to-two deductions that can be derived from the clues.

4. Block-splitting deductions

Even though there must be a day between them, C and D form a block. In essence, they form a block of three days if you include the unassigned day between them. Looking at your diagram, you notice that there are only two days before J, so you know that C or D cannot be on day 1, because that would force the other to be assigned to day 3, which is unavailable. You also know that neither can be assigned to day 5. This would also force the other to be assigned to day 3, and there is no day beyond day 6. To represent these, you can put ~C/D over days 1 and 5.

Adding your deductions you now have this diagram:

Images

STEP 6:  Answer the questions in the smartest order.

On test day, answer the questions in this order:

1. Answer the Complete and Accurate List question.

2. Answer questions that give you more information to work with.

3. Answer the remaining questions.

Questions 1 and 2 are Complete and Accurate List questions, although the second is not the typical variety of this kind of question.

None of the questions gives further information.

That leaves questions 3, 4, and 5, and these can be answered in that order.

THE ANSWERS

1. Answer: D

The question asks you to identify the one answer with an acceptable schedule for all six days. Using your diagram, you can quickly work through each answer and eliminate those that break with your clues. For example, you can quickly eliminate answer A because C and G are scheduled consecutively, which is inconsistent with your second clue. You can also eliminate answer B because it does not have J scheduled on day 3. You eliminate answer C because G is scheduled before F, which is inconsistent with your third clue. You can also eliminate answer E because C and D are scheduled consecutively, which is inconsistent with your first clue. You are left with answer D and this schedule is consistent with your clues. You can represent this schedule on the diagram as follows:

Images

The correct choice is answer D.

2. Answer: B

The question asks you to identify a complete list of the days that C can be assigned to. You know from your deductions that C cannot be scheduled on day 1 or day 5, so you can eliminate answers A and E. You also know that nobody but J can be scheduled for day 3, so you can eliminate answer C. You are left with answers B and D. The only difference between these answers is that B includes day 6 as a possibility, and you know from your rules that C can easily be scheduled for day 6 as long as D is scheduled for day 4 and G is scheduled on any day that is not consecutive with day 6. Both are possible, so answer B offers you a complete and accurate list for C. The correct choice is answer B.

3. Answer: A

The question asks you to identify the scheduling option among the possible answers that cannot be true. This means all but one of the options is possible. You can evaluate each answer at a time. For answer A, you use your diagram to test the scheduling of H on day 2. You see that C and D must occupy day 4 and day 6 (or vice versa) to have a day between them. According to your second clue, G cannot be scheduled consecutively before or after C. This forces you to schedule G on day 1, but this means that G is scheduled before F, which is inconsistent with clue three. Thus, it cannot be true that H is scheduled on day 2, and this is the correct answer to the question. For learning purposes, the remaining scenarios have been diagrammed to demonstrate that they all work.

Images

The correct choice is answer A.

4. Answer: D

The question asks which scenario must be true. Your experience up to this point will be very helpful in assessing each of the possible answers. Many of the answers you can eliminate based on the scenarios of question 3, but in case you skipped that question we will work through each answer here. The easiest way to test the answers is to create a schedule that does not abide by the scenario given in each answer, and if it is possible to create such a schedule, then the answer is not necessarily true. The one answer or schedule that is impossible to create must be your answer. For example, answer A says that either F or G is scheduled to speak on day 2. You test this answer by creating a schedule in which neither F nor G is scheduled on day 2. If such a schedule is consistent with the rules of the game, then the answer is not necessarily true and cannot be your answer. What follows are the scenarios you created to test each answer. As you can see, for answer D, it was impossible to create a schedule in which C or D was not scheduled on day 4.

Images

The correct choice is answer D.

5. Answer: C

The question asks you to identify which scenario among the possible answers cannot be true. Similar to question 4, you test each answer by attempting to create a schedule that is consistent with the answer and the rules of the game. If you can create such a scenario, then the answer can be true and it cannot be your answer. For example, answer A says that D and G are scheduled to speak consecutively. As you can see in the diagram that follows, you can create a schedule in which D and G are scheduled to speak on consecutive days and the schedule is consistent with the rules of your game, so this cannot be the correct choice. For answer C, it is impossible to create a schedule in which F and H are scheduled on consecutive days, because if you put them before day 3, then it forces G and C to be on consecutive days, which is inconsistent with clue 2. If you schedule them on days 4 and 5, then G must be scheduled on day 6 to be consistent with clue 3, and C and D must be scheduled on days 1 and 2, which is impossible because they must have one day between them according to clue 1. If you schedule them on days 5 and 6, then G will have to be scheduled before F and this is inconsistent with clue 3. Answer C cannot be true and must be your answer.

Images

The correct choice is answer C.

Questions 6–12

As with all logic games you follow the six-step process.

STEP 1:  Identify the game type.

This is a grouping game. You can tell that it is a grouping game because it is asking you to assign each student to a particular tutor and each tutor is assigned at least one student. The tutors in their respective disciplines serve as the groups (or group leaders, if that helps). There are six students to assign to four tutors (groups). There is a twist to this grouping because each student needs to be tutored in only one subject matter and all but one of the tutors can teach more than one subject.

STEP 2:  Begin your diagram.

Your diagram will be a grid with four columns, one for each group. You shorten the group names to their initials: S, U, W, and Z. You can do the same thing with the students—B, C, D, F, G, and H—and list them in the upper right corner above your diagram for reference. Since each group specializes in a particular subject matter, it might help to show each tutor’s specialties in the diagram. You can shorten those as well, to Ch, E, M, and P. Your initial diagram might look like this:

Images

STEP 3:  Symbolize the clues.

You go one by one through the clues and symbolize them.

Clue 1: At least Bjorn and Chaim are assigned to Willie.

This clue need not be symbolized. The clue says that B and C are assigned to W. It also says they might not be the only ones assigned to W. You can put B and C into the W column and they will remain there through each question.

Clue 2: At least Fran is assigned to Zane.

This is another clue that need not be symbolized. Similar to the previous clue, it says that F is assigned to Z and there may be more students assigned to Z. You can put F into the Z column and it will remain there through each question.

Clue 3: If Dottie is assigned to Upton, then Gertie needs tutoring in Math.

This clue can be symbolized, but you must first recognize that only two teachers tutor in math, S and W. You can reduce this clue to an “if … then” statement as follows:

D = U → G = S/W

STEP 4:  Double-check your symbolizations.

To double-check your symbolizations, you translate your symbols back into normal English and see whether they match the original language of each clue. In this case only one clue has been symbolized, but once you have verified that it works, then you can add your symbolizations to your diagram. Your diagram should look as follows:

Images

STEP 5:  Make deductions.

Finally, before you tackle the questions, you see if you can make any deductions based on the setup of the game and the clues. You go through each type of deduction.

1. Can’t-be-first-or-last deductions

This is not an ordering game, so this deduction does not apply to this particular game.

2. Repeated-element deductions

There are no repeated elements among the rules in this game.

3. Down-to-two deductions

There are no down-to-two deductions in this game.

4. Block-splitting deductions

There are no instances of block splitting in this game.

STEP 6:  Answer the questions in the smartest order.

Approach the questions in this order:

1. Answer the Complete and Accurate List question.

2. Answer questions that give you more information to work with.

3. Answer the remaining questions.

This group of questions does not have a Complete and Accurate List question. Questions that provide more information are:

Question 9 (“Fran and Heste need tutoring in the same subject …”)

Question 10 (“Dottie and Fran need tutoring in the same subject …”)

Question 11 (“Exactly two students are assigned to Upton …”)

Question 12 (“Bjorn, Chaim, Gertie, and Heste all need tutoring in the same subject …”)

That leaves questions 6, 7 and 8, which you can answer in that order.

THE ANSWERS

6. Answer: D

The question asks which one of the possible answers cannot be true. You evaluate each answer, one by one. You assume your diagram remains as drawn, with B and C in W and F in Z. You also notice that the subject matter (math, physics, etc.) does not come into play for this question. Placement is the only factor. For answer A, if G = U and D = Z, then you can put H with S and you have a valid grouping. For answer B, if G = S and D = Z, then you can put H with U and you have a valid grouping. For answer C, if F = Z and G = Z, then you can put D with S and H with U and you have a valid grouping. For answer D, if D = W and H = Z, you know from clue 3 that G must be with W or S. Regardless of which tutor G is assigned to, you are still left with no student to assign to U. Each tutor must have at least one student, so this scenario cannot work and it must be your correct answer. For answer E, if D = S and G = U, then you can assign H to any of the tutors and still have a valid grouping. The correct choice is answer D.

7. Answer: B

The question asks which scenario among the possible answers must be true. This means all but one of the answers has the possibility of being not true, but only one must be true no matter what the grouping. Again, you review each answer, one at a time. To determine whether the answer has the possibility of being not true, you attempt to create a scenario in which the stipulation in the answer is not true, and if you can create such a grouping and it is consistent with the rules of your game, then it cannot be your choice. You assume that your diagram remains as drawn, with B and C in W and F in Z. Also, you notice that the subject matter (math, physics, etc.) does not come into play in this question. Placement is the only factor. Answer A says that S tutors exactly one student, so you test whether more than one student can be assigned to S. You assign two students, H and D, to S and then G to U. This is a valid grouping, so answer A cannot be your choice. Answer B says that Z is assigned fewer than three students, so you can test whether Z can be assigned three or more students, but such a test is unnecessary. If Z is assigned three student and you already know that W is assigned two students, then you are left with only one student to assign to S or U. One of the tutors will be left with nobody to teach, and your game requires that each tutor be assigned at least one student. It must be true that Z is assigned fewer than three students and B must be your answer for this question. Let’s review the remainder of the possible answers for learning purposes. Answer C says U is assigned fewer than two students, so you test whether U can be assigned two or more students. You assign G and H to U, which leaves D to be assigned to S. This is a valid grouping and thus cannot be your answer. Answer D says exactly one student is assigned to Z, so you test whether more than one student can be assigned to Z. You assign D to Z, along with F, making it two students. You assign G to S, and H to U. This is a valid grouping, and this cannot be your answer. Finally, answer E says that W is assigned exactly two students. You already know that W must be assigned at least two students, B and C, so you test whether W can be assigned three. You assign G to W (along with B and C), D to U, and H to S. This is a valid grouping and cannot be your answer. Therefore, the correct choice is answer B.

8. Answer: A

The question asks that you find the one statement that cannot be true, no matter the grouping. That means all but one of the answers has the possibility of being true. You review each answer and test whether it can be true or not. To test the answer, you create a grouping that is consistent with the answer and the rules of the game, and if such a grouping is possible, then it cannot be the correct choice. If such a grouping is impossible, then it must be your choice. Your diagram remains as drawn, with B and C in W and F in Z. Also, this is the first question that uses the subject matter as a determinant for the grouping. It will change the dynamic of your scheduling. Answer A says D and G need tutoring in English, which means they must be with Z, since Z is the only tutor that can teach English. This would mean Z would teach three students (F included), and you know from question 7 that this cannot be true. If Z has three students and W already has two students, there is only one student left to assign to either S or U. The game requires that each tutor have at least one student, so the correct answer to this question is A. Let’s review the remaining options for learning purposes. Answer B says that D and H need to learn chemistry, which means they could be assigned to U or Z. There are numerous groupings possible here, but one could be that you assign D to Z and H to U, which leaves H to be assigned to S, and you have a valid grouping. Answer C says that D and H need tutoring in math. Only S and W teach math. You can assign both D and H to S and G to U and you have a valid grouping. Answer D says that D and G need tutoring in math. Only S and W teach math. You can assign D and G to S and assign H to U and you have a valid grouping. Finally, answer E says both D and B need tutoring in physics. Only U and W tutor physics. If both D and B are assigned to U, then clue 3 says you must assign G to S or W. You choose to assign G to S, and you have a valid grouping. The correct choice is answer A.

9. Answer: B

The question tells you that F and H need tutoring in the same subject and then asks what is the maximum number of students who need physics. Essentially, the question is asking you to create a situation that maximizes the number of students U and W are teaching under the given scenario. You use your diagram to illustrate this grouping. You know from your clues that F must be assigned to Z, but you do not know whether F is being tutored in chemistry or English. In the interest of maximizing the number available to assign to W, you will assume F is being tutored in chemistry. Therefore, you can assign H to U (if you assumed English, they would both have to be assigned to Z). This allows you to assign D to S and G to W. There is no way to assign more to U or W without depriving another tutor of a student. For the sake of this question you can assume all of W’s students are studying physics, and you know that H is studying chemistry. Therefore, the maximum number of students studying physics is three.

Images

The correct choice is answer B.

10. Answer: D

The question tells you that D and F need tutoring in the same subject and then asks you to determine which of the possible answers must be true. This means all but one of the answers may not true, but only one must be true no matter what the grouping. You know that F is assigned to Z, but you do not know in which subject, chemistry or English, F is being tutored, so D also may be tutored in either chemistry or English. Therefore, while F can be assigned only to Z, D can be assigned to either U or Z. You will test both scenarios. First, you will assign D to U. According to clue 3, if D is assigned to U, then G must be with either S or W. If you assign G to S, then H can be assigned to any tutor. If you assign G to W, then H must be assigned to S.

Images

In the second scenario you assign D to Z. In this case, G and H must be distributed between S and U.

Images

Now, you look at your answers and evaluate each one. Answer A says D and F are tutored in English, but you have already shown that it is possible for D to be tutored in chemistry, since Z and U both tutor chemistry. Answer B says at least two students need tutoring in math. If you assume all of W’s students are being tutored in physics, then it is possible that only one student (G or H) is tutored in math. Therefore, it is possible that this answer is not true. Answer C says at least two students are tutored in English. In the first scenario above, there are no students tutored in English (F and D are tutored in chemistry), so this answer has the possibility of being untrue. Answer D says that at least one of G and H is tutored in math. In both scenarios above, no matter the configuration, you see that either or both of G and H are being tutored in math. This answer must be true, so this is your choice. The final option, answer E, says that at least one of G and H must be tutored in chemistry. In the first scenario you see a grouping where neither G nor H is being tutored in chemistry, so this answer may be untrue. The correct choice is answer D.

11. Answer: B

The question tells you that U is assigned exactly two students and asks you to determine which of the answers gives facts that could be true. You must determine which is possible, which means that all but one of the answers are impossible. For the remaining unassigned students, D, G, and H, there are three scenarios for assigning exactly two students to U: DG, DH, GH. You can quickly eliminate the DG scenario because it is inconsistent with your clues. When D is assigned to U, G must be assigned to S or W, so G cannot be assigned to U. You are left with DH and GH. You create both of these scenarios in your diagram. Your initial diagram remains as drawn previously, with B and C in W and F in Z. Answer A cannot be correct because if G needs tutoring in chemistry, then D must need math. Answer B could be true, because when G needs Math, D could be learning chemistry. This is your answer, because it could be true. Answer C cannot be true because if H is learning math, D and G must be together with U, and this is impossible under the rules of the game. Answer D cannot be correct because under the provisions of the question, H must be with either S or U. Answer E cannot be correct because if H and G are both learning math, then there is only one student with U, and that breaks the rules set by the question.

Images

The correct choice is answer B.

12. Answer: C

The last question tells you that B, C, G, and H all need tutoring in the same subject and then asks you to identify the one statement among the possible answers that could be true. This means that all but one of the statements must be impossible. You know from your clues that B and C are assigned to W, and W tutors only math and physics. Therefore, S, W, and U are the only tutors who can receive G and H. If B and C are learning math, then at least one of G and H must go to S. Also, if B and C are learning physics, at least one of G and H must go to U. You evaluate each answer and eliminate those that are impossible. Answer A says that exactly two students learn physics. This is impossible. If the two physics students are B and C, then G and H must also learn physics and that adds up to more than two. If you try to put two physics students with U, then one of them must be G or H, which would mean B, C, G, and H are learning physics and again you have more than two. This answer cannot be true and cannot be the correct choice. Answer B says D is learning English. This is also impossible. Since G and H must be assigned to the same subject, either S or U will be left without a student. D must be assigned to either S or U. Answer C says D learns math. This is very possible if B, C, G, and H all learn physics. They will be distributed among U and W, leaving S to teach D. This is your answer, but let’s review the remaining options for learning purposes. Answer D says exactly two students learn English. This is impossible because you have already proven that G, H, and D must be assigned to S or U depending on the dominant subject. Thus, Z must have only one student and Z is the only tutor teaching English. This cannot be your answer. Answer E says that exactly three students learn chemistry. This is impossible because you know that B, C, G, and H learn the same subject and chemistry cannot be one of them. That leaves only two possible students to learn chemistry. The correct choice is answer C.

Questions 13–19

As with all logic games you follow the six-step process.

STEP 1:  Identify the game type.

This game is a two-tiered ordering game. It has all three of the telltale signs of this type of game. First, it has the language of both ordering and grouping games. For each inspector, all the buildings must be scheduled over six weeks, and within each week, the schedule of one inspector affects the schedule of the other inspector. Some buildings cannot be inspected before another building and certain buildings cannot be inspected by one inspector until the other inspector has inspected it. This is typical language for an ordering game. At the same time, each building must be assigned to a particular week with another building, and no building can be inspected by both inspectors in the same week, and they can only be inspected once by each inspector. This is typical grouping language.

STEP 2:  Begin your diagram.

Your diagram will take the form of a grid with the inspectors listed on the left and the weeks going from left to right. You can abbreviate government to G and commercial to C. The building letters appear above and to the right of the diagram and they are grouped into their respective sets (G and C). Since there are only two inspectors, you will not abbreviate their names.

Images

STEP 3:  Symbolize the clues.

Once you have your diagram, you can proceed to symbolize your clues. You symbolize them one at a time below.

Clue 1: Limon cannot inspect any government building until Harris has evaluated that building.

This can be reworded to say that if it is a government building, then Harris must inspect it before Limon. This can be symbolized as follows:

G → H — L

Clue 2: Harris cannot evaluate any commercial building until Limon has evaluated that building.

This is similar to the first clue and can be reworded to say that if it is a commercial building, then Limon must inspect it before Harris. This can be symbolized as follows:

C → L — H

Clue 3: Limon cannot inspect any two government buildings consecutively.

You can use box notation to symbolize this clue. There are several ways to symbolize this, but the simplest is as follows:

Images

You put G in the bottom boxes because in your diagram Limon’s schedule appears below Harris’s schedule. Another way to represent the same clue is to show each configuration of consecutively scheduled government buildings. You use the double arrow to show that regardless of order, these buildings cannot be inspected consecutively. For example:

Images

For the purposes of the exercise here, we will use the simpler one.

Clue 4: Harris must inspect building M during week 4.

This clue need not be symbolized. You can simply put M on Harris’s schedule during week 4.

STEP 4:  Double-check your symbolizations.

To double-check your symbolizations, you translate your symbolized clues back into normal English and see whether they match the original language of each clue. Once you have confirmed that your symbolizations represent the clues correctly, you can insert them to the right of your diagram for easy reference. Your page should look as follows:

Images

STEP 5:  Make deductions.

Before you tackle the questions, you see if you can make any deductions based on the setup of the game and the clues. You go through each type of deduction.

1. Can’t-be-first-or-last deductions

Because Harris must inspect each government building before Limon can inspect it, you can deduce that Harris cannot schedule a government building during week 6, because this would not give Limon a chance to inspect it. Nor can Limon schedule a government building in week 1, because Harris would not have had a chance to inspect it. Similarly, because Limon must inspect a commercial building before Harris can inspect it, you can deduce that Limon cannot schedule a commercial building in week 6, because this would not give Harris time to inspect it. Nor can Harris schedule a commercial building in week 1 because Limon would not have had a chance to inspect it. These deductions can be symbolized as follows:

Images

2. Repeated-element deductions

There are no repeated element deductions you can make for this game.

3. Down-to-two deductions

There are no down-to-two deductions you can make for this game.

4. Block-splitting deductions

In effect, the government buildings form a block on Limon’s schedule. They cannot be consecutive and none of them can be scheduled for week 1. Therefore, they must be scheduled for weeks 2, 4, and 6, and Limon’s commercial buildings must be scheduled for weeks 1, 3, and 5. Instead of symbolizing this deduction you will note it at the bottom of the diagram, keeping in mind this is a deduction for Limon’s schedule and not Harris’s.

You add your deductions to the diagram so that it now looks as follows:

Images

STEP 6:  Answer the questions in the smartest order.

Answer the questions in the following order:

1. Answer the Complete and Accurate List question.

2. Answer questions that give you more information to work with.

3. Answer the remaining questions.

Questions 13 and 18 are both Complete and Accurate List questions and should be answered first. Questions that give more information include the following:

Question 14 (“Harris inspects building H during week 3 and Limon inspects building G during week 6 …”)

Question 15 (“Harris inspects building O during week 2 …”)

Question 17 (“Limon inspects M during week 1 and I during week 2 …”)

That leaves questions 16 and 19, and they can be answered in that order.

THE ANSWERS

13. Answer: A

The question asks which of the schedules is acceptable. You must analyze each schedule to see whether it is inconsistent with your clues. For answer A, you notice that Harris inspects every government building a week before Limon, and Limon inspects every commercial building a week before Harris. Also, Limon does not inspect two government buildings consecutively. Therefore, this is the correct answer, but for learning purposes let’s evaluate the remaining answers. Answer B is an unacceptable schedule because Harris inspects commercial building N before Limon, and this is inconsistent with clue 2. Answer C is an unacceptable schedule because it does not have Harris inspecting building M in week 4 and this is inconsistent with clue 4. Answer D is unacceptable because Limon inspects government building I before Harris and this is inconsistent with clue 2. Lastly, answer E is unacceptable because Limon inspects government buildings J and K consecutively, and this is inconsistent with clue 3. The correct choice is answer A.

14. Answer: D

The question gives you new information, that Harris inspects building K in week 3 and Limon inspects building J in week 6. You add this new information to your diagram, and you deduce certain requirements that this scenario creates. If Harris inspects K in week 3, then Limon must schedule K for week 4, because he must inspect K after Harris inspects it. Also, Limon cannot schedule K for week 5 because Limon cannot have two government buildings scheduled consecutively. Therefore, Limon must schedule I for week 2. He cannot schedule I for week 3 or 5 because I and K or I and J would be consecutive, and he cannot schedule I for week 1 because Harris must have at least a week before to inspect a government building. You now evaluate your answers. Answer A cannot be correct because Limon can easily schedule N in week 1 or 3, which still gives Harris enough time to inspect it after him. Answer B is completely inconsistent with clue 4. Answer C cannot be the answer because Harris does not have to schedule J during week 2. He can also schedule it during week 5 because that is still before the week that Limon has scheduled it. Answer D, as you know from your deductions, must be true. Harris must inspect I during week 1. This is your answer. Answer E cannot be correct because Harris can also schedule O during week 2 and 5, both of which give Limon advance time to inspect it before him.

Images

The correct choice is answer D.

15. Answer: C

The question tells you that Harris inspects O in week 2 and asks which of the buildings Limon must inspect in week 5. You add this new information to your diagram and see what deductions you can make with regard to which building Limon must inspect in week 5. You can deduce that if Harris inspects O in week 2, then Limon must have inspected O in week 1, because Limon must inspect each commercial building before Harris and week 1 is the only week available for Limon to inspect building O. Your deduction tells you that the government buildings are in weeks 2, 4, and 6. This means that Limon will have to inspect M in week 3 in order to inspect it before Harris and remain consistent with clue 2. That leaves N as the one remaining building Limon must schedule, and it must be in week 5. This is your answer. You do not have to evaluate the other answers.

Images

The correct choice is answer C.

16. Answer: D

The question asks you to identify among the possible answers the one statement that must be true. This means that all but one of the answers may not be true. If you give the answers a quick read, you can quickly see that answer D must be true because you already deduced that Limon must inspect a commercial building in week 3, but for learning purposes let’s review the remaining answers. The following diagram shows a scenario for each answer that proves it does not have to be true. Answer A shows that Harris can inspect two government buildings consecutively and the scheduling can still be consistent with the rules of the game. Answer B shows that Harris can inspect J before N. Answer C shows that Harris can inspect a government building during week 2. Lastly, answer E shows that Limon can inspect a government building other than K during week 6.

Images

The correct choice is answer D.

17. Answer: B

The question tells you that Limon inspects building M in week 1 and I during week 2. It asks that you identify the one possible answer that could be true. This means that all but one of the answers must be false. You add the new information to your diagram. You can use clue 1 to deduce that Harris inspects I during week 1. Now you evaluate your possible answers. For answer A, Limon does not inspect his two remaining commercial buildings until week 3. Therefore, Harris cannot do his two remaining commercial buildings (N and O) until weeks 5 and 6. Harris must inspect his two remaining government buildings (J and K) during weeks 2 and 3, which means he must inspect two consecutive government buildings and answer A must be false. Using the same logic, answer B could be true. One of the commercial buildings Harris inspects during week 5 or 6 can be O, so answer B could be true and this is your answer. We’ll analyze the remaining answers for learning purposes. Answer C cannot be true because, based on your initial deductions, Harris cannot inspect a government building in week 5. Answer D cannot be true because you have already deduced that Harris’s first inspection must be I so that Limon can inspect I during week 2. Finally, answer E cannot be true because clue 4 tells you that Harris must schedule M for week 4.

Images

The correct choice is answer B.

18. Answer: D

The question asks you to identify from among the possible answers the complete and accurate list of the weeks during which Harris must inspect government buildings. This means that all but one of the answers include weeks that may or may not be scheduled for a government building for Harris. The only clue that affects when Harris inspects a government building is the requirement that he inspect it before Limon, so Limon’s schedule will most likely be the determining factor in your consideration of each answer. Your initial deductions tell you that Limon must inspect a government building during weeks 2, 4, and 6. From this knowledge you can determine that in week 3 Harris must inspect the building that Limon inspects in week 1. Therefore, whichever answer is correct, it must include week 1. This deduction allows you to eliminate answer E, which does not include week 1. Determining the remaining weeks during which Harris must inspect a government building means determining when he must inspect the government buildings Limon inspects during weeks 4 and 6. Harris must inspect the week 4 building during either week 2 or week 3, but it does not really matter which. Regarding the week 6 building, Harris can inspect this in week 2, 3, or 5. All of these weeks come before the week Limon inspects the building and do not cause conflicts with the scheduling of commercial buildings after Limon inspects them. Because the other two buildings have such flexibility, week 1 is the only week that he must inspect a government building. The correct choice is answer D.

19. Answer: A

The question asks which among the answers could be true. This means all but one of the possible answers must be false. You analyze each answer to determine whether a schedule could exist with the stated fact. Luckily, the first answer turns out to be your answer. Answer A could indeed be true because K is a government building and your initial deductions tell you that Limon can schedule only government buildings in weeks 2, 4, and 6. Answer B cannot be true because I is a government building and Limon cannot schedule a government building in week 3. Answer C cannot be true because Harris cannot inspect a commercial building until Limon has inspected it. Harris must schedule M for week 4 according to clue 4. Therefore, Limon cannot inspect M after week 4. Answer D cannot be true because O is a commercial building and your deductions tell you that Harris cannot schedule a commercial building during the first week. Such scheduling does not allow Limon to inspect the commercial building before Limon. Answer E cannot be true because I is a government building and your deductions tell you that Harris cannot schedule a government building in week 6 because it does not allow time for Limon to inspect it after Harris. The correct choice is answer A.

Question 20–26

As with all logic games you follow the six-step process.

STEP 1:  Identify the game type.

This is a grouping game, but the terse nature of the descriptions somewhat obscures the game’s nature. Put simply, there are two categories of lumber, domestic and exotic, and they are being allocated to the “On Sale” and “Not on Sale” groups. Within each category there are five different types of wood: cherry, maple, oak, pine, and walnut.

STEP 2:  Begin your diagram.

The diagram will be a simple four-box grid with Domestic and Exotic on one side and On Sale and Not on Sale across the top (you could do it the other way as well). You abbreviate the types of wood as C, M, O, P, and W. Each type of wood will appear in the grid twice, because both domestic and exotic are offered.

Images

STEP 3:  Symbolize the clues.

You symbolize the clues one at a time.

Clue 1: Exotic oak is on sale; domestic maple is not on sale.

These do not need to be symbolized. You will simply put an “O” in the exotic and on sale box, and you will put an “M” in the domestic and not on sale box.

Clue 2: If both types of oak are on sale, then all walnut is on sale.

This can be symbolized using your block format, as follows:

Images

Clue 3: If both types of cherry are on sale, then no pine is on sale.

This can be symbolized using your block format, as follows:

Images

Clue 4: If neither type of cherry is on sale, then domestic oak is on sale.

This can be symbolized using your block format, as follows:

Images

Clue 5: If either type of pine is on sale, then no walnut is.

This can be symbolized using your block format, as follows:

Images

It might be useful to convert this to a contrapositive. You must keep in mind that the right side could be interpreted as meaning that both domestic walnut and exotic walnut are on sale and could be rewritten as follows:

Images

You form the contrapositive as follows:

Images

Notice that terms have switched sides of the arrow and the negation has switched the letters to the opposite boxes. The “or” has become an “and” and vice versa. You can compact the right side into one box as follows:

Images

This says that if either type of walnut is on sale, then both types of pine must not be on sale.

STEP 4:  Double-check your symbolizations.

To double-check your symbolizations, you translate your symbolized clues back into normal English and see whether they match the original language of each clue. Once you confirm that you have the correct symbolizations, you proceed to write them next to your diagram, including the contrapositive for the last clue. Your page should look something like this:

Images

STEP 5:  Make deductions.

1. Can’t-be-first-or-last deductions

This is not an ordering game, so there are none of these types of deductions to be made.

2. Repeated-element deductions

There is a repeated element implied by the clues that is a bit tricky. Clue 1 says that exotic oak is on sale. Clue 4 says that if no type of cherry wood is on sale, then domestic oak is on sale. In essence, this says that if no type of cherry wood is on sale, then all oak is on sale. Clue 2 says that if all oak is on sale, then all walnut is on sale. You can deduce that if both types of cherry wood are not on sale, then both types of walnut are on sale. This deduction could be written as follows:

Images

3. Down-to-two deductions

There are none of these types of deductions to be made.

4. Block-splitting deductions

There are none of these types of deductions to be made.

STEP 6:  Answer the questions in the smartest order.

On test day, answer the questions in this order:

1. Answer the Complete and Accurate List question.

2. Answer questions that give you more information to work with.

3. Answer the remaining questions.

Question 20 is a Complete and Accurate List question and should be answered first. Questions that offer more information are as follows:

Question 21 (“domestic walnut is not on sale …”)

Question 22 (“both types of cherry are on sale …”)

Question 24 (“neither type of cherry is on sale …”)

Question 25 (“domestic walnut is the only type of domestic lumber on sale …”)

Question 26 (“exactly four of the types of exotic lumber are the only lumber types on sale …”)

The only remaining question is 23.

THE ANSWERS

20. Answer: B

The question asks you to identify a complete and accurate list of the types of lumber that are on sale. You must review each answer and test them against your clues. You apply each one to your diagram. You place the letters in the On Sale column and the remaining in the Not on Sale column. With answer A, both types of cherry wood are not on sale and according to clue 4, domestic oak must be on sale, so this grouping is not consistent with the game and cannot be the answer. Answer B is completely consistent with your rules, and it can be a complete and accurate list of the lumber on sale. This is your answer, but let’s review the remaining answers for learning purposes. Answer C cannot be your answer because it is inconsistent with clue 2. Both oaks are on sale so both types of walnut must be on sale, but in this grouping exotic walnut is not on sale. Answer D is inconsistent with clue 5. Exotic pine is on sale and therefore neither walnut can be on sale, but in this grouping domestic walnut is on sale, so this answer cannot be a complete and accurate sale list. Answer E cannot be your answer because both domestic and exotic cherry wood are on sale, which means neither pine type can be on sale, but in this grouping domestic pine is on sale.

Images

The correct choice is answer B.

21. Answer: A

The question tells you that domestic walnut is not on sale and asks which of the possible answers must be true based on this fact. You look first for the clues that affect walnut, which brings you to the second clue. It says that if both types of oak are on sale, then both types of walnut must be on sale as well, but this question says that domestic walnut is not on sale so you know that both oaks cannot be on sale. Clue 1 says that exotic oak must be on sale. Taking both of these clues together, you can deduce that in order for domestic walnut not to be on sale, domestic oak cannot be on sale. Answer A says that at least one type of oak is not on sale, and based on your deductions this must be true, so this is your answer. Neither answer B nor answer C can be correct because according to the clues, whether or not walnut is on sale has no effect on the sale status of the types of cherry wood or maple. Neither answer D nor answer E can be your choice because your clues do not help you understand the sale status of pine if domestic walnut is not on sale. Clue 5 works in only one direction (from pine to walnut). Knowing that one of the walnuts is not on sale tells you nothing about the types of pine. Your contrapositive would help only if the domestic walnut was on sale, but that is not the fact given to you. The correct choice is answer A.

22. Answer: A

The question tells you that both types of cherry wood are on sale, then asks you the minimum number of domestic lumber types that can be included in the sale. You add this information to your diagram and attempt to ascertain what this number might be. Clue 3 tells you that if both cherry woods are on sale, then both pines must be not on sale. The rest of the types of wood can also be put in the Not on Sale column without breaking any of your rules. Remember that having both types of walnut not on sale tells you nothing. This question once again tempts you to make a conclusion based on reversing the “if … then” statement in clue 5. The only domestic wood that must be on sale is cherry wood.

Images

The correct choice is answer A.

23. Answer: E

The question asks which of the possible answers cannot be true. Answer A could be true. The clues tell you nothing about the relationship between cherry and maple. Regardless of whether both types of cherry are on sale, you can put exotic maple on sale or not. Answer B has the same problem. Your clues tell you nothing about a relationship between maple and pine, and therefore it could be true that neither type of maple and neither type of pine is on sale. If both types of cherry are on sale, clue 4 tells you that domestic oak must be on sale, but it says nothing about pine. Clue 3 tells you only what happens if both cherry woods are on sale, not when they are not on sale, so answer C cannot be correct. Answer D has the same problem as answers A and B. There is no relationship between maple and any of the other woods, so this scenario could be true. Finally, consider answer E. This answer conflicts with your deduction that if neither type of cherry is on sale, then both walnut types must be on sale. This answer cannot be true and must be your answer. The correct choice is answer E.

24. Answer: E

The question tells you that neither cherry type is on sale and asks you to determine which statement among the possible answers is not necessarily true. You create the scenario using your diagram. Because both cherry types are not on sale, clue 4 tells you that domestic oak is on sale. Because both oak types are now on sale, clue 2 tells you that both walnuts are on sale. Your contrapositive tells you that because either the domestic walnut or the exotic walnut is on sale, both pines are not on sale. This leaves only one type undetermined, exotic maple. Based on the preceding logic, answers A, B, C, and D all must be true. Only answer E may or may not be true. Exotic maple can be either on sale or not on sale. Nothing in this scenario tells you one way or the other.

Images

The correct choice is answer E.

25. Answer: B

The question tells you that domestic walnut is the only type of domestic lumber on sale and then asks you to determine which among the possible answers cannot be true. This means that all but one of the options are possible. You use your diagram to visualize the scenario. Answer A could be true. If exotic walnut is not on sale, you can still maintain domestic walnut as the only domestic wood on sale. No clue changes this requirement. Regarding answer B, if exotic cherry is not on sale, then both cherries are not on sale and clue 4 requires that domestic oak be on sale. This would mean that walnut would not be the only domestic wood on sale, and that goes against the information given by the question. This is your answer. Answer C is true because the very fact that domestic walnut is on sale triggers the contrapositive of clue 5. Exotic maple is one wood that is not restricted by the clues of this game, so it can be on sale or not on sale. It does not matter. In this particular question, exotic walnut is not restricted by the clues either and can be on sale or not on sale. Therefore, neither answer D nor answer E can be correct.

Images

The correct choice is answer B.

26. Answer: D

The question tells you that four of the five types of exotic lumber are the only lumber types on sale. It then asks you to determine which of the possible answers could be true. This means that all but one of the possible answers is impossible given the clues in the game. Because the exotic woods are the only ones on sale, there can be no domestic woods on sale. Everything else is not on sale. You test each one on your diagram. Answer A cannot be the correct choice because clue 4 tells you that when both cherry woods are not on sale, then domestic oak must be on sale. Answer B cannot be correct because you have only five types of exotic woods. If neither exotic pine nor exotic walnut is on sale, that leaves only three exotic woods on sale. Answer C has the same problem as answer A. If exotic cherry is not on sale, then both types of cherry wood are not on sale, and clue 4 tells you that if this is the case, then domestic oak is on sale. Answer D could be true. This grouping is consistent with all of the clues, including clue 5, which is triggered by this configuration. This is your answer. Answer E cannot be true because it is inconsistent with clue 5. If exotic maple is not on sale, then exotic pine must be. However, because pine is on sale, then both types of walnut must not be on sale, and that means you would have only three exotic woods on sale, not the four required.

Images

The correct choice is answer D.